You are on page 1of 91

MATHEMATICS TODAY MAY ’18 3

4 MATHEMATICS TODAY MAY ’18


MATHEMATICS TODAY MAY ’18 5
6 MATHEMATICS TODAY MAY ’18
10 31
19

41

Vol. XXXVI No. 5 May 2018


27
Corporate Office:
Plot 99, Sector 44 Institutional Area,
Gurgaon -122 003 (HR), Tel : 0124-6601200
e-mail : info@mtg.in website : www.mtg.in
Regd. Office: 77
406, Taj Apartment, Near Safdarjung Hospital, 68
Ring Road, New Delhi - 110029.
Managing Editor : Mahabir Singh
57
Editor : Anil Ahlawat

8
CONTENTS
74 84

63
8 Maths Musing Problem Set - 185
10 Practice Paper - JEE Advanced 60
19 Solved Paper - JEE Main
Competition Edge

27 Olympiad Corner Subscribe online at www.mtg.in


31 Target JEE
TO OUR READERS
41 Practice Paper - JEE Advanced
We are happy that intelligent students, teachers and other professionals
55 Math Archives continue to patronise Mathematics Today, Chemistry Today, Physics For You

57 Mock Test Paper - ISI and Biology Today.


To them, we are addressing this open letter in view of increase in the cost
60 You Ask We Answer of production and postage in the last seven years. All round spiralling prices
have pushed production costs so high, that many in out fraternity find it
62 Maths Musing Solutions impossible to continue business. We are compelled to raise the price to
` 40 from July 2018 issue.
68 Ace Your Way We understand the pressure of cost on the student-teacher community in
(Series 1) general but, we are hoping our readers will understand our problems and
Class XI

that we have no option but to comply with this unavoidable move.


46 Concept Map We on our part, will keep up our efforts to improve the magazines in all
its aspects.
74 MPP-1
47 Concept Map Printed and Published by Mahabir Singh on behalf of MTG Learning Media Pvt. Ltd. Printed
at HT Media Ltd., B-2, Sector-63, Noida, UP-201307 and published at 406, Taj Apartment, Ring

63 Solved Paper - CBSE Board Road, Near Safdarjung Hospital, New Delhi - 110029.
Class XII

Editor : Anil Ahlawat


Readers are adviced to make appropriate thorough enquiries before acting upon any advertisements
77 Ace Your Way published in this magazine. Focus/Infocus features are marketing incentives. MTG does not vouch or
subscribe to the claims and representations made by advertisers. All disputes are subject to Delhi
(Series 1) jurisdiction only.
Copyright© MTG Learning Media (P) Ltd.

84 MPP-1 All rights reserved. Reproduction in any form is prohibited.

MATHEMATICS TODAY | MAY‘18 7


M aths Musing was started in January 2003 issue of Mathematics Today. The aim of Maths Musing is to augment the chances of bright
students seeking admission into IITs with additional study material.
During the last 10 years there have been several changes in JEE pattern. To suit these changes Maths Musing also adopted the new
pattern by changing the style of problems. Some of the Maths Musing problems have been adapted in JEE benefitting thousand of our
readers. It is heartening that we receive solutions of Maths Musing problems from all over India.
Maths Musing has been receiving tremendous response from candidates preparing for JEE and teachers coaching them. We do hope
that students will continue to use Maths Musing to boost up their ranks in JEE Main and Advanced.

Set 185
JEE MAIN COMPREHENSION
1. If z and w are two non-zero complex numbers such Mr. X is a teacher of mathematics. His students want
π to know the ages of his son’s S1 and S2. He told that
that |zw| = 1 and arg (z) – arg (w ) = , then zw is
equal to 2 their ages are ‘a’ and ‘b’ respectively such that f(x + y)
(a) 1 (b) –1 (c) i (d) – i – axy = f(x) + by2 " x, y ∈ R. After some time students
said that information is insufficient, please give more
 a a 2  3
2. If sin–1  a − + + ...  + cos −1(1 + b + b2 + ...) information. Teacher says that f(1) = 8 and f(2) = 32.
 3 9  7. The age of S1 & S2 will be respectively
π (a) 4, 16 (b) 8, 16 (c) 16, 8 (d) 32, 8
= , then
2 8. The function f(x) is
2a − 5 3a − 2
(a) b = (b) b = (a) even (b) odd
3a 2a
(c) neither even nor odd
3 2
(c) a = (d) a = 3 − 2b (d) periodic as well as odd
2 − 3b INTEGER TYPE
x + 2 y + 6 z − 34
3. The point on the line = = 9. A triangle is formed by the lines x + y = 1 and the
2 3 − 10
x+6 y−7 z −7 two common tangents of the circles x2 + y2 – 2x = 0
which is nearest to the line = = and x2 + y2 – 4x – 2y + 1 = 0. Its area is
4 −3 −2
is (a, b, c), where a + b + c = MATRIX MATCH
(a) 9 (b) 10 (c) 11 (d) 12
10. Match the curves given in Column-I with the
4. C0Cr + C1Cr + 1 + C2Cr + 2 + … + Cn – r Cn = differential equations satisfied by the curves in
 2n   2n   2n   2n  Column-II.
(a) (b) (c) (d)
 r   r − 1  r + 1  n − r  Column-I Column-II
(P) The circles which touch (1) (x2 – y2)y1 = 2xy
5. If f ′(x) = 2(x –1) (x – 2)3 + 3(x – 1)2 (x – 2)2, then the x-axis at the origin
f (x) has minimum at x = (Q) The circles which touch (2) 2xyy1 = y2 – x2
(a) 1 (b) 7/5 (c) 2 (d) –2 the y-axis at the origin
(R) The circle of unit radius (3) (1 – x2) y12 = x2
JEE ADVANCED with centres on the x-axis
6. Given that x ∈ [0, 1] and y ∈ [0, 1]. Let A be the (S) The circles of unit radius (4) (1 + y12)y2 = 1
event of (x, y) satisfying y2 ≤ x and B be the event of with centres on xy-axis
(x, y) satisfying x2 ≤ y. Then, P Q R S
1 (a) 1 2 3 4
(a) P ( A ∩ B) = (b) A, B are exhaustive (b) 1 2 4 3
3
(c) A, B are mutually exclusive (c) 3 4 1 2
(d) A, B are independent (d) 4 3 2 1
See Solution Set of Maths Musing 184 on page no. 62
8 MATHEMATICS TODAY | MAY‘18
on
Exathm ay
M
20

*ALOK KUMAR, B.Tech, IIT Kanpur


SINGLE OPTION CORRECT TYPE 104
(a) 103 (b)
 π 3 3
1. f :  0,  → A, f (x ) = log e (sin x sin x + 1), then
 2 (c) 106 (d) none of these
the minimum value of f(x) is   1 1/e  3
(a) loge2 (b) log e    + 1 7. The set of values of a for which inequation
  e  
(c) loge((e)2 + 1 (d) 2 (a – 1)x2 – (a + 1)x + a – 1 ≥ 0 is true for all x ≥ 2 is
2. If x5 – x3 + x = a, then minimum value of x6 is  7
(a) 1,  (b) (–∞, 1)
(a) a (b) 2a – 1 (c) a + 2 (d) a – 3  3
3. If z be a complex number satisfying, 7 
(c)  , ∞  (d) none of these
|z|2 + 2(z + z) + 3i(z – z) + 4 = 0 then complex number 3 
z + 3 + 2i will lie on 8. If x1, x2, x3, ..., xn are the roots of xn + ax + b = 0, then
(a) circle with centre 1 – 5i and radius 4 the value of (x1 – x2)(x1 – x3)(x1 – x4)...(x1 – xn) is
(b) circle with centre 1 + 5i and radius 4 (a) n1n (b) nx1n – 1 + a
(c) circle with centre 1 + 5i and radius 3 (c) nx1 (d) nx1 + an
(d) circle with centre 1 – 5i and radius 3
1/n
4. If f(x – y) = f(x)g(y) – f(y)g(x) " x, y ∈ R, (where  n! 
f(x) is not identically zero), then 9. The value of lim   is
n→∞  (mn)n 
(a) If f ′(0+) exists then f ′(0–) also exists but not equal
e
(b) If f ′(0+) exists then f ′(0–) does not exist (a) em (b)
(c) If f ′(0+) exists then it is equal to f ′(0–) m
(c) 1 (d) none of these
(d) none of these em
5. A quadratic polynomial maps from [–2, 3] onto 10. If one of the roots of the equation
[0, 3] and touches x-axis at x = 3, then the polynomial is
7 6 x 2 − 13
3 2 3 2
(a) (x − 6 x + 16) (b) (x − 6 x + 9) 2 x 2 − 13 2 = 0 is x = 2, then sum of all
16 25
(c)
3 2
(d) none of these x 2 − 13 3 7
(x − 6 x + 16)
25 other five roots is
x
6. Let f ( x ) = ∫ (t + 2t + 2) dt ,
2
where x is (a) 2 15 (b) –2
0 (c) 20 + 15 − 2 (d) none of these
set of real numbers satisfying the inequation
11. A unit vector is orthogonal to 5iˆ + 2 jˆ + 6kˆ and is
log 2 (1 + 6 x − x 2 − 8 ) ≥ 0. If f(x) ∈ [a, b] then
coplanar to 2iˆ + jˆ + kˆ and iˆ − jˆ + kˆ , then the vector is
maximum value of |a – b| is
* Alok Kumar is a winner of INDIAN NATIONAL MATHEMATICS OLYMPIAD (INMO-91).
He trains IIT and Olympiad aspirants.
10 MATHEMATICS TODAY | MAY‘18
(a) a1a2 + b1b2 (b) a1b2 + a2b1
3 jˆ − kˆ 2iˆ + 5 jˆ
(a) (b) (c) b1a2 – b2a1 (d) a1a2 – b1b2
10 29
100
6iˆ − 5kˆ 2iˆ + 2 jˆ − kˆ
(c)
61
(d)
3
19. If f (x ) =  x  , then ∫ f (x )dx is equal to (where
0
2 3 [.] denotes the greatest integer function)
12. The range of the function y = 2 {x} − {x} − is (a) 4950 (b) 5000 (c) 4250 (d) 4590
4
(where {.} denotes fractional part)
n2
 1 1  1 20. The largest term of an = 3 , n ∈ N is
(a)  − ,  (b) 0,
 4 4  2  n + 200
29 49 43 41
1 1 (a) (b) (c) (d)
(c) 0, 1  (d)  , 453 543 553 451
 4
  4 2 
         1
dx 21. If r = λ(a × b ) + µ(b × c ) + γ(c × a ) and [a b c ] = ,
8
13. ∫ is equal to then l + m + g is
cos3 x cos(x − α)    
(a) 8(r . a ) (b) 8(r . b ) 
    
(a) 2 tan x sin α + cot α + c (c) 8(r . c ) (d) 8r ⋅ (a + b + c )
(b) 2 c osecα tan x sin α + cos α + c
ONE OR MORE THAN ONE OPTION CORRECT TYPE
(c) 2 sin α + tan x cos α + c
22. If [x] represents the greatest integer less than or
(d) 2cosecα tan x cos α + sin α + c equal to x, then which of the following statement is
14. Inside a big circle exactly n small circles each of true?
radius r can be drawn in such a way that each small (a) sin[x] = cos[x] has no solution
circle touches the big circle and two small circles. If (b) sin[x] = tan[x] has infinitely many solutions
n ≥ 3, then the radius of the bigger circle is (c) sin[x] = cos[x] possess unique solution
π   π  (d) sin[x] = tan[x] for no value of x
(a) r cosec   (b) r 1+ cosec   
n   n  23. If p, q, r are in H.P. and p, q, –2r are in G.P;
  2π     π  (p, q, r > 0) then
(c) r 1 + cosec    (d) r 1 + cosec   
  n    2n   (a) p2, q2, r2 are in G.P. (b) p2, q2, r2 are in A.P.
15. If m1 and m2 be the slopes of two perpendicular (c) 2p, q, 2r are in A.P. (d) p + q + r = 0
chords of equal length passing through origin of circle
d2 + r de df
(x – 1)2 + (y + 2)2 = 5, then the value of m12 + m22 is equal to
2
80 82 24. The det ∆ = de e +r ef is divisible by
(a) (b) 2
9 9 df ef f +r
(c) 83 (d) none of these (a) r (b) (d + e2 + f 2 + r)
9 (c) (d2 + e2 + f 2 + r) (d) (d2 + e + f 2 + r2)
16. If the system of equations x – ky – z = 0, kx – y – z = 0,
25. If both the roots of equation ax2 + x + c – a = 0 are
x + y – z = 0 has a non-zero solutions, then the possible
imaginary and c > – 1, then
values of k are
(a) 3a > 2 + 4c (b) a > 0
(a) –1, 2 (b) 0, 1 (c) 1, 2 (d) –1, 1
(c) c < a (d) None of these
17. Period of |cos|x|| + |sin|x|| is
(a) 2p (b) p 26. The solution of the equation
2 2
π 3sin 2 x +2 cos x + 31−sin 2 x +2 sin x = 28
(c) (d) none of these
2 π π
18. If O is the origin and A and B are respectively (a) (2n + 1) , n ∈ I (b) nπ − , n ∈ I
2 4
(a1, b1) and (a2, b2), then OA ⋅ OB sin(AOB) is equal to (c) 2np (d) none of these

MATHEMATICS TODAY | MAY‘18 11


dy Passage for Q.No. 34 to 36
27. If sin y = x cos(a + y), then is
dx x2
Consider an ellipse + y 2 = α, (a is parameter > 0)
cos2 (a + y ) cos (a + y ) 4
(a) (b) and a parabola y2 = 8x. A common tangent to the ellipse
cos a x sin (a + y ) + cos y
and the parabola meets the co-ordinate axes at A & B.
cos a
(c) (d) none of these 34. Locus of mid point of AB is
3x 2 − 2 x cos a − 1
(a) y2 = –2x (b) y2 = –x
COMPREHENSION TYPE x x2 y2
Passage for Q.No. 28 to 30 (c) y2 = − (d) + =1
2 4 2
A ray of light is coming along the line L = 0 and strikes
35. If the eccentric angle of a point on the ellipse where
the plane mirror kept along the plane P = 0 at B.
 2π 
A(2, 1, 6) is a point on the line L = 0 whose image about the common tangent meets it is   , then a is equal to
 3 
x − 2 y −1 z − 6
P = 0 is A′. It is given that L = 0 is = = (a) 4 (b) 5 (c) 26 (d) 36
3 4 5
and P = 0 is x + y – 2z = 3. 36. If two of the three normals drawn from the
28. Co-ordinates of A′ are point (h, 0) on the ellipse to the parabola y2 = 8x are
(a) (6, 5, 2) (b) (6, 5, –2) perpendicular, then
(c) (6, –5, –2) (d) none of these (a) h = 2 (b) h = 3
29. Co-ordinates of B are (c) h = 4 (d) h = 6
(a) (5, 10, 6) (b) (10, 15, 11) MATRIX MATCH TYPE
(c) (–10, –15, –14) (d) none of these 37. Match the following :
30. If L1 = 0 is the reflected ray, then its equation is List-I List-II
x + 10 y − 5 z + 2 (A) 3x2 + 2(b2 + 1)x + b2 – 3b + 2 = 0
(a) = = 2 
4 4 3 will have roots of opposite sign if (1)  , 1
3 
x + 10 y + 15 z + 14 b∈
(b) = =
3 5 5 (B) The probability of a problem
b eing s olve d by 3 students
(c) x + 10 = y + 15 = z + 14 1 1
4 5 3 are , and α ∈(0, 1). T h e n (2) [ 2 , ∞)
2 3
(d) none of these probability that problem will be
Passage for Q.No. 31 to 33 solved ∈
Given A(3 + 4i), B(–4 + 3i) and C(4 + 3i) be the vertices (C) If 3 real numbers satisfy the
 7
of DABC which is inscribed in a circle S = 0. Let AD, BE, equation x + y + z = 5 and (3) 1, 
xy + yz + zx = 8, then z ∈  3
CF are altitudes through A, B, C which meet the circle
S = 0 at D(z1), E(z2) and F(z3) respectively. (4) (1, 2)
31. Complex number z1 is equal to 38. In a tournament there are twelve players S1, S2, ....,
(a) (2 6 + i) (b) (1 + 2 6i) S12 and divided into six pairs at random. From each
(c) 3 – 4i (d) –3 – 4i pair a winner is decided on the basis of a game played
32. Orthocentre of triangle ABC is at between the two players of the pair. Assuming all the
(a) 2 + 9i (b) 3 + 10i pairs are of equal strength, then match the following :
(c) 3 + 11i (d) none of these List-I List-II
33. The value of z1z2z3 is equal to (A) Probability that S is among the
2 5
(a) 75 + 100i (b) –100 +75i (1)
losers is 22
(c) 100 + 75i (d) none of these

12 MATHEMATICS TODAY | MAY‘18


(B) Probability that exactly one of S 10 (C) If x 3 + ax 2 + x + 1 = 0 is an (3) (0, 4)
3
and S4 is among the losers
(2) increasing function, then a2 may
11
belong to
(C) Probability that exactly one S and 1 (D) If quadratic equation x2 + 3ax + (4) (–3, 3)
2
(3)
S4 are among the winners is 2 a2 – 9 = 0 has roots of opposite
sign, then a belongs to
6
(4) 42. Match the following :
11
List-I List-II
39. Match the following : (A) The set of values of b for which (1) (–∞, 0)
List-I List-II the function f(x) = x2 + bx + 5
is an increasing function on
(A) For a rectangular hyperbola
[1, 3] is
xy = c2 (c is purely imaginary).
If a point on it is (a, a), where (1) 3 (B) India plays 4 matches with Sri (2) 0.2375
‘a’ is a positive number, then a Lanka and South Africa each.
can take value In any match the probability
of getting points 0, 1, 2 are
(B) If sides of a triangle are in A.P.
0.15, 0.35, 0.5 respectively
and (a – b + c)s = kb2 (where s 1− 3
(2) assuming the outcomes are
is the semi perimeter), then k 2 independent. The probability
is equal to
of getting at least 7 points is
(C) Radius of the circle having
(C) The radical centre of 3 circles (3) orthocentre
centre (3, 4) and touching the (3) 17
described on the 3 sides of
circle x2 + y2 = 4 can be
a triangle ABC as diameter
3 is the _____ of the triangle
(4)
2 ABC.
(D) The domain of the function (4) [–2, ∞)
40. Match the following : 1
List-I List-II f (x ) = is
x −x
(A) If x, y, z ∈ N, then number of (1) 19
ordered triplet (x, y, z) satisfying INTEGER ANSWER TYPE
xyz = 243 is
43. In DABC if BC is unity,
(B) The numb er of ter ms in t he (2) 28 A B A B
expansion of (x + y + z)6 is sin = x1 , sin = x2 , cos = x3 and cos = x 4
2 2 2 2
(C) If x ∈ N, then number of solutions (3) 21 2007 2006
of x2 + x – 400 ≤ 0 is with  x1  x 
 x  − 3  = 0, then the length of AC
(D) If x, y, z ∈ N, then number of (4) 36 2  x4 
solution of x + y + z = 10 is

41. Match the following : 44. If ln = ∫ e − x (sin x )n dx (n > 1), then the value of
List-I List-II 0
2
(A) If inequation x – ax + a > 0 " (1) [0, 4) 101l10
is equal to
x ∈ R, then a belongs to 90l8
(B) a (2) [0, 3] 45. The A.M., G.M. and H.M. of first and last term of
If x 3 − 3x + = 0 has three real
2 the series 100, 101, 102,...., n are also the term of this
and distinct root, then |a| belong n
to series, then the value of (100 < n ≤ 500) is
200

MATHEMATICS TODAY | MAY‘18 13


SOLUTIONS x3 8 32
1. (b) : Let h(x) = log(xx + 1), for 0 < x < 1, h′(x) > 0 f (x ) = + x 2 + 2 x , α = f (2) = + 4 + 4 =
3 3 3
1 1 64 136 104
if x > . If x > , for 0 < x < 1, function will be β = f (4) = + 16 + 8 = , α − β max =
e e 3 3 3
increasing, then it will be one-one for x ∈(0, 1).
1 1 7. (c) : Given inequation is (a – 1)x2 – (a + 1)x +
h′(x) < 0, if x < , so, at x = , there is minimum. a–1≥0
e e
⇒ a(x2 – x + 1) – (x2 + x + 1) ≥ 0
1 1/ e 
f min = log e    + 1 . x2 + x + 1 2x 2
  e   ⇒ a≥ =1+ =1+ ...(i)
x2 − x + 1 x2 − x + 1 1
x + −1
x(x 6 + 1) x
2. (b) : x(x 4 − x 2 + 1) = . Suppose x > 0, a > 0, 1
x2 + 1 Let y = x + , y is increasing in [2, ∞)
a(x 2 + 1) 1 x

x6 + 1 = = a  x +  ≥ 2a, x 6 ≥ 2a − 1. 2  7
x  x ⇒ 1+ ∈1,  for all x ≥ 2 equation (i)
If x ≤ 0 ⇒ a ≤ 0, 2a – 1 < 0 ≤ x6 ⇒ x6 ≥ 2a – 1. 1  3
x + −1
3. (c) : Given equation of circle is x
|z|2 + z(2 + 3i) + z(2 – 3i) + 4 = 0 centre –(2 – 3i), should be true .
7
radius = aa − b = 3 ⇒ a≥ .
Let w = z + 3 + 2i = z + 2 – 3i + 1 + 5i, 3
|w – 1 – 5i| = |z + 2 – 3i| = 3 8. (b) : Given xn + ax + b = 0 = (x – x1)(x – x2)(x – x3)
so, w lies on circle whose centre is 1 + 5i, radius = 3. ... (x – xn)
4. (c) : f(x – y) = f(x)g(y) – f(y)g(x) ...(i) x n + ax + b
⇒ (x − x2 )(x − x3 )...(x − xn ) =
Put x = y in (i), f(0) = 0, x − x1
Put y = 0 in (i), f(x) = f(x) g(0) – f(0) g(x) ⇒ g(0) = 1 \ (x1 – x2) (x1 – x3)(x1 – x4) ... (x1 – xn)
f (0 + h) − f (0)
Now, f ′(0+ ) = lim  x n + ax + b  n−1
h→0 h = lim   = nx1 + a.
x → x1  x − x1 
f (0) g (−h) − f (−h) g (0) f ( − h)
= lim = lim = f ′(0− ) 1 1 2 3 n 
1/n
h→0 h h → 0 −h 9. (c) : Let L = lim  . . ... 
i.e. f ′(0+) = f ′(0–). n→∞ m  n n n n 
5. (b) : Let f(x) = ax2 + bx + c.   1  1 1 2 n 
As it touches x-axis at x = 3 ⇒ ln L = lim  ln   +  ln + ln + ...... + ln  
n→∞   m  n  n n n 
−b
⇒ = 3 ⇒ b = −6a ...(i)
2a 1 n r 
also, 9a + 3b + c = 0 ...(ii) = − ln m + lim ∑ ln  
n→∞ n r =1  n 
4a – 2b + c = 3 ...(iii) 1
Solving (i), (ii) and (iii), we get  1  1
= − ln m + ∫ ln xdx = − ln m − 1 = ln   ∴ L = .
3 18 27  em  em
a= ,b=− ,c= 0
25 25 25 10. (b) : Let x 2 – 13 = t, then t 3 – 67t + 126 = 0
3 2
⇒ f (x ) = (x − 6 x + 9) ⇒ t = −9, 2, 7 ⇒ x = ±2, ± 20 , ± 15 .
25

11. (a) : A vector orthogonal to c and coplanar with
6. (b) : log 2 (1 + 6 x − x 2 − 8 ) ≥ 0     
a and b is c × (a × b )
⇒ 1 + 6x − x 2 − 8 ≥ 1   
Here, c = (5, 2, 6) , a = (2,1,1) , b = (1, −1,1)
⇒ 6x − x 2 − 8 ≥ 0 ⇒ x 2 − 6x + 8 ≤ 0   
Then c × (a × b ) = (0, 27, − 9)
⇒ (x – 2)(x – 4) ≤ 0 ⇒ 2 ≤ x ≤ 4
3 jˆ − kˆ
Now f ′(x) = x2 + 2x + 2 > 0 " x ∈ R \ Unit vector is
⇒ f (x) is strictly increasing in [2, 4] 10

14 MATHEMATICS TODAY | MAY‘18


3 ⇒ a12 + b12 a22 + b22 sin(θ1 − θ2 ) = b1a2 − b2a1
12. (b) : Set {x} = t so that 0 ≤ t < 1, y = −t 2 + 2t −
4
1 ⇒ OA⋅OB sin(AOB) = b1a2 – b2a1.
The domain of definition is ≤ t < 1
2
1 2 3 100
1
  1 3 3 1 1 99 × 100
y   = − + 1 − = 0 ; y(1) = −1 + 2 − = = 19. (a) : ∫ 0dx + ∫ 1dx + ∫ 2dx + ... + ∫ 99dx =
2 4 4 4 4 2 2
0 1 2 99
1 1
But is not attained, so the range is 0,  . x2
= 4950
2  2 20. (b) : Let f (x ) = 3 ,x ≥1
x + 200
dx x(400 − x 3 )
13. (b) : Let I = ∫ f ′(x) = 3
cos3 x (cos x cos α + sin x sin α ) (x + 200)2
sec2 xdx f ′(x) > 0 if 0 < x < 3
400 and f ′(x) < 0 if x > 3
400
=∫ , Put tan x sin α + cos α = t 2
cos α + tan x sin α ⇒ f(x) has a maximum at x = 3 400 , 7 < 3 400 < 8
2tdt 2tdt
⇒ sec2 xdx = ∴ I=∫ = 2cosecα . t ⇒ Either a7 or a8 is largest term.
sin α sin α . t
49 8
a7 = , a = , a > a8
= 2cosecα tan x sin α + cos α + c 543 8 89 7
49
π r π ∴ largest term =
14. (b) : We have, sin   = ⇒ R − r = rcosec   543
n R−r n
          
21. (d) : [a b c ] = a ⋅(b × c ) = b ⋅ (c × a ) = c ⋅(a × b )
  π 
∴ R = r 1 + cosec   
  n  a1 a2 a3
m+2 = b1 b2 b3 = D
15. (b) : Let slope of chord, OA = m, tan 45° =
1 − 2m c1 c2 c3
1          1
⇒ m + 2 = 1 − 2m ⇒ m + 2 = ± (1 − 2m) ⇒ m = 3 or − Given r = λ(a × b ) + µ(b × c ) + γ(c × a ), [a bc ] =
3 8
64
⇒ m12 + m22 = (m1 + m2 )2 − 2m1m2 = + 2 =
82   
Multiply the above relation with a , b , c , we get
9 9
   µ
16. (d) : Set of homogeneous equations will have non- r . a = λ . 0 + µ[a b c ] + 0 =
8
trivial solution if D = 0  
⇒ 8(r ⋅ a ) = µ
1 −k −1    
Similarly g = 8(r ⋅ b ), l = 8(r ⋅ c )
⇒ k −1 −1 = 0    
\ l + m + g = 8r ⋅ (a + b + c )
1 1 −1

1 −k 0 EXAM CORNER 2018


⇒ k −1 k − 1 = 0 [Applying C3 → C3 + C1]
Exam Date
1 1 0
MHT CET 10th May
⇒ k2 – 1 = 0 ⇒ k = ±1
COMEDK (Engg.) 13th May
π 
17. (c) : f (x ) = f  + x  .
2  AMU (Engg.) 13th May
18. (c) : sin(q1 – q2) = sinq1 cosq2 – cosq1 sinq2
BITSAT 16th to 31st May
b1 a2 a1 b2
= ⋅ − ⋅
JEE Advanced 20th May
a12 + b12 a22 + b22 a12 + b12 a22 + b22

MATHEMATICS TODAY | MAY‘18 15


22. (a, b) : Since, according to option (a), we have
dy cos2 (a + y )
sin[x] = cos[x] ⇒ = ...(ii)
dx cos a
π  1
⇒ [x] = nπ +   =  n +  π cos2 (a + y ) cos(a + y )
4  4 = =
Which is not possible as LHS ∈ Z and RHS ∈ Q. cos y cos(a + y ) + sin y sin(a + y ) cos y + x sin(a + y )
Hence, sin[x] = cos[x] has no solution. [Using (i)]
However, sin[x] = tan[x] ⇒ [x] = np, n ∈ I which is 28. (b) : Let A′(x2, y2, z2) be image of A(2, 1, 6) about
possible when [x] = 0 and n = 0 \ 0 ≤ x < 1. mirror x + y – 2z = 3, then
23. (b, c) : p, q, r are in H.P. ⇒ q =
2 pr x2 − 2 y2 − 1 z2 − 6 −2 (2 + 1 − 12 − 3)
p+r = = = =4
1 1 −2 12 + 12 + 22
⇒ q2(p + r)2 = (2pr)2
⇒ (x2 , y2 , z2 ) = (6, 5, − 2)
Also, q2 = – 2pr ⇒ q2(p + r)2 = q4
y −1 z − 6
⇒ q2[(p + r)2 – q2] = 0 ⇒ (p + r + q)(p + r – q) = 0, 29. (c) : Let x − 2 = = = λ,
⇒ p+r=q ( p + q + r ≠ 0) 3 4 5
q \ x = 2 + 3l, y = 1 + 4l, z = 6 + 5l, lies on plane
∴ p, , r are in A.P. or 2p, q, 2r are in A.P. x + y – 2z = 3
2
\ 2 + 3l + 1 + 4l – 2(6 + 5l) = 3
Now, q2[p2 + r2 + 2pr] = q4 ⇒ q2[p2 + r2 – q2] = q4
⇒ 3 + 7l – 12 – 10l = 3 ⇒ –3l = 12 ⇒ l = – 4
⇒ [p2 + r2 – 2q2] = 0
\ Point B ≡ (–10, –15, –14).
or p2 + r2 = 2q2 \ p2, q2, r2 are in A.P.
30. (c) : Equation of reflected ray L1 = 0 is a line joining
24. (a, c) : Multiply R1, R2, R3 by d, e, f and take out d, e, f
A′(6, 5, –2) and B(–10, –15, –14)
d2 + r d2 d2 x + 10 y + 15 z + 14 x + 10 y + 15 z + 14
i.e. = = ⇒ = =
common from C1, C2, C3, then ∆ = e2 e2 + r e2 16 20 12 4 5 3

f2 f2 f 2 +r (−4 + 3i)(4 + 3i)


31. (c) : z1 = − = 3 − 4i
Operate R1 → R1 + R2 + R3 and take out d2 + e2 + f 2 + x 3 + 4i
common from R1, then 32. (b) : Circumcentre of DABC = (0, 0).
1 1 1 3 + 10i
Now, centroid of ∆ABC =
3
∆ = e2 e2 + r e2 (d 2 + e 2 + f 2 + r ) \ Orthocentre of DABC = 3 + 10i.
f2 f2 f 2 +r 33. (a) : z1z2z3 = –zazbzc = –(3 + 4i)(–4 + 3i)(4 + 3i)
= (3 + 4i)(4 – 3i)(4 + 3i) = (3 + 4i)25 = 75 + 100i.
= r 2 (d 2 + e 2 + f 2 + r ).
(34-36) : 34. (b) 35. (d) 36. (d)
25. (b) : Here f(1) = a + 1 + c – a. Equation of tangent to y2 = 8x is yt – x – 2t2 = 0 ...(1)
As c > –1 ⇒ f(1) > 0
Equation of tangent to ellipse is x cos θ + y sin θ = 1 ...(2)
26. (a, b) : We have, 2 α α
2 2 27 If the tangent meets the coordinate axes at A and B
3sin 2 x +2 cos x + 31−sin 2 x +2 sin x = 28, y + = 28,
y 2 α   α 
⇒ y2 – 28y + 27 = 0 ⇒ y = 1 then A is  , 0  and B is  0, .
 cos θ   sin θ 
( y ≠ 27, since sin2x + cos2x = 2 )
\ sin2x + 2cos2x = 0 ⇒ 2cosx(sinx + cosx) = 0 Let mid point of AB is (h, k) then,
π π α α
⇒ x = (2n + 1) or x = nπ − h= ,k = , h = − t 2 , k = t ⇒ k 2 = − h or y 2 = − x
2 4 cos θ 2 sin θ
27. (a, b) : sin y = x cos(a + y ) ⇒
sin y
= x ...(i) α −4 α 4 sin2 θ
k2 = –h ⇒ = ⇒ α = − =6
cos(a + y ) sin2 θ cos θ cos θ
dx cos y cos(a + y ) + sin y sin(a + y ) cos a Any normal to parabola is
= =
dy 2
cos (a + y ) cos2 (a + y ) y = mx – 4m – 2m3 ⇒ 2m3 + (4 – h)m = 0 ⇒ h = 6

16 MATHEMATICS TODAY | MAY‘18


MATHEMATICS TODAY | MAY‘18 17
37. (A) → (4), (B) → (1), (C) → (3) 40. (A) → (3), (B) → (2), (C) → (1), (D) → (4)
(A) The equation will have roots of opposite sign (A) xyz = 35 number of solution (x, y, z) is
if it has real roots and product of roots is negative 3+5–1
C5 = 21
b2 − 3b + 2 (B) Number of terms = 6+3–1C3–1 = 28.
4(b2 + 1)2 – 12(b2 – 3b + 2) ≥ 0 and <0
(C) x2 + x – 400 ≤ 0 ⇒ x(x + 1) ≤ 400
3
⇒ 1<b<2
\ The numbers which satisfied the equation are
(B) The probability of problem being solved
1, 2, 3, ..., 19. Thus, number of solutions = 19.
 1 1
= 1 − P ( A). P (B ). P (C) = 1 − 1 −  1 −  (1 − α ) (D) x + y + z = 10
 2 3 number of solutions = coefficient of t10 in (t + t2 + ...+t10)3
2 α 2  3
= + ∈  , 1 10 3
 1 − t10 
3 3 3  = coefficient of t in t  
(C) Here, x = 5 – (y + z) ...(i)  1− t 
Now, yz + x(y + z) = 8 ⇒ yz + (y + z)(5 – (y + z)) – 8 = 0 = coeffieicent of t7 in (1 + 3C1t + 4C2t2 + ..) = 9C7 = 36.
[Using (i)] 41. (A) → (3), (B) → (1), (C) → (2), (D) → (4)
2 2
⇒ y + y(z – 5) + (z – 5z + 8) = 0 (A) x2 – ax + a > 0 " x ∈ R, D < 0 ⇒ a2 – 4a < 0,
 7 7
For real solution, ( z − 1)  z −  ≤ 0, 1 ≤ z ≤ . a ∈(0, 4).
 3 3 a
38. (A) → (3), (B) → (4), (C) → (1) (B) Let f (x ) = x 3 − 3x + , f ′(x ) = 3x 2 − 3 = 0 ⇒ x = ± 1
2
11 ⇒ f(1)f(–1) < 0.
C5 1
(A) =  a a a a 
12
C6 2 ⇒ 1 − 3 +   −1 + 3 +  < 0 ⇒  − 2   + 2  < 0
 2 2 2 2 
(B) Let E1 be the event that S3 and S4 are in same group a
and E2 be the event that S3 and S4 are in different group ⇒ − 2 < < 2 ⇒ a ∈(−4, 4) ⇒ a ∈[0, 4)
2
1 10 (C) Let f(x) = x3 + ax2 + x + 1, f ′(x) = 3x2 + 2ax + 1,
\ P ( E1 ) = , P ( E2 ) =
11 11 f ′(x) ≥ 0 ⇒ 4a2 – 12 ≤ 0 ⇒ a2 ≤ 3
Let E be the event that exactly one of S3 and S4 is (D) x2 + 3ax + a2 – 9 = 0
among the losers, then Product of roots = a2 – 9 < 0 ⇒ a2 < 9 ⇒ a ∈ (–3, 3)
E  E  1 10
P ( E ) = P ( E1 ) P   + P ( E2 ) P   = × 1 + 42. (A) → (4), (B) → (2), (C) → (3), (D) → (1)
E
  1 E
  11
2 11
dy
1 1 1 1 6 (A) = 2 x + b > 0 ∀x ∈[1, 3] ⇒ 2 + b ≥ 0 ⇒ b ∈[−2, ∞)
× . + . = dx
 2 2 2 2  11 (B) India played 4 matches in which maximum point
(C) S2 and S4 should be in different groups for both can be earned is 2. So, maximum point in 4 matches
being winner is 8. Getting atleast 7 points means 7 or 8
10
10  1 1  5 C 5 \ Required prob. = P(7) + P(8);
\ Required probability =  .  = or 12 4 =
11  2 2  22 C6 22 P(7) = 4C1(0.35) × (0.5)3 = 0.175; P(8) = (0.5)4 = 0.0625
39. (A) → (2), (B) → (4), (C) → (1) \ Required prob. = 0.2375
(C) S1 = x2 + y2 – a2 = 0,
A(h, k)
(A) xy = c = − ve ( c is purely imaginary )
2
S2 = (x – h)(x – a) + (y – k)y = 0
1− 3
⇒ If x is positive y must be negative real ⇒ or x2 + y2 – x(h + a)
2
(B) 2b = a + c, – ky + ah = 0 D
b (a + b + c ) Replace a by – a, we get
Now, (a + c − b) s = kb2 ⇒ = kb2 S3 = x2 + y2 – x(h – a) – ky – ah = 0
2
3b2 3 Radical axis of S1 and S2 is S1 – S2 = 0
⇒ = kb2 ⇒ k =
2 2 x(h + a) + ky – ah – a2 = 0
Replacing a by –a, radical axis of S1 and S3 is
(C) r = (3 − 0)2 + (4 − 0)2 − 2 = 3 . x(h – a) + ky + ah – a2 = 0
Contd. on Page no. 83
18 MATHEMATICS TODAY | MAY‘18
Exam held
on
8th April

*ALOK KUMAR, B.Tech, IIT Kanpur


1. y2
If the curves = 6x, 9x2
+ by2
= 16 intersect each 1 1
other at right angles, then the value of b is (a) ( 2 − 1) (b) ( 3 − 1)
2 2
9 7 1 1
(a) (b) 6 (c) (d) 4 (c) ( 3 + 1) (d) ( 3 − 2 )
2 2 2 2
 7. If sum of all the solutions of the equation
2. Let u be a vector coplanar with the vectors
 ^ ^ ^  ^ ^   π  π  1
a = 2 i + 3 j − k and b = j + k . If u is perpendicular 8 cos x ⋅  cos  + x  ⋅ cos  − x  −  = 1
      6   6  2
to a and u ⋅ b = 24, then | u |2 is equal to in [0, p] is kp, then k is equal to
(a) 336 (b) 315 (c) 256 (d) 84 20 2 13 8
(a) (b) (c) (d)
3. For each t ∈ R, let [t] be the greatest integer less 9 3 9 9
than or equal to t. Then 2 1 1
8. Let f (x ) = x + 2 and g (x ) = x − ,
1  2  15   x x
lim x    +   + .... +   
x →0+   x   x   x  x ∈ R – {–1, 0, 1}. If h(x ) =
f (x )
, then the local
(a) does not exist in R (b) is equal to 0 g (x )
(c) is equal to 15 (d) is equal to 120 minimum value of h(x) is
4. If L1 is the line of intersection of the planes (a) 2 2 (b) 3 (c) –3 (d) −2 2
2x – 2y + 3z – 2 = 0, x – y + z + 1 = 0 and L2 is the 9. The integral
line of intersection of the planes x + 2y – z – 3 = 0, sin2 x cos2 x
3x – y + 2z – 1 = 0, then the distance of the origin ∫ (sin5 x + cos3 x sin2 x + sin3 x cos2 x + cos5 x)2 dx
from the plane containing the lines L1 and L2 is
1 1 1 1 is equal to
(a) (b) (c) (d) −1 1
2 4 2 3 2 2 2 (a) +C (b) +C
1 + cot 3 x 3(1 + tan3 x )
π /2
sin2 x
5. The value of ∫ x
dx is
(c)
−1
+ C (d) +C
1
− π /2 1 + 2 3(1 + tan x ) 3
1 + cot 3 x
π π π
(a) (b) (c) (d) 4p 10. A bag contains 4 red and 6 black balls. A ball is
4 8 2 drawn at random from the bag, its colour is
6. Let g(x) = cosx2, f (x ) = x and a, b (a < b) be observed and this ball along with two additional
the roots of the quadratic equation 18x2 – 9px + p2 = 0. balls of the same colour are returned to the bag. If
Then the area (in sq. units) bounded by the curve now a ball is drawn at random from the bag, then
y = (gof)(x) and the lines x = a, x = b and y = 0 is the probability that this drawn ball is red, is

* Alok Kumar is a winner of INDIAN NATIONAL MATHEMATICS OLYMPIAD (INMO-91).


He trains IIT and Olympiad aspirants.

MATHEMATICS TODAY | MAY‘18 19


1 19. If the system of linear equations
3 3 2
(a) (b) (c) (d) x + ky + 3z = 0
4 10 5 5
3x + ky – 2z = 0
11. Let the orthocentre and centroid of a triangle 2x + 4y – 3z = 0
be A(–3, 5) and B(3, 3) respectively. If C is the xz
has a non-zero solutions (x, y, z), then is equal to
circumcentre of this triangle, then the radius of the y2
circle having line segment AC as diameter, is (a) 30 (b) –10 (c) 10 (d) –30
3 5 5 x − 4 2x 2x
(a) (b) 10 (c) 2 10 (d) 3
2 2
20. If 2 x x − 4 2 x = ( A + Bx )(x − A)2 , then
12. If the tangent at (1, 7) to the curve x2 = y – 6 touches
2x 2x x−4
the circle x2 + y2 + 16x + 12y + c = 0, then the value
of c is the ordered pair (A, B) is equal to
(a) 95 (b) 195 (c) 185 (d) 85 (a) (4, 5) (b) (–4, –5)
(c) (–4, 3) (d) (–4, 5)
13. If a, b ∈ C are the distinct roots of the equation
x2 – x + 1 = 0, then a101 + b107 is equal to 21. Two sets A and B as under :
(a) 2 (b) –1 (c) 0 (d) 1 A = {(a, b)} ∈ R × R : |a – 5| < 1 and |b – 5| < 1};
B = {(a, b)} ∈ R × R : 4(a – 6)2 + 9(b – 5)2 ≤ 36}.
14. PQR is a triangular park with PQ = PR = 200 m. Then
A T.V. tower stands at the mid-point of QR. If the (a) neither A ⊂ B nor B ⊂ A
angles of elevation of the top of the tower at P, Q (b) B ⊂ A (c) A ⊂ B
and R are respectively 45°, 30° and 30°, then the (d) A ∩ B = f(an empty set)
height of the tower (in m) is 22. Tangent and normal are drawn at P(16, 16) on the
(a) 50 2 (b) 100 parabola y2 = 16x, which intersect the axis of the
parabola at A and B, respectively. If C is the centre
(c) 50 (d) 100 3
of the circle through the points P, A and B and
9 9
∠CPB = q, then a value of tanq is
15. If ∑ (xi − 5) = 9 and ∑ (xi − 5)2 = 45, then the 4 1
i =1 i =1 (a) (b) (c) 2 (d) 3
standard deviation of the 9 items x1, x2, ....., x9 is 3 2
(a) 3 (b) 9 (c) 4 (d) 2 23. Let S = {t ∈R : f(x) = |x – p|·(e|x| – 1) sin|x| is not
differentiable at t}, then the set S is equal to
16. The sum of the co-efficients of all odd degree terms
(a) {0, p} (b) f (an empty set)
in the expansion of (x + x 3 − 1)5 + (x − x 3 − 1)5 , (c) {0} (d) {p}
(x > 1) is 24. The Boolean expression ~(p ∨ q) ∨ (~ p ∧ q) is
(a) 2 (b) –1 (c) 0 (d) 1 equivalent to
17. Tangents are drawn to the hyperbola 4x2 – y2 = 36 (a) ~q (b) ~ p
at the points P and Q. If these tangents intersect (c) p (d) q
at the point T(0, 3), then the area (in sq. units) of 25. A straight line through a fixed point (2, 3) intersects
DPTQ is the coordinate axes at distinct points P and Q. If O
(a) 36 5 (b) 45 5 is the origin and the rectangle OPRQ is completed,
(c) 54 3 (d) 60 3 then the locus of R is
(a) 3x + 2y = 6xy (b) 3x + 2y = 6
18. From 6 different novels and 3 different dictionaries,
(c) 2x + 3y = xy (d) 3x + 2y = xy
4 novels and 1 dictionary are to be selected and
arranged in a row on a shelf so that the dictionary 26. Let A be the sum of the first 20 terms and B be the
is always in the middle. The number of such sum of the first 40 terms of the series
arrangements is 12 + 2·22 + 32 + 2·42 + 52 + 2·62 + .....
(a) at least 750 but less than 1000 If B – 2A = 100l, then l is equal to
(b) at least 1000 (c) less than 500 (a) 496 (b) 232
(d) at least 500 but less than 750 (c) 248 (d) 464

20 MATHEMATICS TODAY | MAY‘18


27. Let y = y(x) be the solution of the differential For b = 0 the intersection is non orthogonal. So we
dy can rule out b = 0 in the beginning only to conclude
equation sin x + y cos x = 4 x , x ∈(0, π). If
dx a ≠ 0 in the end.
π π [Rating : Medium]
y   = 0, then y   is equal to
2 6  ^ ^ ^
2. (a) : Let u = x i + y j + z k
4 4 2  
(a) − π2 (b) π u ⋅ a = 0 gives 2x + 3y – z = 0 ...(i)
9 9 3  
u ⋅ b = 24 gives y + z = 24 ...(ii)
−8 2 8 2    
(c) π (d) − π Also u is coplanar with a and b , so u a b  = 0
9 3 9 which yields
28. The length of the projection of the line segment
x y z
joining the points (5, –1, 4) and (4, –1, 3) on the
plane x + y + z = 7 is 2 3 −1 = 0
2 2 0 1 1
(a) (b) i.e. 4x – 2y + 2z = 0 ...(iii)
3 3
(ii) and (iii) gives 2x + 2z = 24 i.e. x + z = 12
2 1
(c) (d) From (i), we get z = 16 and thus x = – 4
3 3 and y = 8.
29. Let S = {x ∈ R : x ≥ 0 and  ^ ^ ^
Hence, u = − 4 i + 8 j + 16 k
2| x − 3 | + x ( x − 6) + 6 = 0}. Then S 
(a) contains exactly four elements | u | = 4 12 + 22 + 42 = 4 21
(b) is an empty set 
∴ | u |2 = 336
(c) contains exactly one element
Alternative solution-1:
(d) contains exactly two elements   
As u is coplanar with a and b , we have
30. Let a1, a2, a3, .................. a49 be in A.P. such that   
12 u = λa + µb for scalars l and m.
∑ a4k + 1 = 416 and a9 + a43 = 66.  ^ ^ ^  ^ ^
a = 2 i + 3 j − k and b = j + k gives
k=0  ^ ^ ^
u = 2 λ i + (3 λ + µ) j + (µ − λ) k
a12 + a22 + ............ + a17
2
= 140m, then m is equal to  
(a) 33 (b) 66 u ⋅ b = 24 gives l + m = 12
 
(c) 68 (d) 34 u ⋅ a = 0 gives 7l + m = 0
Solving the above, we get l = –2, m = 14
SOLUTIONS  ^ ^ ^
Hence, u = − 4 i + 8 j + 16 k
1. (a) : Let the curves y2 = 6x and 9x2 + by2 = 16 
\ | u|2 = 336
intersect at (a, b).
Alternative solution-2:
b2 = 6a and 9a2 + bb2 = 16
This is the most efficient solution, involving the idea
dy dy 3 3
for curve y2 = 6x is = = of triple product.
dx (α, β) dx y β  
A vector that is coplanar with a and b and perpendicular
dy 
dy 9x 9α to a can be taken as
for curve 9x2 + by2 = 16 is =− =−    
dx (α, β) dx by bβ u = ma × (a × b )
    
As the curves are orthogonal, we have = m{| a |2 b − (a ⋅ b )a}
 3   9α  ^ ^
= m{−4 i + 8 j + 16 k}
^
 β   − bβ  = − 1
^ ^ ^
= − 4m{i − 2 j − 4 k}
As b2 = 6a, we get 27a = b(a.6)
^ ^ ^
27 9 = m′{i − 2 j − 4 k}
⇒ b= = (as α ≠ 0)  
6 2 As u ⋅ b = 24 we have m′(–2 – 4) = 24 ⇒ m′ = – 4

MATHEMATICS TODAY | MAY‘18 21


 ^ ^ ^ π /2
Thus u = − 4( i − 2 j − 4 k )  1 1 
2I = ∫ sin2 x  +  dx
 x
1 + 2− x 
As before | u |2 = 42 (12 + 22 + 42 ) = 16 ⋅ 21 = 336 − π /2 1 + 2
[Rating : Difficult] π /2
3. (d) : Observe that t – 1 < [t] ≤ t = ∫ sin2 x dx
1 2 15 − π /2
Applying this to numbers , , ......., and summing
x x x π /2
them, we have =2 ∫ sin2 x dx (As the integral function is even)
15 15 15 0
k k k
∑x − 15 < ∑  x  ∑ x

π
k =1 k =1 k =1 ∴ I=
Multiplying throughout by x, we have 4
[Rating : Easy]
15 15 15
k
∑ k − 15x < x ∑  x  ≤ ∑ k 6. (b) : y = (gof)(x) = cos x 2 = cos|x| = cos x
k =1 k =1 k =1 [Q cos (–x) = cos x]
Putting the limit x → 0+, we have Consider 18x2 – 9px + p2 = 0
120 < L ≤ 120 ⇒ (3x – p)(6x – p) = 0
As the limit from both sides approaches to 120, we π π
have by sandwich principle, the required limit = 120. ⇒ x= ,
3 6
[Rating : Difficult] \ Required area
4. (c) : A plane passing through the intersection of π /3
π /3 3 1 1
the given planes is = ∫ cos x dx = [sin x ]π/6 = − = ( 3 − 1)
(2x – 2y + 3z – 2) + l (x – y + z + 1) = 0 2 2 2
π /6
i.e. (l + 2)x – (2 + l)y + (l + 3)z + (l – 2) = 0 [Rating : Easy]
The plane is having infinite number of solutions with 7. (c) : Note that cos(A + B) cos(A–B) = cos2A – sin2B
x + 2y – z – 3 = 0 and 3x – y + 2z – 1 = 0. We have,
(λ + 2) −(λ + 2) (λ + 3) 8cosx {cos (p/6 + x) cos(p/6 – x) – 1/2} = 1
∴ 1
3
2
−1
−1 = 0
2
⇒ 8 cos x cos2 { π
6 }
− sin2 x − 1 / 2 = 1

⇒ (l + 2)(4 – 1) + (l + 2)(2 + 3) + (l + 3)(–1 – 6) = 0 3 1


⇒ 8 cos x  − 1 + cos2 x −  = 1
⇒ l=5 4 2
\ The equation of the plane becomes ⇒ 8 cos x(cos2 x − 3 / 4) = 1
7x – 7y + 8z + 3 = 0
The perpendicular distance from origin is (4 cos2 x − 3)
⇒ 8 cos x =1
3 3 3 1 4
= = = 1
162 9 2 3 2 ⇒ (4cos3x – 3cosx) =
72 + 72 + 82 2
[Rating : Medium] 1
⇒ cos3x =
π /2 2
sin2 x As x ∈ [0, p] ⇒ 3x ∈ [0, 3p] which gives
5. (a) : Let I = ∫ x
dx ...(i)
− π /2 1 + 2 3x = p/3, 5p/3, 7p/3
Changing x to –p/2 + p/2 – x = – x, we have \ x = p/9, 5p/9, 7p/9
13π
π /2
sin2 x which gives sum of all values of x =
9
I= ∫ −x
dx ...(ii)
− π /2 1 + 2 13
∴ k=
Adding (i) & (ii), we get 9 [Rating : Medium]

22 MATHEMATICS TODAY | MAY‘18


2 The requested probability is the sum of the product of
2 1  1
8. (a) : We have, f (x) = x + = x −  +2
2 
the probabilities along the two possible paths and is
x  x
2 1 3 1 1 1 2
1 equal to ⋅ + ⋅ = + =
And g (x ) = x − for x ∈ R – {– 1, 0, 1} 5 2 5 3 5 5 5
x [Rating : Medium]
2
 1 11. (d) : We know that the centroid divides orthocentre
 x −  +2
\ h (x ) = x 
= x −
1 2 and circumcentre in the ratio 2 : 1.
  +
1 x  1
x−  x − 
x x
1
Put x − =t
x 3 3 3
2 AC = AB = ⋅ 62 + 22 = ⋅ 2 10 = 3 10
∴ H (t ) = t + for t ∈(−∞, ∞) − {0} 2 2 2
t 3 5
Radius of the circle with AC as diameter = 10 = 3
Consider H(t) on the interval (0, ∞) 2 2
2 [Rating : Difficult]
H (t ) = t +
t 12. (a) : The equation of tangent at (1, 7) to x2 = y – 6
2 is 2x – y + 5 = 0
H ′(t ) = 1 − 2
t The perpendicular distance of centre (–8, –6) to the
line 2x – y + 5 = 0 should be equal to the radius of
So, H(t) is decreasing on (0, 2 ) and increasing on
the circle.
( 2 , ∞). Thus H(t) has a local minimum at t = 2 . | −16 + 6 + 5 |
\ H( 2 ) = 2 2 is the local minimum value of the \ 64 + 36 − c =
5
function at 2. ⇒ 5 = 100 − c ⇒ c = 95
Remark : Observe that [Rating : Easy]
2
t + ≥2 2 13. (d) : x2 – x + 1 = 0 has its roots –w, –w2.
t
thereby again contribute that 2 2 is a local minimum. Now (–w)101 + (–w2)107 = – {w2 + w4} = – (w2 + w) = 1
[Rating : Difficult] [Rating : Easy]
14. (b) :
sin2 x cos2 x
9. (c) : Let I = ∫ dx
(sin2 x + cos2 x )2(sin3 x + cos3 x )2
sin2 x cos2 x tan2 x sec2 x
=∫
(sin3 x + cos3 x )2 ∫ (1 + tan3 x )2
= dx

Put 1 + tan3x = t so that 3tan2x sec2x dx = dt


1 dt 1 1 1 1
∴ I = ∫ 2 =− ⋅ +C =− ⋅ +C
3 t 3 t 3 (1 + tan3 x )
[Rating : Difficult] Let the height of tower MN be h. N
10. (c) : Let's draw a state diagram to understand whole The triangle NMQ gives
h
condition 1
QM = h 3 , as tan 30° = 30° M
3 Q
N
The triangle NMP gives
PM = h h
45°
M P

MATHEMATICS TODAY | MAY‘18 23


As DPQR is isosceles, PM is also an altitude. 1
\ PM2 + QM2 = PQ2 gives 4h2 = (200)2 ⇒ h = 100 = .6 5 .15 = 45 5
2 [Rating : Difficult]
[Rating : Medium]
15. (d) : The standard deviation is independent of 18. (b) : The number of ways to choose 4 novels out
change of origin. So, put xi – 5 = yi of 6 is 6C4.
\ Given equations become The number of ways to choose 1 dictionary out of 3
9 9 is 3C1.
∑ yi = 9 and ∑ yi2 = 45 As the place of dictionary is fixed, so total number
i =1 i =1 of ways
= 6C4 3C1. 4! = 15.3.24 = 1080 [Rating : Easy]
2
1  Σy  19. (c) : For non-zero solutions, we have
SD =
n
∑ yi2 −  i 
 n  1 k 3
2 3 k −2 = 0
45  9 
= −   = 5 −1 = 4 = 2 2 4 −3
9 9
⇒ 1(–3k + 8) – k(–9 + 4) + 3 (12 – 2k) = 0
[Rating : Medium]
which gives k = 11
16. (a) : We have, Now, the system of equations become
(a + b)5 + (a – b)5 = 2{a5 + 5C2.a3b2 + 5C4 ab4} x + 11y + 3z = 0 ...(i)
with a = x, b = x 3 − 1 3x + 11y – 2z = 0 ...(ii)
5 5
2x + 4y – 3z = 0 ...(iii)
∴  x + x3 − 1  +  x − x3 − 1  The equation (i) and (iii) gives
    3x + 15y = 0 i.e. x = – 5y
= 2{x5 + 10x3 (x3 – 1) + 5x(x3 – 1)2} Putting x = – 5y in (i), we have
Sum of the coeff. of odd degree terms is –5y + 11y + 3z = 0
2{1 – 10 + 5 + 5} = 2 ⇒ z = – 2y
[Rating : Easy] xz (−5 y )(−2 y )
Now 2 = = 10
17. (b) : Let the tangent at (a, b) be y y2
4xa – yb = 36
[Rating : Difficult]
As (0, 3) lies on the tangent, so we have
–3b = 36 ⇒ b = – 12 20. (d) : As both sides are polynomial in x, let's set
Now 4a2 – b2 = 36 gives 4a2 – 122 = 36 x = 0 to obtain
−4 0 0
⇒ 4α 2 = 180 ⇒ α 2 = 45 ⇒ α = ±3 5
0 −4 0 = A3
Thus the points P and Q are P (3 5 , − 12), Q(−3 5 , − 12)
The area of the DTQP is given by 0 0 −4
0 3 1 which gives A3 = – 64 \ A = – 4
1 Taking x common from all rows of given determinant,
∆ = 3 5 −12 1 we get
2
−3 5 −12 1 4
1− 2 2
1
2
( ) 1
= [−3 6 5 − 36 5 − 3 6 5 ] = 90 5 = 45 5
2
x
4  4 4
2
2 1− 2 =  B −  1 + 
x  x  x 
Alternative solution:
The equation of chord of contact from (0, 3) is 4
2 2 1−
4.x.0 – y.3 = 36 ⇒ y = – 12 x
\ The point P is (3 5 , − 12) Take the limit as x → ∞ to obtain
1 1 2 2
Area of triangle PQT = .PQ.TR
2 2 1 2 =B⇒B=5
[Where R is the mid-point of PQ] 2 2 1

24 MATHEMATICS TODAY | MAY‘18


Alternative solution: 16 4
The slope of PC : m1 = =
The most efficient way to obtain the result is to use 12 3
this result −16
The slope of PB : m2 = =−2
a b b 8
4 10
b a b = (a + 2b)(a − b)2 m1 − m2 +2
tan θ = = 3 = 3 =2
b b a 1 + m1m2 4 5
1 − (2) −
This result gives A = – 4, B = 5 3 3
Note that the determinant, when a = b, vanish and all Alternative solution:
the three rows become identical hence (a – b)2 is a factor. Note that for a general point (at2 , 2at) the diagram
[Rating : Medium] is as under
21. (c) : Let's effect a change of origin
a – 6 = u and b – 5 = v

1 1 1
As tan(90° − θ) = = ⇒ cot θ =
t 2 2

u2 v 2 So, tanq = 2 [Rating : Difficult]


The set B becomes + ≤1 23. (b) : At x = 0, we have
9 4
which is the interior of the ellipse. f (0 − h) − f (0)
L.H.D. = lim
Now the set A becomes h→0 −h
|u + 1| < 1 i.e. –2 < u < 0
|v| < 1 i.e. –1 < v < 1 (π + h)(e h − 1)sin h
= lim = π(0)(−1) = 0
which is a square. h→0 ( − h)
Drawing the two diagram together, we have
f (0 + h) − f (0) (π − h)(e h − 1)sin h
R.H.D. = lim = lim
h→0 h h→0 h
= (p)(0)(1) = 0
Let's check at x = p
f (π − h) − f (π)
L.H.D. = lim
h→0 −h
π−h
h(e− 1)sin h
So A ⊂ B as all the four points (0, 1), (0, –1), (–2, –1) = lim = (0) (e π − 1)(−1) = 0
h→0 + − h
and (–2, 1) lie inside the ellipse.
[Rating : Difficult] f (π + h) − f (π)
R.H.D. = lim
22. (c) : The equation of tangent at P(16, 16) is h→0 h
π+h
x – 2y + 16 = 0 h(e − 1)(− sin h)
The equation of normal at P(16, 16) is = lim = (0) (e π − 1)(−1) = 0
h→0 h
2x + y – 48 = 0
Thus f is differentiable at both x = 0 and x = p.
Remark : This happens as x = 0 and x = p both are
repeated roots of the given function.
[Rating : Difficult]
24. (b) : ~ (p ∨ q) ∨ (~ p ∧ q)
= (~ p ∧ ~ q) ∨ (~ p ∧ q)
= ~ p ∧ (~ q ∨ q) = ~ p [Rating : Medium]

MATHEMATICS TODAY | MAY‘18 25


25. (d) : 29. (d) : Given 2 | x − 3 | + x ( x − 6) + 6 = 0
The equation of the given line is
x y ⇒ 2 | x − 3 | + ( x − 3)2 − 3 = 0
+ = 1 ...(i)
α β set | x − 3 | = t , which gives
2 3
As (2, 3) lies on (i), + = 1 t2 + 2t – 3 = 0
α β ⇒ (t + 3)(t – 1) = 0
⇒ 2b + 3a – ab = 0 ⇒ t = –3, 1
changing (a, b) to (x, y) we have the locus of R as
As t ≥ 0 we have t = 1
3x + 2y – xy = 0
[Rating : Medium] Now x − 3 =1
26. (c) : Let S = 1 + 2 2 + 3 + 2.42 + 52 + 2.62 + ....
2 . 2 2
⇒ x − 3 = 1 or − 1 ⇒ x = 4, 2
The sum of first 20 terms is
A =(12 + 22 + .... + 202) + (22 + 42 + .... + 202) So, x = 16, 4
20 ⋅ 21 ⋅ 41 10 ⋅11 ⋅ 21 20 ⋅ 21 Thus, there are two solutions.
= + 4⋅ = (41 + 22) = 4410 [Rating : Medium]
6 6 6
12
B = 12 + 2.22 + .... + 2.402
= (12 + 22 + .... + 402) + (22 + 42 + .... + 402)
30. (d) : ∑ a4k + 1 = 416
k=0
40 ⋅ 41 ⋅ 81 4 ⋅ 20 ⋅ 21 ⋅ 41 40 × 41 ⇒ a1 + a5 + .... + a49 = 416
= + = (81 + 42)
6 6 6 13
⇒ (a + a ) = 416
40 ⋅ 41 2 1 49
= × 123 = 33620 As a49 = a1 + 48d, so we have
6
B – 2A = 33620 – 8820 = 24800 13
(2a1 + 48d ) = 416,
\ 100l = 24800 ⇒ l = 248 2
[Rating : Medium] ⇒ a1 + 24d = 32 ...(i)
Given, a9 + a43 = 66
dy
27. (d) : + (cot x ) y = 4 xcosecx ⇒ a1 + 8d + a1 + 42d = 66
dx
⇒ a1 + 25d = 33 ...(ii)
I.F. = e ∫
cot x dx
= e log(sin x ) = sin x The equation (i) and (ii) gives
Then the solution is given by a1 = 8, d = 1
2
y ⋅ sin x = ∫ 4 x cosec(x ) sin x dx + C Now a12 + a22 + ..... + a17 = 82 + 92 + .... + 242
i.e. ysinx = 2x2 + C = (12 + 22 + ..... + 242) – (12 + 22 + .... + 72)
As y(p/2) = 0, we have C = –p2/2 24 ⋅ 25 ⋅ 49 7 ⋅ 8 ⋅15
= −
So, ysinx = 2x2 – p2/2 6 6

∴ y (π / 6) = 2 
 2 π2 π2 
 36
−  = 2 π2
2
1 1
− = − π2
18 2
8
9 { } = 4.25.49 – 7.20 = 4900 – 140 = 4760 = 34(140)
\ m = 34
[Rating : Medium]
[Rating : Medium]
28. (a) : The direction ratios of AB, where A(5, –1, 4) 
and B(4, –1, 3) are (1, 0, 1)
Solution Sender of Maths Musing
Let the angle between AB and plane is q, which gives
2 1 SET-183
sin θ = i.e. cos θ = • Gouri Sankar Adhikary (West Bengal)
6 3
The projection of AB on the plane = ABcosq SET-184
1 2 • N. Jayanthi (Hyderabad)
= 2⋅ = • Gajula Ravinder (Karimnagar)
3 3
[Rating : Medium] • Devjit Acharjee (West Bengal)

26 MATHEMATICS TODAY | MAY‘18


1. Let x be a real number with 0 < x < p. Prove that, for SOLUTIONS
all natural numbers n, the sum 1. We use mathematical induction.
sin 3x sin 5x sin(2n − 1)x n
sin(2k − 1)x
sin x + + + ... + is positive. Let Sn(x) = ∑ .
3 5 2n − 1
k =1 (2k − 1)
2. The incircle of ABC touches BC, CA and AB at D, S1(x) = sin x > 0 for x ∈ (0, p). Thus the proposed
E and F respectively. X is a point inside ABC such inequality is true for n = 1. Let S r (x) > 0 for
that the incircle so XBC touches BC at D also, and r = 1, 2, ..., n – 1. We will deduce that Sn(x) > 0 for
touches CX and XB at Y and Z, respectively. Prove x ∈ (0, p). Suppose that S n (x 0 ) ≤ 0 for some
that EFZY is a cyclic quadrilateral.
x0 ∈ (0, p), and that Sn(x) attains its minimum at
3. An acute triangle ABC is given.Points A1 and A2 are d
taken on the side BC (with A2 between A1 and C), x = x0. Hence [Sn(x)]x = x0 = 0.
dx
B1 and B2 on the side AC (with B2 between B1 and That is
A) and C1 and C2 on the side AB (with C2 between n
C1 and B) so that Sn′(x0) = ∑ cos((2k − 1)x0) = 0,
∠AA1A2 = ∠AA2A1 = ∠BB1B2 = ∠BB2B1 = ∠CC1C2 k =1
= ∠CC2C1. so that n
The lines AA1, BB1 and CC1 bound a triangle, and 2 sin x0Sn′(x0) = ∑ 2 cos((2k − 1)x0)sin x0
k =1
the lines AA2, BB2 and CC2 bound a second triangle. n
Prove that all six vertices of these two triangles lie = ∑ [sin(2kx0 ) − sin((2k − 2)x0 )]
on a single circle. k =1

4. Prove that the average of the numbers n sin n°, = sin 2nx0.
n = 2, 4, 6, ..., 180 is cot 1°. sin 2nx0
Thus Sn′(x0) = = 0 implying sin 2nx0 = 0.
5. Let p be a prime. Find all solutions in positive 2 sin x0
integers of the equation : Hence
2 3 5  π 2π 3π (2n − 1) 
+ = . x0 ∈ , , ,..., .
a b p  2n 2n 2n 2n 
It is easily verified that at each of these values
6. Find the value of the continued root : Sn(x0) > 0, a contradiction. Hence Sn(x) > 0 for
x ∈ (0, p).
4 + 27 4 + 29 4 + 31 4 + 33 .... .
2. A
7. a1, ..., ak, ak + 1, ..., an are positive numbers (k < n).
suppose that the values of ak +1, ..., an are fixed. How F
should one choose the values of a1, ..., an in order to Z XY E
a
minimise ∑ i ? B P
i, j, i≠ j a j D C

MATHEMATICS TODAY | MAY‘18 27


Let P be the intersection of EF with BC. Then by Therefore HE = HD. Similarly, we prove that
Menelaus' Theorem, we have HD = HI, and so on for all six vertices of these
BP CE AF two triangles of the problem.
⋅ ⋅ =1 ...(1) Thus, all six vertices lie at the same distance from
PC EA FB
the point H, and the points are concyclic.
Since CE = CD, EA = AF, and FB = BD, we get
BP CD 4. Let x = 2 sin 2° + 4 sin 4° + ... + 90 sin 90° + ...
⋅ =1 + 178 sin 178°
PC BD
BP BD = (2 + 178)sin 2° + (4 + 176)sin 4° + ...
so that = ...(2) = 180(sin 2° + sin 4° + ... + sin 88°) + 90 sin 90°
PC CD
Then
Since XZ = XY, BZ = BD and CY = CD, we have x

x= = 2 sin 2° + 2 sin 4° + ... + 2 sin 88° + 1
from (2)
90
BP CY XZ BD CD XY –
x sin1° = 2 sin 2° sin 1° + 2 sin 4° sin 1° + ...
⋅ ⋅ = ⋅ ⋅ =1
PC YX ZB CD YX BD + 2 sin 88° sin 1° + sin 1°.
Hence by Menelaus' Theorem P, Z and Y are Now, 2 sin 2° sin 1° = cos 1° – cos 3°
collinear. Since PF ⋅ PE = PD2 and PZ . PY = PD2, 2 sin 4° sin 1° = cos 3° – cos 5°
we have PF ⋅ PE = PZ ⋅ PY. ...
Hence EFZY is a cyclic quadrilateral. 2 sin 88° sin 1° = cos 87° – cos 89°
Hence, – x sin 1° = cos 1° – cos 89° + sin 1° = cos 1°.
3. A Thus – x = cot 1°, as required.
5. We have p(3a + 2b) = 5ab; hence there are three
cases to consider.
xx First case : p = 5, Then we get
C1 B1 3a + 2b = ab or (a – 2)(b – 3) = 6 = 1 ⋅ 6 = 2 ⋅ 3.
I
C2 H Hence all pairs of positive integers (a, b) are
(3, 9), (4, 6), (5, 5) and (8, 4).
x x
E Second case : p divides a. Let a = a1p. Hence
B
A1 A2
C 3a1p = b(5a1 – 2).
Then p divides either b or 5a1 – 2. If b = b1p, then
Let AA1, BB1 meet at the point E; AA1, CC2 meet at 3a1 = b1(5a1 – 2) or (5a1 – 2)(5b1 – 3) = 6
the point F; and BB1, CC1 meet at the point I. Also which has only one solution : (a1, b1) = (1, 1),
∠A1AA2 = ∠B1BB2 = ∠C1CC2 = 2x. (1) hence (a, b) = (p, p). Otherwise, 5a1 – 2 = a2 p.
The bisectors of the angles at A1, B1 and C1 in a p+2
Therefore, 3 2 = ba2 or 3a2 p + 6 = 5ba2.
triangles DA1AA2, DB1BB2 and DC1CC2 respectively 5
are perpendicular to their respective bases. Hence Hence a2 divides 6.
they are the altitudes of DABC. Let H be the  p( p + 2) 3( p + 2) 
If a2 = 1, then (a, b) =  ,  , but
orthocentre of DABC.  5 5 
Since ∠A1AH = ∠B1BH = x and ∠A1AH = ∠C1CH = x only if p ≡ 3 (mod 5).
each one of the quadrilateral AHEB, AHDC is  2 p( p + 1) 3( p + 1) 
If a2 = 2, then (a, b) =  ,  , but
 5 5 
inscribable in a circle.
only if p ≡ 4 (mod 5).
These two circles have a common chord, the segment
 p(3 p + 2) 3 p + 2 
AH and since ∠ABH = ∠ACH = 90° – ∠BAC, then If a2 = 3, then (a, b) =  ,  , but
 5 5 
the circles have equal radii. only if p ≡ 1 (mod 5).
Thus, since the inscribed angles ∠EAH, ∠DAH  2 p(3 p + 1) 3 p + 1 
If a2 = 6, then (a, b) =  ,  , but
are equal, the corresponding chords HE and HD  5 5 
are equal. only if p ≡ 3 (mod 5).

28 MATHEMATICS TODAY | MAY‘18


Third case : p divides b. Let b = b1p. Hence n(n + 2) n(n + 2)
2b1p = a(5b1 – 3). g (n, m) < g (m, m) < .
m(m + 2) m
Then p divides either a or 5b1 – 3. If p divides a, Therefore g(n, m) → 0 as m → ∞
then we have the same case (p divides a and b) as II. Let
already considered above. Again (p, p) is the (only)
solution. Otherwise, 5b1 – 3 = b2p. Therefore, Sn = 4 + (2n − 1) 4 + (2n + 1) 4 + (2n + 3) …
b p+3 Sn satisfies the recurrence relation
2 2 = ab2 or 2b2p + 6 = 5ab2.
5 Sn = 4 + (2n − 1)Sn+1
Hence b2 divides 6.
if and only if
 2( p + 3) p( p + 3)  (Sn – 2) (Sn + 2) = (2n – 1)Sn + 1
If b2 = 1, then (a, b) =  ,  , but
 5 5  By inspection, this admits Sn = 2n + 1 as a solution.
only if p ≡ 2, (mod 5).
We only have to prove that S1 = 3 to make this
 2 p + 3 p(2 p + 3)  induction complete. Let
If b2 = 2, then (a, b) =  ,  , but
 5 5
only if p ≡ 1, (mod 5).
Tn = 4 + 4 + 3 …(2n − 3) 4 + (2n − 1) (2n + 3)
 2( p + 1) 3 p( p + 1) 
If b2 = 3, then (a, b) =  ,  , but
 5 5 and
only if p ≡ 4, (mod 5).
 2 p + 1 3 p(2 p + 1)  U n = 4 + 4 + 3 …(2n − 3) 4 + (2n − 1)(2n + 3) = 3.
If b2 = 6, then (a, b) =  ,  , but
5 5 Clearly Tn ≤ Un and the latter is identically equal to 3.
only if p ≡ 2 (mod 5).
Therefore, using the fact that B ≥ A > 0 implies
[Note that if p = 2 or 3, this generates only two
that (4 + A) / (4 + B) ≥ A / B
distinct solutions : (2, 2), (1, 6) for p = 2 and (3, 3),
(12, 2) for p = 3. If p > 5, then the three solutions
Tn Tn 4 + … + (2n − 1) (2n + 3)
are all distinct.] 1≥ = =
3 Un 4 + … + (2n − 1)(2n + 3)
6. I. More generally, for any positive integer n, we
claim that
… + (2n − 1) (2n + 3) n +1 1
≥ ≥…≥ 2
4 + n 4 + (n + 2) 4 + (n + 4) ... = n + 2, … + (2n − 1)(2n + 3) 2n + 3

where the left side is defined as the limit of


1
= n +1
→1
1
F (n, m) = 4 + n 4 + (n + 2) 4 + (n + 4) ... 4 + m 4  
2
(2n + 3)
as m → ∞ (where m is an integer and (m – n) is even)
as n → ∞ [for example, by rewriting as
If g(n, m) = F(n, m) – (n + 2), we have
exp{–ln(2n + 3)/2n + 1} and using L’Hospital rule].
F(n, m)2 – (n + 2)2 = (4 + nF(n + 2, m))
This proves that S1 = lim Tn = 3. The required
– (4 + n(n + 4)) n→∞
= n(F(n + 2, m) – (n + 4)), expression is precisely S14 and hence its value is 29.
n
so, g (n, m) = g (n + 2, m).
F (n, m) + n + 2 MPP-1 CLASS XII ANSWER KEY
Clearly F(n, m) > 2, so 1. (d) 2. (d) 3. (b) 4. (b) 5. (d)
n 6. (d) 7. (a,b,c) 8. (c) 9. (b,d) 10. (a,b)
g (n, m) < g (n + 2, m) .
n+4 11. (a,b,c) 12. (a,b,c,d) 13. (a,c) 14. (d) 15. (a)
By iterating this, we obtain 16. (c) 17. (0) 18. (0) 19. (3) 20. (3)

MATHEMATICS TODAY | MAY‘18 29


7. To minimise the given rational function, choose m mA 
1/2 = ∑  xi +
 a +… + an
ai =  k +1

= (A · H)1/2 , i = 1, 2, ... , k i H xi 
1 1 
 +… +  where A is the arithmetic mean and H is the
 ak +1 an 
harmonic mean of the br.
where A is the arithmetic and H the harmonic β
Now we recall that the simple function ax +
mean of ak + 1, ..., an. x
(with a, b, x all positive) assumes its minimum
To prove this, we will be forgiven if we change
β β
notation : let xi = ai, i = 1, 2, ..., k and br = ak + r, when ax = ; that is x = . Thus each of the
r = 1, ..., m with k + m = n, and denote the given x α
terms in Y (and so Y itself) assumes its minimum
rational function F(x 1, ... , x k). Then we have
F(x1, ..., xk) = X + Y + B, where when we choose, for i = 1, 2, ..., k,
mA
 x xj  x b  xi = = AH ,
X= ∑  xi + x  , Y = ∑ ∑  b i + xr  , (m / H)
1≤i < j ≤ k  j i  1≤i ≤ k 1≤ r ≤ m r i
as asserted.
 br bs  But there is more. It is also known that each term
B= ∑  +b  . in X, (and so X itself ) assumes its minimum
1≤r < s ≤m  bs r
Note that B is fixed and Y can be improved to when xi = xj, with 1 ≤ i < j ≤ k. Thus choosing all
 1  1 xi = AH minimises both X and Y and, since B
Y = ∑  ∑  xi +  ∑ bi  
is fixed, minimises F(x1, … , xk) as claimed.
1≤i ≤ k   1≤ r ≤ m br  1≤r ≤m  xi 


ATTENTION
COACHING CLASSROOM STUDY MATERIAL
INSTITUTES :
a great offer from
MTG
MTG “Classroom Study Material” for JEE (Main & Advanced),
NEET and FOUNDATION MATERIAL for Class 6, 7, 8, 9, 10, 11 & 12 with
YOUR BRAND NAME & COVER DESIGN.
This study material will save you lots of money spent on teachers,
typing, proof-reading and printing. Also, you will save enormous
time. Normally, a good study material takes 2 years to develop. But
you can have the material printed with your logo delivered at your
doorstep.

educational publishing for JEE (Main & Advanced)/NEET/PMT....


Order sample chapters on Phone/Fax/e-mail.

 EXCELLEN T
Phone : 0124-6601200 | 09312680856 QUALITY
e-mail : sales@mtg.in | www.mtg.in  CONTENT
 PAPER
 PRINTING

30 MATHEMATICS TODAY | MAY‘18


Sets, Relations and Functions
SET disjoint sets. In other words, if their intersection is
A set is a well defined collection of distinct objects non-empty, then they are called joint i.e. A = {a, b, c, 4}
(elements or members). and B = {2, 4, 6} are joint sets and if A = {a, b, c},
Sets are denoted by capital alphabets and their members B = {2, 4, 6}, then A and B are disjoint sets.
or elements/objects by small alphabets like A = {a, b, c}. z Pairwise Disjoint Family of Sets
As a is an element of set A, we write it a ∈ A and read The family of sets A1, A2, A3 is said to be pairwise disjoint
as a belongs to A or a is a member of A. If ‘a’ does not family of sets if no two sets of the family are joint i.e.
belong to A, then we write a ∉ A. A1 ∩ A2 = f, A2 ∩ A3 = f, A3 ∩ A1 = f.
REPRESENTATION OF SETS e.g. A1 = {1, 4, 7, 10}, A2 = {2, 5, 8, 11},
There are two ways of representing sets. A3 = {3, 6, 9, 12} are paiwise disjoint sets.
z Roster method or Listing method or Tabular z Equal and Equivalent Sets
method. (1) Two sets A and B are called equal sets if A and
B having identical elements i.e. A = {a, b, c},
z Rule or property or Set builder method
B = {b, a, c}.
For example, set of even numbers between 7 and 15
(2) The two sets A and B are said to be equivalent if they
can be written as
have equal number of elements i.e. n(A) = n(B).
(a) Roster/Listing/Tabular method :
(3) Equal sets are always equivalent but equivalent sets
A = {8, 10, 12, 14}
may or may not be equal.
(b) Set builder or Rule method :
A = {x : x is an even number, 7 < x < 15} z Singleton Set
A set consisting of single element is called a singleton set.
TYPES OF SETS
z Universal Set
z Empty Set If all the sets under consideration are subsets of a
(1) A set having no element in it, is called empty or null larger set, then this larger set is called universal set
or void set. which we denote by U. The set of prime numbers
(2) Empty set is subset of every set and every set is subset {2, 3, 5, 7, 11, 13, ...}, the set of even numbers
of itself. We denote it by f or { }. {2, 4, 6, 8, 10, ...} and the set of odd numbers
z Finite and Infinite Sets {1, 3, 5, 7, ....} are subsets of N = {1, 2, 3, 4, 5, 6, ....}, which
A set whose elements can be counted. In other words a set is universal set which can be denoted by U instead of N.
which has finite members in it is called finite set and set In the plane geometry, the straight lines in a plane are
whose elements cannot be counted is said to be infinite. subset of all points in the plane, which is universal set in
The set of vowels is finite set, the set of rational numbers respect of all straight lines.
between two rationals is an infinite set. ORDER/CARDINAL NUMBER OF A FINITE SET
z Joint and Disjoint Sets If a set A having m elements then n(A) = m is known as
When two sets A and B having at least one common order of finite set A. The order of the set A = {1, 2, 3, 4,
element between them is said to be joint other wise 5, 6, 9} is 7 as it contains 7 members.
By : R. K. Tyagi, Retd. Principal, HOD Maths, Samarth Shiksha Samiti, New Delhi

MATHEMATICS TODAY | MAY‘18 31


SUBSET two sets A and B is denoted by A ∪ B (read as ‘A union B’). If
For any two sets A and B, the set A is said to be subset A = {a, 2, 3, 4}, B = {2, 3, 7, 9, 11}, then
of B if every element of set A is also the element of set B. A ∪ B = {1, 2, 3, 4, 7, 9, 11}
SUPERSET z Intersection of Sets
For any two sets A and B if all elements of set A are present The intersection of two sets A and B is the set which
in B but some elements of set B are not present in A, then contains all those elements which belongs to both the sets
B is said to be super set of A and A is called subset of B. A and B. It is denoted by A ∩ B (read as ‘A intersection
B’). For the set A and B given above A ∩ B = {2, 3}
COMPLEMENT OF A SET
Let U be the universal set and A is subset of U, then
z Difference of Two Sets
complement of A with respect to U is the set of elements Let A and B are two sets given above, then the difference
which belongs to U, but not belongs to A. The complement from set A to set B denoted by A – B is the set of all those
elements which belongs to A but not belongs to B
of set A is denoted by A′ or A or Ac and defined as
i.e. A – B = {x : x ∈ A but x ∉ B}
A′ = U – A = {x : x ∈ U but x ∉ A}
Thus A – B = {1, 4} and B – A = {7, 9, 11}
In the venn diagram shaded region
The Venn diagrams of A – B are shown below.
is complement of A and in this case
we write x ∈ A′ ⇔ x ∉ A.
POWER SET
Let A be a set. Then family of all subsets of A is called
power set of A, which is denoted by P(A) and defined as
P(A) = {R | R ⊂ A}.
e.g. If A = {a, b, c}, then
z Symmetric Difference of two sets
P(A) = {f, {a}, {b}, {c}, {a, b}, {b, c}, {a, c}, A}
The symmetric difference of two sets A and B is denoted
If a set having n elements then it has 2n subsets.
by A D B and defined as the union of A – B and B – A, i.e.,
As A = {a, b, c}, then n(P(A)) = 23 = 8.
A D B = (A – B) ∪ (B – A)
INTERVALS AS SUBSETS OF R = (A ∪ B) – (A ∩ B)
On real line, various types of subsets are designated The Venn diagram of A
as intervals as defined below: D B is shown by shaded
z Closed Interval region as
Let a, b ∈ R such that a < b. Then the set of all real \ x ∈ A ∪ B but
numbers x such that a ≤ x ≤ b is called a closed interval x ∉ A ∩ B.
which is denoted by [a, b]. PROPERTIES OF UNION AND INTERSECTION (ALGEBRA
OF SETS)
i.e. a ≤ x ≤ b (i) A ∪ A = A A∩A=A
z Open Interval (ii) A ∪ B = B ∪ A A∩B=B∩A
(iii) A ∪ (B ∪ C) A ∩ (B ∩ C)
i.e. a < x < b = (A ∪ B) ∪ C = (A ∩ B) ∩ C
z Semi Open or Semi-Closed Interval (iv) A ∪ f = A A∩f=f
]a, b] or [a, b[ (v) A ∪ U = U A∩U=A
(1) PROPERTIES OF COMPLEMENT OF A SET
i.e. a < x ≤ b z U′ = {x ∈ U : x ∉ U} = f
(open left or closed right) z f′ = {x ∈ U : x ∉ f} = U
(2) z (A′)′ = {x ∈ U : x ∉ A′} = {x ∈ U : x ∈ A} = A
i.e. a ≤ x < b (open right or closed left) z A ∪ A′ = {x ∈ U : x ∈ A} ∪ {x ∈ U : x ∉ A} = U
OPERATION ON SETS z A ∩ A′ = {x ∈ U : x ∈ A} ∩ {x ∈ U : x ∉ A} = f
z Union of Sets DE-MORGAN’S LAW
Union of two sets A and B is the set of all those elements If A and B are subsets of universal set X, then we have
which belongs to either A or B or both A and B. The union of two following results:

32 MATHEMATICS TODAY | MAY‘18


z (A ∪ B)′ = A′ ∩ B′ = U – (A ∪ B) (12) Number of elements in exactly one of the sets A, B
read as complement of the and C
union of two sets is equal = S n(A) – 2Sn(A ∩ B) + 3n(A ∩ B ∩ C)
to the intersection of their
RELATIONS
individual complements. The
Venn diagram is shown by shaded region. ORDERED PAIR
A pair of objects in a definite (specific) order is known
z (A ∩ B)′ = A′ ∪ B′ = U – (A ∩ B)
as ordered pair. In the ordered pair (ai, bi), ai is known
r e a d a s c o mp l e m e nt o f
as first element and bi is known as second element. Two
intersection of two sets is
equal to the union of their ordered pairs (a, b) and (c, d) are said to be equal if
individual complements. The and only if a = c, b = d. A set of two elements {a, b}
Venn diagram is shown by shaded region and a ordered pair (a, b) have different meaning i.e.
{a, b} ≠ (a, b) and (a, b) ≠ (b, a).
z For any three sets A, B and C, we have
Let A and B be two non-empty finite sets consisting of
(1) A – (B ∩ C) = (A – B) ∪ (A – C)
m and n elements (members) respectively, then number
(2) A – (B ∪ C) = (A – B) ∩ (A – C)
of ordered pairs in A × B are m × n
(3) A ∩ (B – C) = (A ∩ B) – (A ∩ C)
(4) A ∩ (B D C) = (A ∩ B) D (A ∩ C) CARTESIAN PRODUCT
z If U be a finite universal set and A, B and C are finite Let A and B be two non-empy sets. The Cartesian
(countable) sets, then product of A and B is the set A × B = {(a, b) : a ∈ A,
(1) n(A ∪ B) = n(A) + n(B) – n(A ∩ B) b ∈ B}. If A = {1, 2} and B = {a, b, c}.
(2) n(A ∪ B) = n(A) + n(B) ⇔ A and B are disjoint Then, A × B = [(1, a), (1, b), (1, c), (2, a), (2, b), (2, c)}
non-emtpy sets. In general, A × B ≠ B × A.
(3) n(A – B) = n(A) – n(A ∩ B) ...(i) A×B=B×A⇔A=B
⇒ n(A) = n(A – B) + n(A ∩ B) For any three sets A, B, C (non-empty)
(4) n(A D B) = number of elements which belongs to (i) A × (B ∪ C) = (A × B) ∪ (A × C)
exactly one of A or B (ii) A × (B ∩ C) = (A × B) ∩ (A × C)
= n[(A – B) ∪ (B – A)] = n(A – B) + n(B – A) (iii) A × (B – C) = (A × B) – (A × C)
= n(A) + n(B) – 2n(A ∩ B) (Using (i)) (iv) If A ⊆ B, then A × B ⊆ (A × B) ∩ (B × A)
= n(A ∩ B′) + n(B ∩ A′) (v) If A ⊆ B and C ⊆ D, then A × C ⊆ B × D.
n( A ∩ B ∩ C ′) = n( A ∩ B) − n( A ∩ B ∩ C ) (vi) If A ⊆ B, then A × C ⊆ B × C for any non-empty
  set C.
(5) n(B ∩ C ∩ A ′) = n(B ∩ C ) − n( A ∩ B ∩ C ) (vii) (A × B) ∩ (B × A) = (A ∩ B) × (B ∩ A)
n(C ∩ A ∩ B ′)) = n(C ∩ A) − n( A ∩ B ∩ C )
  Note : (1) If A and B are two non-empty sets each
(6) n((A ∪ B)′) = n(A′ ∩ B′) = n(U) – n(A ∪ B) having m members, then n(A × B) = m2.
(7) n((A ∩ B)′) = n(A′ ∪ B′) = n(U) – n(A ∩ B) (2) If A and B be two non-empty sets having n elements
(8) n(A ∪ B ∪ C) = n(A) + n(B) + n(C) – [n(A ∩ B) in common, then A × B and B × A have n2 elements
+ n(B ∩ C) + n(C ∩ A) – n(A ∩ B ∩ C)] in common.
(9) n(only A) = n(A ∩ B′ ∩ C′) = n[A ∩ (B ∪ C)′] RELATION
= n(A) – n(A ∩ B) – n(A ∩ C) + n(A ∩ B ∩ C)
z Let A and B be two non-empty sets, then a relation
(10) None of the elements of A, B and C
R from A to B is a subset of A × B and we write
= n(A′ ∩ B′ ∩ C′) = n(A ∪ B ∪ C)′
R ⊆ A × B.
= n(U) – n(A ∪ B ∪ C)
(11) Number of elements of exactly two of the three sets z If an ordered pair (a, b) ∈ R then we write a R b
A, B, C and if (a, b) ∉ R then we write a R/ b and we say a is
= n(A ∩ B) + n(B ∩ C) + n(C ∩ A) – 3n(A ∩ B ∩ C) not related to b.
= n(A ∩ B) – n(A ∩ B ∩ C) + n(B ∩ C) z Number of relations : Let A and B be two non-
– n(A ∩ B ∩ C) + n(C ∩ A) – n(A ∩ B ∩ C) empty finite sets consisting of m and n elements
= n(A ∩ B ∩ C ′) + n(B ∩ C ∩ A′) (members) respectively, then number of relations
+ n(C ∩ A ∩ B′) from A to B is 2m × n i.e., 2n(A) × n(B)

MATHEMATICS TODAY | MAY‘18 33


z Domain, Range and Codomain of Relation ⇒ (1, 1) ∈ R and 2 ∈ A ⇒ (2, 2) ∈ R and 3 ∈ R
As we know that relation from the set A to set B is a ⇒ (3, 3) ∈ R.
subset of A × B = {(a, b) : a ∈ A, b ∈ B} Note : Every identity relation is reflexive but every
Let A and B be two non-empty finite sets consisting of reflexive relation may or may not be an identity relation.
m and n elements (members) respectively, then number z Symmetric Relation
of ordered pair in A × B are m × n A relation R is said to be symmetric if
(1) The set of all first elements ‘a’ of the ordered pairs a R b ⇒ b R a i.e. (a, b) ∈ R ⇒ (b, a) ∈ R.
in R (relation) is called the domain of the relation (1) For symmetric relation, R–1 = R
R, and is denoted by DR. (2) Identity relation and universal relation are always
(2) The set of all second elements ‘b’ of the ordered pairs symmetric.
in relation (R) is called the range of the relation R
(3) The examples of symmetric relations are “is parallel
and is denoted by RR.
to”, “is perpendicular to”, “is equal to” etc.
(3) The whole set B is called the co-domain of the
relation (R). z Transitive Relation
e.g. If A = {a, b, c} and B = {1, 2, 3} and a relation A relation R is transitive if (a, b) ∈ R, (b, c) ∈ R implies
R from A to B is R = {(a, 2), (b, 3), (a, 3)}, then (a, c) ∈ R " a, b, c ∈ A.
Domain of R i.e. DR = {a, b} e.g. If L be the set of all straight lines in a plane, then the
Range of relation R i.e. RR = {2, 3} relation ‘is parallel to’ L is transitive relation as for any l1,
and co-domain of the relation = set B = {1, 2, 3} l2, l3, we have l1 || l2 and l2 || l3 ⇒ l1 || l3.
z Equivalence Relation
TYPES OF RELATIONS
The relation R is said to be equivalence if R Satisfies the
(1) Void or null or empty relation in a set : A relation
conditions of reflexivity, symmetricity and transitivity.
in a set A is subset of A × A, the relation R in set A
is called empty relation, if no element of A is related (1) An equivalence relation on a set ‘A’ divide the set
to any element of A i.e. R = f ⊂ A × A into mutually disjoint subsets in such a way that
(2) Identity relation : A relation R is said to be identity every element in a subset is related to each element
relation, if in the set of distinct ordered pairs first in that subset and not related to elements of other
and second elements are identical or equal. i.e. sets. If n(A) = n then we have following results :
relation R in a set A is called identity relation if (2) The number of reflexive relations on A are 2n(n + 1)
R = {(a, a) : a ∈ A} or R = {(a, b) : a, b ∈ A and a = b} n(n+1)
(3) Universal relation : A relation R in a set A is called (3) The number of symmetric relations on A are 2 2
universal relation, if each element of A is related to (4) The number of relations which satisfies the
every element of A, i.e. R = A × A. conditions of reflexivity and symmetricity on set A
Let us consider the relation R on a set A = {1, 2, 3} n(n−1)
given by R = {(a, b) : a, b ∈ A, |a – b| ≥ 0}. are 2 2
Here, all pairs (a, b) in A × A satisfy |a – b| ≥ 0. (5) The number of divisions (partitions) of A into m
so, R is the whole set of A × A. Therefore R is
disjoint subsets are
universal relation.
1
INVERSE RELATION = [mn = C1m (m − 1)n + C2m (m − 2)n .....]
m!
Let A and B be two non-empty sets and R be a relation
from the set A to the set B, then inverse of R, denoted (6) The number of equivalence relations on A are
by R–1, is also a relation from set B to set A and it is 1 n
denoted by
= ∑m!
(m − C1m (m − 1)n + C2m (m − 2)n ....)
R–1 = {(y, x) : (x, y) ∈ R, x ∈ A, y ∈ B} z Anti -Symmetric Relation
SPECIAL TYPES OF RELATIONS A relation R on set A is said to be an anti-symmetric
z Reflexive Relation relation ⇔ (a, b) ∈ R and (b, a) ∈ R gives a = b " a, b ∈ A.
A relation R in a set A is reflexive if each element of A is FUNCTIONS
related to itself i.e. a R a " a ∈ A. Let A and B be two non empty sets, A correspondence
e.g. A = {1, 2, 3}, then the relation R = {(1, 1), (2, 2), between the elements of A and B is said to be a function
(3, 3), (2, 1)} which is a subset of A × A is reflexive as 1 ∈ A from A to B if it satisfies the conditions :

34 MATHEMATICS TODAY | MAY‘18


z All elements of set A are associated to elements in (i) Domain of f = Domain of g
set B (ii) Co-domain of f = codomain of g
z An element of set A is assoiciated to one and only (iii) f(x) = g(x) " x is an element of their common
one element in set B. domain.
If f is function from A to B, we write it as f : A → B. e.g., If f : A → B, A = {2, 3}, B = {7, 12},
z The elements of set A are called pre-images of the f(x) = x2 + 3 and g : A → B, g(x) = 5x – 3, then
elements of set B. f = g as the domain and co-domain of f and g both
equal and f(2) = 7 = g(2), f(3) = 12 = g(3)
z The elements of set B are called images of the
(2) Real valued functions : A function whose domain
element of set A.
and co-domain are subsets of real number, then it
z Every function is a relation but a relation may or
is called real valued function.
may not be function. (3) Bounded function : A real function f(x) is said to
z Domain, Co-Domain and Range of a Function be bounded function if there exist numbers m and
(1) Let f : A → B then set A is known as the domain M such that m ≤ f(x) ≤ M " x ≤ Df. The examples of
of f and set B is known as the co-domain of f. The bounded function are sinx and cosx as –1 ≤ sinx ≤ 1
collection of all f-images of elements of A is known and –1 ≤ cosx ≤ 1.
as the range of the function f. (4) Periodic function : A function f(x) is said to be
(2) The images of element of set A under f is denoted periodic function if there exists a fixed positive
by f(A). Thus range of f = f(A) = {f(a) : a ∈ A} real number a independent of x which satisfies
(3) f(A) is contained in B i.e. f(A) ⊆ B. i.e., range of f is the condition f(x + a) = f(x) " x ∈ domain and a
subset of its co-domain. is called period of the function. In other words, a
(4) Domain and range of a function f are usually function is called periodic function if its each value
abbriviated by Df and Rf respectively. is repeated after a certain interval. The least value of
TYPES OF FUNCTIONS this time interval is called fundament period of the
Generally the functions can be classified into two time function.
category namely (i) Algebraic and (ii) Transcendental. (i) All trigonometric functions are periodic in their
domain. sinx, cosx, secx and cosecx are having
their fundamental period 2p. The period of tanx
and cotx is p.
(ii) sinnx, cosnx, secnx, cosnx are periodic function
with period 2p or p according as n is add
function or even function.
(iii) If f1(x), f2(x), ..., fn(x) are periodic functions with
periods p1, p2, ..., pn respectively then period
of h(x) = f1(x) ± f2(x) ± f3(x) + ...... + fn(x) =
L.C.M. of (p1, p2, p3, .... , pn)if h(x) is not an even
function. If h(x) is even function then period of
1
h(x ) = L.C.M. of (p1, p2, p3, ..., pn)
2
(iv) If a constant be added, subtracted, multiplied in
a periodic function, then period of the function
does not get effected.
(v) If f(x) is periodic with period a, then period
α
of k f[c(x + a)] is . For example period of
c
1 1
2sin3x, sin 3x , 2sin(3x ± 2), sin(3x ± 2) are
2 2
(1) Equal function : Two functions f and g are said to same and equal to 2 π
.
be equal if and only if 3

MATHEMATICS TODAY | MAY‘18 35


(vi) The of 2sinx, 2cosx, 2sinx + 2cosx is 2p (iii) We can check the monotonicity of the
(vii) Every periodic function is many one function. function by using derivates the function
(5) Even function : A function f(x) (x ∈ domain of the y = f(x) be a monotonically increasing
function) is said to even function if it satisfies the dy
function if = f ′(x ) > 0 in its domain and it
condition f(–x) = f(x) e.g., f(x) = e2x + e–2x, f(x) = sin2x, dx dy
f(x) = x4 + x2 + 2 , f(x) = x sinx all are even functions. is monotonically decreasing if = f ′(x ) < 0
(6) Odd function : If a function f(x) satisfies the in its stated domain. dx
condition f(–x) = –f(x)" x ∈ the domain of f(x) is z Algebraic function : y is said to be algebraic
called odd function. function of x, if it satisfies an algebraic equation of
e.g., f(x) = e 2x – e –2x , f(x) = x 3, f(x) = x 3 cosx, the form A0(x)yn + A1(x)yn – 1 + A2(x)yn – 2 + ... +
f(x) = x + sin x all are odd functions. An – 1(x)y + An(x) = 0, where n is natural number and
z Every even function y = f(x) is A0(x), A1(x), A2(x), ..., An(x) are polynomials in x.
symmetric al about y-axis which z All polynomial function are algebraic but all
can be observed by plotting the algebraic functions need not be a polynomial.
graph of function. The graph z Rational function : If p(x) = a0xn + a1xn – 1 + ... +
of y = x2 is symmetrical about an and q(x) = b0xn + b1xn – 1 + b2xn – 2 + ... bn be
y-axis therefore it is even function. p(x )
z The graph of odd function is two polynomials. Then f (x ) = where q(x) ≠ 0
is called rational function. q(x )
symmetrical about the origin.
The graph of f(x) = x, x3 .... are z Implicit and explicit function : Let x be an
symmetrical about origin. independent variable and y is dependent on x, the
z Every even function is many one function and function is said to be explicit if y can be expressed in
z The derivative of odd function is even and derivative the form of x directly. e.g., y = x2 + 3x – 1, y = x3 + x
of even function is odd. etc., but when y can not be directly expressed in
Remark : Every function can be expressed as the terms of x (i.e. in term of independent variable),
sum of an even and odd function. then function is said to be implicit
f ( x ) + f (− x ) f ( x ) − f (− x ) e.g., y = x2 + y2 + 2xy , xy2 + yx2 + x3 + y3 = 0 etc.
i.e. f (x ) = +
2 2 COMPOSITION OF FUNCTIONS
= h(x) + g(x) (say) where h(x) is even and g(x) If f : A → B and g : B → C, then the function h : A → C
is odd defined as h(x) = gof(x) g(f(x)) " x ∈ A is composition
(i) The product of two even functions is even. of f and g denoted by gof.
(ii) The sum and difference of two odd function is (i) Domain of gof(x) i.e., g{f(x)} = {x : x ∈ D f ,
an odd function. f(x) ∈ Dg}
(iii) The sum and difference of two even function is (ii) Domain of fog(x; i.e., f(g(x)) = {x : x ∈ Dg, g(x) ∈ Df}
an even function.
(iv) The product of two odd functions is an even
function.
(v) The sum of an odd and an even function is
neither odd or even function.
(vi) Every constant function is both even and odd.
z Monotonic function : A function which is either
increasing or decreasing in a certain domain, where
it is defined is called monotonic function. z Properties of Composite Function
(i) A function f(x) is said to be monotonically (1) If f and g both functions are even, then h = fog is also
increasing function if as we increase x, the value an even function.
of f(x) also increases and vice-versa (2) If f and g both are odd functions, then h is also an
(ii) A function f(x) is said to be monotonically odd function.
decreasing function if as x increases then f(x) (3) If any one of the function is even/odd, then h is even
also decreases. function.

36 MATHEMATICS TODAY | MAY‘18


(4) In general fog ≠ gof (1) All even functions are many-one function.
(5) (fog)oh = fo(goh) (Associative property) e.g., y = x2 + 3, y = cos x.
(6) gof exist only if Rf is subset of Ig (2) All periodic functions are many-one e.g. y = sinx,
(7) If f and g are one-one/ onto, then fog and gof/are tanx i.e. all trigonometric functions defined in their
also one-one/onto. domain are many-one.
(3) If a line drawn parallel to x-axis cuts the graph of
SOME SPECIAL TYPE OF FUNCTIONS AND THEIR
f(x) function at more than one-point then f(x) is
PICTORIAL REPRESENTATION/ DIAGRAM.
said to be many one.
z One-one or Injective Function z Onto Function
A mapping f from the set A to set B is said to be one- A mapping f : A → B is said to be onto if each element of
one if each element in the domain of a function has a B is the image of at least one element of A.
distinct image in its co-domain. In other words, if no or A function f : A → B is said to be onto if range of f =
element of B is the image of more than one element A. co-domain of f.
(1) If x1, x2, ∈ A such that x1 ≠ x2 ⇒ f(x1) ≠ f(x2),
f f
then we called the function one-one or injective A B A B
a a1 1 4
function.
b 2 5
(2) If n(A) = m, n(B) = n and m ≤ n, then total number b1
c
n! d c1 3 6
of one-one functions from A to B is nPm =
(n − m)!
If A and B are two sets having m & n elements respectively
(a) Method of check the injectivity of a function
such that 1 ≤ n ≤ m, then number of onto function from
z Take two arbitrary members a, b in the domain
A to B is
of f.
n n  n 
n−r n
z Solving f(a) = f(b), if gives a = b only, then we say f : ∑ (−1) Cr r m = nm −   (n − 1)m +   (n − 2)m +
1  2
A → B is a one-one function, otherwise not. r =1
(b) To check the injectivity by graphical method, n − 1 n  m
.... + (−1)1
plot the graph of y = f(x), if each line drawn parallel to 1 
x-axis cuts the graph exactly at one point then we called Note : Total number of functions from A to B are nm
the function 1-1 function. (All Distinct)
(c) To check the injectivity by using calculus. z One-One onto function
(1) If domain of f(x) is continuous and f ′(x) > 0 or < 0 for A function f : A → B is said to be one-one onto function
all values of x in the stated domain than f is one-one if it satisfies the conditions of one one and onto both.
function. It is also called bijective functions. Let f be a bijection
(2) Number of one-one (injective) functions is 0 if from set A to set B if
n(A) > n(B) (i) f(x) = f(y) ⇒ x = y " x, y ∈ A (condition of 1-1)
(ii) " y ∈ B, there exists x ∈ A such that f(x) = y
z Many One Function
(condition of onto)
A mapping f : A → B is said to be many one if two or
Note : (1) Number of one-one onto functions is m!,
more elements of the domain have the same image in its
where m = n(A) = n(B)
co-domain or f : A → B said to be many one function if (2) Number of one-one onto functions is 0 if
there exist a, b ∈ A such that a ≠ b but f(a) = f(b). n(A) ≠ n(B)
A B
f z Into function
1 4
A function f : A → B is said to be into function if
–1 12
there exist at least one element in B having no pre-
3 7
image in A. The following arrow diagrams shows into
2 19
functions

MATHEMATICS TODAY | MAY‘18 37


f z Square function : The function
B A B
A defined as f(x) = y = x2 is called
f
x1 y1 x1 1 square function, Df = R (set of
x2 y2 x2 2 all real values)
x3 y3 3 Rf = R+ (set of all positive real numbers) ∪ {0}
x3
y4 4 which is also called parabolic functions passes
through origin and symmetrical about y-axis.
Note : Into function can not be onto function & vice-versa.
z Extention of even function : f(x) = x2n, " n ∈ N, the
z Many one-into and Many one-onto Function domain of f is set of all real numbers i.e. Df = R and
A function f : A → B is said to be many one into
Rf (range of f) = R+ ∪ {0}. The graph is symmetrical
function if it satisfies the condition of many one as
about y-axis
well as into function. The similar definition is used
for many one-onto function.
INVERSE FUNCTION
Let f : A → B be a one-one and onto function, defined
by b = f(a), let there exists a unique function g : B → A
such that for each b ∈ B, g(b) = a. The function g so
defined is said to be inverse of f and denoted by f–1. It is
to be remember that f is inverse of g
f and f–1 are symmetric about the line y = x
SOME IMPORTANT FUNCTIONS AND THEIR INVERSE
Functions Inverse functions z Cubical function : A function y = x3 is called cubic
1. f : R → ( 0 , ∞ ) 1. f–1 : (0, ∞) → R defined or cubical function passing through origin.
defined by by f–1 (x) = logax Df = Rf (set of real numbers)
x
f(x) = a , a > 0 Graph of cubic function is
2. f : R → R, 2. f–1 : R → R defined by symmetrical about opposite
f(x) = x 3 f–1(x) = x1/3 quadrant though origin (see
graph)
3. f : R → (0, ∞) 3. f–1 : (0, ∞) → R defined
defined by f(x) = e x by f –1 (x) = logx
z Graph of f(x) = x2n – 1, n ∈ N
If n ∈ N, then the function y = f(x) = x2n – 1 is
4. f : R → R, 4. f –1 : R → R given by
an odd function whose Df = Rf = set of all real
f(x) = cx + d , a ≠ 0 x −d
f–1(x) = numbers.
c
5. f : [0, p] → [–1, 1] 5. f –1 : [–1, 1] → [0, p]
f(x) = cosx defined by f –1 (x) =
cos–1x
 −π π   −π π 
6. f :  , → R 6. f–1 : R →  , ,
 2 2   2 2 
f(x) = tanx f–1(x) = tan–1x
SOME FUNCTIONS AND THEIR GRAPH
z Identity Function : A function f defined by
The graph of the function is symmetrical about
f(x) = x " x ∈ R is called the identity function. Here,
origin or the opposite quadrants.
Df = Rf (set of real numbers).
Behaviours of f(x) in various intervals is given as
z x ∈ (–∞, –1) , x > x3 > x5 > ....
z x ∈ (–1, 0) , x < x3 < x5 < ....
z x ∈ (0, 1) , x > x3 > x5 > ....
z x ∈ (1, ∞) , x < x3 < x5 < ....

38 MATHEMATICS TODAY | MAY‘18


z Graph of f (x) = 1/x, the function f(x) = 1/x, x ≠ 0 The modulus function is nowhere discontinuous.
called the reciprocal function or the rectangular It is many one function.
function. z Signum function : The signum function is denoted
by y = sgn (x) and defined by
| x |
 x ≠0
y = sgn( x ) =  x
 0, x = 0

 1 if x >0

=  0 if x =0
−1 if x <0

z Df = Rf is R – {0}
z f(x) is odd function, its graph is symmetrical
about opposite quadrants.
z When x → 0+ ⇒ f(x) → +∞ and
x → 0– ⇒ f(x) → –∞.
z When x → ±∞, f (x) → 0.
z Graph of f(x) = 1/x2 (i) If f(x) = R and range of f(x) = {–1, 0, 1}
(ii) Signum function is discontinuous and many
one function.
z Greatest integer function or floor function or
step function
If f (x) = k " x ∈ [k, k + 1] where k ∈ I, then f is
called greatest integer function usually denoted
by f(x) = [x].

z The function f(x) = 1/x2 is an even function.


z The graph of the function is symmetrical about
y-axis.
z Df = R – {0}, Rf = (0, –∞)
z y → ∞ as lim f (x ) or lim f (x )
x →0+ x →0−
z Modulus function : A function f(x) is said to be
modulus function if for all values of x the values of y
are positive. It is denoted by f(x) = |x| and defined as Df = R (set of real numbers)
− x , x < 0 Rf = set of integers, the function is discontinuous.
f (x ) = | x | =  z f(x) = [x] could be expressed as
 x, x ≥ 0 Df = x Rf = [x]
For –1 ≤ x < 0 –1
For 0 ≤ x < 1 0
For 1 ≤ x < 2 1 and so on
z Fractional Part Function
The fractional part function is denoted by y = {x} where
{ } denotes the fractional part of x.
We know that, x = [x] + {x}
Df = R = integral part + fractional part
Rf = [0, ∞) \ y = {x} = x – [x] where 0 ≤ {x} < 1

MATHEMATICS TODAY | MAY‘18 39


Table to draw the graph z Drawing the graph of y = a f(x) from the graph of
y = f(x). In such a graph the abscissa point
x {x} (x-coordinates) remains same but points of
–2 ≤ x < –1 x+2 ordinates have their ratio 1 : a.
–1 ≤ x < 0 x+1 1
The graph of sin x with the help of y = sinx is
given as 2
0≤x<1 x
1≤x<2 x–1
2≤x<3 x–2

z When f(x) transforms to f(–x) i.e. f(x) → f(–x)


To draw the graph of y = f(–x), take the image of
the graph of the curve y = f (x) as y-axis on plane
mirror i.e. turn the graph of f (x) by 180° about
y-axis Graphically
z Properties of fractional part of x
(1) {x} = x if 0 ≤ x < 1
(2) {x} = 0 if x ∈ integer thus
0, x ∈ I
{x} + {− x} = 
1, x ∉ I
(3) {–x} = 1 – {x}; x ∉ integer e.g. y = f (x) = x3
GRAPHICAL TRANSFORMATION
When f(x) transforms to f(x) ± a, a > 0. We need to
draw the graph of f(x) + a and f(x) – a.
z For the graph of f(x) + a, shift the graph of f (x)
upward through a unit and for the graph of f(x) – a,
shift the graph of f(x) downward through unit a.
z When f(x) transforms – f(x) i.e. f(x) → –f(x). To draw
the graph of y = –f(x) take image of y as the x-axis
on plane mirror, which means turn the graph of f
(x) by 180° about x-axis.
e.g. To draw y = –logx where y = logx is known.

e.g. y = e2x, then e2x + 1 and e2x – 1 are shown as

e.g. To draw the graph of y = –{x} where { } denotes


the fractional part of x.

Graph of e2x + 1 is obtained by shifting the graph


of y = e2x upwards by 1 unit and the graph of
e2x – 1 is obtained by shifting the graph downwards
by 1 unit.

40 MATHEMATICS TODAY | MAY‘18


on
Exahm ay
t M
20

PAPER-1
SECTION-1 (a) is discontinuous at x = – 1
ONE OR MORE THAN ONE OPTION CORRECT TYPE (b) is continuous at x = 2
1. Let f : [– 1, 1] onto [3, 5] be a linear polynomial. (c) has a removable discontinuity at x = 1
Which of the following can be true? (d) has an irremovable discontinuity at x = 1.

 −1  7 −1  15  1 5. The expression (a tan g + b cot g) (a cot g + b tan g)


(a) f   = (b) f   = – 4ab cot2 2g is
2  2 4 4
1  −1  (a) independent of a, b
(c) f(0) ≠ 4 (d) f   + f   = 8 (b) independent of g
2 2 
(c) dependent on g
2. A normal coin is tossed four times. Two events E (d) dependent on a, b
and F are defined as
E : no two consecutive heads occur in 4 tosses. 6. The parabola x = y2 + ay + b intersect the parabola
F : At least 2 consecutive heads occur in 4 tosses. x2 = y at (1, 1) at right angle. Which of the following
The events E and F are is/are correct?
(a) equally likely (a) a = 4 and b = – 4
(b) mutually exclusive (b) a = 2 and b = – 2
(c) exhaustive (c) Equation of the directrix for the parabola
(d) such that one is twice as likely to occur as other. x = y2 + ay + b is 4x + 1 = 0.
3. If the expression (1 + r i)3 is of the form of s(1 + i) (d) Area enclosed by the parabola x = y2 + ay + b
and its latus rectum is 1/24.
for some real 's' where 'r' is also real and i = −1 ,
then the value of 'r' can be 7. The position vectors of the vertices A, B and C of
π a tetrahedron are (1, 1, 1), (1, 0, 0) and (3, 0, 0)
(a) cot (b) sec p respectively. The altitude from the vertex D to the
8
opposite face ABC meets the median line through A
π 5π
(c) tan (d) tan of the DABC at a point E. If the length of side AD is
12 12
n 2 2
4 and volume of the tetrahedron is , then the
4. Given f (x) = ∑ (x r −r 2
+ x ) ; x ≠ ±1 and 3
r =1 correct statement(s) is/are

g (x ) = n→∞
( )
 Lim ( f (x ) − 2n)x −2n−2 (1 − x2 ) ; x ≠ ±1 (a) The altitude from the vertex D is 2.
(b) There is exactly one position for the point E.
 −1, ; x = ±1
(c) There can be two positions for the point E.
then g (x) (d) Vector j − k is normal to the plane ABC.

Sanjay Singh, Mathematics Classes, Chandigarh, Ph : 9888228231, 9216338231


MATHEMATICS TODAY | MAY‘18 41
SECTION-2 slope m1 and m2 of C3 are intersected at the centre
of C5, then compute the value of 5 |m1 m2|.
INTEGER ANSWER TYPE
11. For a positive constant t, let a, b be the roots of the
8. Let a, b, g be distinct real numbers such that quadratic equation x2 + t2x – 2t = 0. If the minimum
aa2 + ba + c = (sin q) a2 + (cos q)a 2 
 1  1  1  a
value of ∫   x +   x +  +  dx is +c
ab2 + bb + c = (sin q) b2 + (cos q)b  α2
 β 2
 αβ b
−1  
ag2 + bg + c = (sin q) g2 + (cos q)g where a, b, c ∈ N, then find the least value of (a + b + c)/10.
(where a, b, c, ∈ R.)  3
1  4 −3
Then the maximum value of the expression 12. Let A =  2  , B =  
   −2 2 
a2 + b2 1 2 
is
a2 + 3ab + 5b2 r ⋅ 3r 2r 
and Cr =  
9. The integer satisfying the inequation  0 (r − 1)3r 
log12/2 x + 4 log2 x < 2 (4 − log16 x 4 ) is 50
10. In a sequence of circles C1, C2, C3, ....... Cn; the be given matrices. If ∑ tr.((AB)r Cr ) = 3 + a ⋅ 3b
centres lie along positive x-axis with abscissae r =1
forming an arithmetic sequence of first term unity where tr.(A) denotes trace of matrix A, then find the
and common difference 3. The radius of these 1
value of (a + b). [Where a and b are relatively
circles are in geometric sequence with first term 20
prime]
unity and common ratio 2. If the tangent lines with
SECTION-3
MATRIX MATCH TYPE
Answer Q.13, Q.14, Q.15 by information given below:
Column I contains information about the family of curves
Column II contains the differential equation of the family of curve.
Column III contains relation between the order (O) and degree (D) of differential equation of the family of
curve. where c1, c2, c3, c4, c5, c6 are arbitrary constant.
Column I Column II Column III

d3 y d2 y dy
(A) y = c1 sin2x + c2 cos2x + c3 sin2x + c4 cos 2x (I) − + − y =0 (i) OD + DO = 3
3 2 dx
dx dx
1 dy 1
(B) y = (c1 + c2)sin(x + c3) – c4e x +c5 +c6 (II) + =0 (ii) OD + DO = 4
(1 + x 2 ) (1 + y 2 ) dx

d3 y dy
(C) 1 + x 2 + 1 + y 2 = c1(x 1 + y 2 + y 1 + x 2 ) (III) +4 =0 (iii) OD + DO = 2
3 dx
dx

d2 y dy
(D) y = c1e3x + c2e5x (IV) −8 + 15 y = 0 (iv) 2OD + 3DO = 9
2 dx
dx

13. Which of the following options is the only correct 14. Which of the following options is the only correct
combination? combination?
(a) (A) - (III) - (ii) (b) (B) - (III) - (i) (a) (A) - (II) - (ii) (b) (B) - (III) - (iv)
(c) (C) - (I) - (iv) (d) (D) - (II) - (i) (c) (C) - (III) - (iv) (d) (D) - (IV) - (i)

42 MATHEMATICS TODAY | MAY‘18


15. Which of the following options is the only incorrect combination?
(a) (B) - (I) - (iv) (b) (C) - (II) - (iii)
(c) (D) - (I) - (i) (d) (A) - (III) - (ii)
Answer Q.16, Q.17, Q.18 by information given below:
Column I contains information about the incident ray from a point on a curve at given point of the curve.
Column II contains the equation of reflected ray after the first reflection.
Column III contains the equation of reflected ray after the second reflection.
Column I Column II Column III
A ray emanating from (0, – 5 ) incident on
(A) the curve 9x2 + 4y2 = 36 at the point with (I) 2x + y 5 + 2 5 = 0 (i) x + 2y + 5 = 0
abscissa 2.
A ray emanating from ( 5 , 0) incident on
(B) the curve 4x2 + 9y2 = 36 at the point with (II) 2x + y − 2 5 = 0 (ii) 4 5x + y + 5 = 0
ordinate –2
A ray emanating from (– 5 , 0) incident on
(C) the curve 4x2 + 5y2 = 100 at the point with (III) 5x + 2 y − 2 5 = 0 (iii) x + 4 5y − 5 = 0
ordinate 2 5

16. Which of the following options is the only correct (a) (B) - (II) - (i) (b) (B) - (I) - (iii)
combination? (c) (B) - (III) - (i) (d) (B) - (II) - (ii)
(a) (A) - (I) - (ii) (b) (A) - (III) - (ii) 18. Which of the following options is the correct
(c) (A) - (III) - (i) (d) (A) - (II) - (ii) combination?
17. Which of the following options is the only correct (a) (C) - (II) - (iii) (b) (C) - (I) - (ii)
combination? (c) (C) - (III) - (iii) (d) (C) - (II) - (i)

PAPER-2
SECTION-1  1 1
(c) 2  tan −1 + tan −1 
 3 2
SINGLE OPTION CORRECT TYPE
(d) 2 tan–13
1. Number of 4 digit numbers of the form N = abcd
which satisfy following three conditions 4. The position vector of a point in which a line
(i) 4000 ≤ N < 6000 through the origin perpendicular to the plane
 ^ ^ ^
(ii) N is a multiple of 5 2x – y – z = 4 meets the plane r .(3 i − 5 j + 2 k) = 6,
(iii) 3 ≤ b < c ≤ 6 is equal to is
(a) 12 (b) 18 (c) 24 (d) 48 (a) (1, – 1, – 1) (b) (– 1, – 1, 2)
2. The radii of the escribed circles of DABC are ra, rb  4 −2 −2 
and rc respectively.
(c) (4, 2, 2) (d)  3 , 3 , 3 
If ra + rb = 3R and rb + rc = 2R, then the smallest n 
5. Let   represents the combination of 'n' things
angle of the triangle is k 
1 −1
(a) tan −1 ( 2 − 1) (b) tan ( 3 ) taken 'k' at a time, then the value of the sum
1 2
(c) tan −1 ( 2 + 1) (d) tan −1 (2 − 3 )  99   98   97  3 2 
2 + + + +
 97   96   95  ....  1  +  0  equaals
3. The angle between pair of tangents drawn to the
curve 7x2 – 12y2 = 84 from M(1, 2) is
 99  100   99  100 
1 (a)   (b)  (c) (d)
(a) 2 tan −1 (b) 2 tan–12  97   98   98  
 97 
2
MATHEMATICS TODAY | MAY‘18 43
6. Let a, b ∈ R. If a, b2 be the roots of quadratic (a) I1 > I0 (b) I2 < I0
equation x2 – px + 1 = 0 and a2, b be the roots of (c) I0 + I1 + I2 > 3 (d) I0 + I1 > 2
quadratic equation x2 – qx + 8 = 0, then the value 11. A circle having its centre at (2, 3) is cut orthogonally
r by the parabola y2 = 4x. The possible intersection
of 'r' if be arithmetic mean of p and q, is
8 point(s) of these curves, can be
83 83 83
(a) (b) (c) (d) 83 (a) (3, 2 3 ) (b) (2, 2 2 )
8 4 2
(c) (1, 2) (d) (4, 4)
7. Let a, b, c, d are non-zero real numbers such that 3 π2
6a + 4b + 3c + 3d = 0, then the equation ax3 + bx2 12. If (sin–1x)2 + (sin–1y)2 + (sin–1z)2 = , then the
value of (x – y + z) can be 4
+ cx + d = 0 has
(a) atleast one root in [ –2, 0] (a) 1 (b) – 1 (c) 3 (d) – 3
(b) atleast one root in [ 0, 2] 13. Which of the following is/are true?
(c) atleast two roots in [ –2, 2] The circles x2 + y2 – 6x – 6y + 9 = 0 and x2 + y2 +
(d) no root in [ –2, 2] 6x + 6y + 9 = 0 are such that
SECTION-2 (a) they do not intersect.
(b) they touch each other.
ONE OR MORE THAN ONE OPTION CORRECT TYPE
(c) their exterior common tangents are parallel.
8. Consider the function f(x) = sin5x + cos5x – 1, (d) their interior common tangents are perpendicular.
 π 14. A line L passing through the point P(1, 4, 3), is
x ∈ 0,  . Which of the following is/are correct?
 2 perpendicular to both the lines
 π x −1 y + 3 z − 2 x + 2 y − 4 z +1
(a) f is monotonic increasing in  0,  . = = and = =
 4 2 1 4 3 2 −2
π π If the position vector of point Q on L is (a1, a2, a3)
(b) f is monotonic decreasing in  ,  .
4 2 such that (PQ)2 = 357, then (a1 + a2 + a3) can be
 π (a) 16 (b) 15 (c) 2 (d) 1
(c) $ some c ∈  0,  for which f ′(c) = 0.
2 SECTION-3
 π
(d) The equation f(x) = 0 has two roots in 0,  . COMPREHENSION TYPE
 2
9. Which of the following statement(s) is/are correct? Paragraph for Question No. 15 and 16
(a) Rolle's theorem is applicable to the function Let f(x) = 4x4 – 24x3 + 31x2 + 6x – 8 be a polynomial
F(x) = 1 – 5 x 6 on the interval [–1, 1]. function. Suppose f(x) = 0 has four roots
(b) The domain of definition of the function a < b < g < d. If sum of two of its roots vanishes, then
log 4 (5 − [x − 1] − [x]2 ) x−δ
α +β + δ
F(x) = is
x2 + x − 2
15. ∫  x − γ  dx is
(–3, –2) ∪ (–2, 1) ∪ (1, 2) 24 16 16
(where [x] denotes the largest integer less than (a) x − 8 ln | x − 2 | − + − +C
x − 2 (x − 2)2 3(x − 2)3
or equal to x)
(c) The value of a for which the function F(q) = a sin q 24 16 16
(b) x − 16 ln | x − 2 | − + − +C
1 (x − 2) (x − 2)2
(x − 2)3
+ sin 3q has an extremum at q = p/3 is –2.
3 2010 24 16 16
{x + k} (c) x + 16 ln | x − 2 | − + − +C
(d) The value of ∑ is {x} 2
2010 x − 2 (x − 2) 3(x − 2)3
k =1
(where {x} denotes the fractional part of x). 24 16 16
(d) x + 8 ln | x − 2 | − − + +C
2
x − 2 (x − 2) 3(x − 2)3
3
dx
10. Let In = ∫ 1 + xn (n = 1, 2, 3 ...... ) and lim = I0 (say),
16.
2β δ +1
x − 5x γ +1 + 2β | x | +1
0
n→∞
∫ x 2 + 4β | x | +1
dx is
then which of the following statement(s) is/are correct? 2α

44 MATHEMATICS TODAY | MAY‘18


(a) ln2 (b) 2ln 2 n n
1
1 = ∑ x 2r + ∑ + 2n
(c) ln 2 (d) None of these r =1 r =1 x
2r
2
Paragraph for Question No. 17 and 18  1 1 1 
Let a, b, c are the sides of triangle ABC satisfying = (x 2 + x 4 + .... + x 2n ) +  + + ..... +  + 2n
x 2
x 4
x 2n 
 c 
log 1+  + l o g a – l o g b = l o g 2 . A l s o t h e 1 
 a 1−
x (1 − x ) 1  x 2n 
2 2n 
a(1 – x2) + 2bx + c(1 + x2) = 0 has two equal roots. = + + 2n
(1 − x 2 ) x2 1 − 1
17. Measure of angle C is
x2
(a) 30° (b) 45° (c) 60° (d) 90°
18. The value of (sin A + sin B + sin C) is equal to x2 (1 − x2n ) (1 − x2n )
f (x ) = + + 2n
5 12 8 1 − x2 (1 − x2 )x2n
(a) (b) (c) (d) 2
2 5 3 (1 − x2n )  2 1 
f (x ) = 2 
x + 2n  + 2n, x ≠ ± 1
SOLUTIONS 1− x x 
PAPER-1  1 
∴ ( f (x ) − 2n)(1 − x 2 ) = (1 − x 2n )  x 2 + 
1. (a, b, d) : Let f(x) = ax + b  x 2n 
Case-I: f is increasing
Now consider g (x ) = Lim (( f (x ) − 2n)
f(–1) = 3 and f(1) = 5 n→∞
⇒ – a + b = 3 and a + b = 5
⇒ f(x) = x + 4 ⇒ f –1(x) = x – 4
x −2n−2
)
(1 − x2 ) for x ≠ ±1
Case-II: f is decreasing  1  1
f(–1) = 5 and f(1) = 3 = Lim (1 − x 2n )  x 2 + ⋅ ; x ≠ ±1
n→∞  x 2n  x 2n+2
⇒ –a + b = 5 and a + b = 3
⇒ f(x) = 4 – x ⇒ f –1(x) = 4 – x (1 − x 2n )(x 2n+2 + 1)
= Lim
Now we can verify the options. n→∞ (x 2n )(x 2n+2 )
2. (a, b, c) : Sample space S is given by
{H T H H, H T T H, H T H T, H T T T,  1  1 
= Lim  −1 + 2n  1 + 2n+2 
T H H H, T H T H, T H H T, T H T T, n→∞  x  x 
T T H H, T T T H, T T H T, T T T T,
H H H H, H H T H, H H H T, H H T T} −1 , | x |> 1

8 1 Now, g (x ) = ∞ , | x |< 1
\ P ( E ) = = = P (F ) −1 , x = ±1
16 2 
3. (b, c, d) : (1 + r i)3 = s(1 + i) Now clearly g has non removable infinite type of
1 + 3r i + 3r2i2 + r3i3 = s(1 + i) discontinuity at x = 1 and – 1
1 – 3r2 + i(3r – r3) = s + si g is continuous at x = 2 ⇒ a, b, d
⇒ 1 – 3r2 = s = 3r – r3 5. (b, d) : We have,
Hence 1 – 3r2 = 3r – r3 (a tan g + b cot g) (a cot g + b tan g) – 4ab cot2 2g
⇒ r3 – 3r2 – 3r + 1 = 0
⇒ (r3 + 1) – 3r(r + 1) = 0 cos2 2 γ
= a2 + (tan2 g + cot2 g) ab + b2 – 4ab
⇒ (r + 1)(r2 + 1 – r – 3r) = 0 sin2 2 γ
⇒ r = – 1 or r2 – 4r + 1 = 0   sin2 γ cos2 γ  4(cos2 γ − sin2 γ )2 
= a2 + b2 + ab   + − 
4 ± 16 − 4 4 ± 2 3   cos2 γ sin2 γ  4 sin2 γ cos2 γ 
⇒ r= =
2 2 = a2 + b2 + ab
⇒ r = 2 + 3 or 2 − 3  (sin4 γ + cos 4 γ ) − (cos 4 γ + sin 4 γ − 2 sin2 γ cos2 γ ) 
n  
 1 
4. (a, b, d) : f (x ) = ∑  x 2r + 2r + 2   sin2 γ cos2 γ 
r =1
 x 

MATHEMATICS TODAY | MAY‘18 45


COMPLEX NUMBERS CONTINUITY

BASIC TERMS CONTINUITY AT A POINT DISCONTINUITY AT A POINT


Number of the form
For complex number z = x + iy  A function f (x) is said to be continuous at a point x = a A function f (x) which is not continuous at point say (x = a),
z = a + ib where a, b ∈ R, in its domain iff Lim f (x ) exist finitely, f (a) is a finite then it is discontinuous at x = a.
DEFINITION a = Real part and  Conjugate : z = x − iy  Modulus : | z | = x 2 + y 2 x →a

b = Imaginary part  y number and Lim f (x ) = f (a) .


 Argument : tan −1   x →a
 x
 In open interval : In an open interval (a, b), f (x) is Types
continuous if it is continuous at every point between (a, b).
 In closed interval : In a closed interval [a, b], f (x) is  Removable discontinuity :
SOME FACTS ABOUT LOCUS Properties continuous if Discontinuity at x = a Lim f (x ) ≠ f (a) . It is called
x →a
• f (x) is continuous in (a, b) removable because it can be made continuous by
If z is a variable point and z1, z2 are two fixed points in the For z1 = x1 + iy1, z2 = x2 + iy2
• Lim f (x ) = f (a), Lim f (x ) = f (b) redefining it at point a.
Argand plane :
 z1 + z2 = z1 ± z2  z1z2 = z1 z2 x →a + x →b −  Discontinuity of 1st kind : Lim f (x ) ≠ Lim f (x )
 |z – zl| = |z – z2| represents perpendicular bisector of the x →a − x →a +
n
 (z ) = (z )
n
 | z | = | z | = | −z |  Continuity everywhere : A function is said to be
segment joining z1 and z2.
n n continuous everywhere if it is continuous on the entire  Discontinuity of 2 kind : Lim f (x ) or Lim or both
nd

 |z – zl| + |z – z2| = K (a fixed quantity > 0) ...(i)  zz = | z |2  |z |=|z | real number line (– ∞, ∞). x →a − x →a +
do not exist.
• If K > |z1 – z2| then (i) represents an ellipse.  | z1 + z2 + z3 + ... + zn | ≤ | z1 | + | z2 | + ... + | zn |
• If K = |z1 – z2| then (i) represents the segment joining  arg | z1z2 | = arg z1 + arg z2  arg | z1 / z2 | = arg z1 − arg z2
z1 and z2.
 arg | z n | = n arg(z ).
• If K < |z1 – z2| then (i) does not represent any curve in
the Argand plane.
RESULTS
 |z – zl| – |z – z2| = K(> 0)
GEOMETRICAL APPLICATIONS ALGEBRA ON
• If K ≠ |z1 – z2| then |z – z1| – |z – z2| = K represent a
CONTINUOUS Identity function, Modulus function, Constant function,
hyperbola with foci at z1 and z2.
Exponential function, Logarithmic function, Polynomial functions
• If K = |z1 – z2| then |z – z1| – |z – z2| = K represents a Triangle FUNCTIONS are continuous.
straight line joining z1 and z2 but excluding the segment
Triangle ABC with vertices A(z1), B(z2), C(z3) is equilateral if
joining z1 and z2.
and only if
 |z – zl|2 + |z – z2|2 = K = |z1 – z2|2 represent a circle with affixes 1 1 1
1 + + =0
z1 and z2 the extremities of a diameter and K ≥ | z1 − z2 |2 z1 − z2 z2 − z3 z3 − z1 A(z ) The largest (greatest) integer function [x] is continuous at all
2 1
points except at integer (integral) points.
 |z – z1| = K|z – z2|, (K ≠ 1), then locus of z is a circle, ⇔ z12 + z22 + z32 = z1z2 + z2 z3 + z3z1
z − z1 1 z1 z2
i.e. = K (K ≠ 0) represent a circle, for K = 1, Let f (x) & g (x) be two continuous
z − z2 ⇔ 1 z2 z3 = 0
represent a straight line. B(z2) C(z3) functions on their common domain
1 z3 z1
D & a be any real number then
 z − z1  All trigonometric function are continuous in their respective
 arg  = α (a fixed quantity) then locus of z is a  a f (x) is continuous
 z − z2  domains like sin x, cos x are continuous ∀ x R.
circle. Circle  f ± g is continuous
 z − z1  π  The equation of circle whose centre is at point having affix  f g is continuous
 arg  = ± , then locus of z is a circle as z1 and z2 are
 z − z2  2 z0 and radius R is |z – z0| = R.  f /g is continuous provided f g
vertices of the end point of the diameter. or zz − z0 z − z0 z + z0 z0 − R2 = 0 If f, g are continuous functions,
g(x) ≠ 0 for any x ∈ D
then fog & gof are continuous. a f (a) g (f (a))
 z − z1  ⇒ zz + az + az + b = 0 where a = –z0 and b = |z0|2 – R2  f n (x), ∀ n ∈ N is continuous. If f is continuous at a point
 arg  = 0 or π then locus of z is a line passing
 z − z 
2
⇒ zz + az + az + b = 0 represents a circle having centre –a x = a & g is continuous at f (a), continuous
through the points z1 and z2. and radius R = 2
| a | − b. then gof is continuous at x = a. gof
= a2 + b2 + 2ab = (a + b)2 , which is independent of λ
g and dependent on a, b. ⇒ = 2 or − 2
λ +1
6. (c, d) : Given curves, x = y2 + ay + b; x2 = y passes
\ These are two positions for E which are (–1, 3, 3)
through (1, 1)
and (3, –1, –1).
\ 1 =1 + a + b ⇒ a + b = 0
dy dy 8. (2) : Let the equation be
Also, 1 = 2 y + a ax2 + bx + c = (sinq) x2 + (cosq) x
dx dx ⇒ x2 (a – sinq) + x (b – cosq) + c = 0
dy  1 dy dy  Then roots of equation are a, b, g
= ; For y = x 2 ; = 2x; =2
dx (1, 1) 2 + a dx dx (1, 1) Since quadratic has three roots then it must be an
Q Given curves intersect at right angles identity so.
2 a – sinq = 0, b – cosq = 0 and c = 0
∴ = −1 \ a = sinq, b = cosq, c = 0.
2+a
a 2 + b2
⇒ – 2 = 2 + a ⇒ a = – 4 and b = 4 \ Required value of
Hence parabola is Y a2 + 3ab + 5b2
y2 – 4y + 4 = x sin2 θ + cos2 θ
⇒ (y – 2)2 = x =
(1/4, 0)
X sin2 θ + 3 sin θ ⋅ cos θ + 5 cos2 θ
⇒ Y2 = X [Putting y – 2 = Y]
1 8 1 1 2
∴ Area = ⋅ × = ⇒
4 3 16 24 6 + 4 cos 2θ + 3 sin 2θ
1 For maximum value, denominator should be minimum
and directrix : x = − ⇒ 4 x + 1 = 0
4 \ 6 + 4 cos 2q + 3 sin 2q = 3 ± 5
2 2 ⇒ 6–5=1
7. (a, c, d) : Given V =
3 \ Maximum value of expression is 2.
^ ^ ^
i j k
1 1 2 2 9. (3) : log12/2 x + 4 log 2 x < 2 (4 − log16 x 4 )
Now ⋅ 0 1 1 h =
3 2 3
2 −1 −1 = log 22 x + 2 log 2 x < 2 (4 − log 2 x )

Put log2 x = t ⇒ t 2 + 2t < 2 (4 − t ) …(i)


⇒ t2 + 2t ≥ 0 ⇒ t(t + 2) ≥ 0
^ ^ ^
i j k ⇒ t ≤ –2 or t ≥ 0 …(ii)
⇒ h 0 1 1 =4 2 ⇒ 4–t≥0 ⇒t≤4 …(iii)
Squaring (i), we get
2 −1 −1
t2 + 2t < 2 (16 + t2 – 8t)
⇒ 0 < t2 – 18t + 32
1 ⇒ (t – 16) (t – 2) > 0
[Note ABC is a right triangle → Area = (2)( 2 ) = 2 ]
^ ^ ^ 2 ⇒ t < 2 or t > 16 …(iv)
h | i (−1 + 1) + 2( j − k )| = 4 2 Using (ii), (iii) & (iv) we get t ≤ –2 or 0 ≤ t < 2
^ ^ ⇒ log2 x ≤ –2 or 0 ≤ log2 x < 2
⇒ h| j − k|= 2 2 ⇒ h = 2
1
Let E divides AM in the ratio l : 1 ⇒ x≤ or 1 ≤ x < 4
4
 2λ + 1 1 1 
Hence E :  , ,  1
 λ + 1 λ + 1 λ + 1  ⇒ x ∈  0,  ∪ [1, 4 )
 4
Now, (AE)2 + (DE)2 = (AD)2 The integral value of x is 3.
2 2 2
 2λ + 1   1   1  10. (4) : Centre: Cn = 1 + (n – 1) 3 ; Cn = 3n – 2
⇒  − 1 + 1 − + 1− + 4 = 16
 λ +1   λ + 1   λ + 1  C5 = 13; C3 = 7; C5(13, 0); C3(7, 0)
2 2 2
Radius: Rn = arn–1 = 2n–1 \ R3 = 4
 λ   λ   λ  Line AB:
 λ + 1  + 2  λ + 1  = 12 ⇒  λ + 1  = 4 y – 0 = m (x – 13); mx – y – 13m = 0

48 MATHEMATICS TODAY | MAY‘18


mx – y – 13m = 0 = – 3 + 2·3 + 2·32 + ....... + 2 · 350 – 99·351

3 ⋅ (350 − 1)
C3 C5 = – 3 + 2· – 99·351 = – 3 + 351 – 3 – 99·351
(7, 0) (13, 0)
x 3 −1
= – 6 – 98 · 351 ⇒ S = 3 + 49·351
C \ a + b = 100
1
Hence (a + b) = 5
−6m 20
⇒ = 4 ⇒ 9m2 = 4m2 + 4
m +1 2 (13 – 15) : 13. (a) 14. (d) 15. (c)
2 4 2 −2 The correct set of combination from the above table is :
⇒ m = ⇒ m1 = and m2 =
5 5 5 (A) - (III) - (ii); (B) - (I) - (iv) ; (C) - (II) - (iii) ;
4 (D) - (IV) - (i)
Hence 5 | m1m2 | = 5 × = 4
5  1 − cos 2 x   1 + cos 2 x 
11. (2) : If a and b are the roots of x2 + t2x – 2t = 0, (A) Q y = c1   + c2  
 2   2
then we have a + b = – t2 and ab = – 2t.
+ c3 sin2x + c4 cos2x = A + B sin2x + C cos2x
α 2 + β2 (α + β)2 − 2αβ t2 1 dy
So = = + \ = 2B cos2x – 2C sin 2x ...(i)
(αβ)2 (αβ)2 4 t dx
1 1 1 1 d2 y
and + = − ⇒ = – 4B sin2x – 4C cos2x
(αβ)2 αβ
4t 2 2t dx 2
2 d3 y dy
 1  1  1  ⇒ = – 8B cos2x + 8C sin2x = – 4
3 dx
Now I = ∫  α2   β2  + αβ  dx
 x + x + dx
[From Eq. (i)]
−1 
2  d3 y dy
 t2 1  1 1  3t 2 3 ⇒ +4
=0 \ O = 3, D = 1
= ∫  x2 +  +  x +  −   dx = + +3 dx 3dx
  4 t  4t 2 2t   8 4t 2 c +c
−1 (B) Q y = (c1 + c2) sin (x + c3) – c4 e 5 6 . e x
dI 3t 3 or y = A sin (x + B) + Ce x ......(i)
Differentiating I w.r.t. 't', we get = − =0
dt 4 2t 3 dy
4 ∴ = A cos( x + B) + Ce x .....(ii)
So, we get t = ± 2 , and since t is taken to be dx
positive then Subtracting Eq. (i) from Eq. (ii), we get
3 2 18 9 a dy
I min = I ( 4 2 ) = +3= +3= +3= +c – y = A cos (x + B) – A sin (x + B) ....(iii)
4 16 8 b dx
\ Least value of (a + b + c)/10 is 2. d2 y dy
⇒ 2
− = – A sin (x + B) – A cos (x + B)
 3  4 −3 dx dx
1   1 0 
12. (5) : AB =  2   = =I d3 y d2 y
   −2 2  0 1 ⇒ −
1 2  dx 3 dx 2
= – A cos (x + B) + A sin (x + B)
\ (AB)1C1 = C1, (AB)2C2 = C2 and so on.  dy 
= –  − y [from Eq. (iii)]
Also, tr(Cr) = r·3r + (r – 1)·3r = (2r – 1)·3r  dx 
50 d 3 y d 2 y dy
⇒ − + − y = 0 \ O = 3, D = 1
Now, ∑ tr((AB)r Cr ) = tr((AB)1C1 ) + tr((AB)2 C2 ) dx 3 dx 2 dx
r =1
+ ........ + tr (( AB)50 C50 ) = S (Let) (C) Put x = tanq, y = tanf
\ S = tr (C1) + tr (C2) + .... + tr (C50) Then, (secq + secf) = c1 (tanq secf + tanf secq)
S = 1·31 + 3·32 + 5·33 + ...... + 99·350  cos θ + cos φ   sin θ + sin φ 
⇒   = c1 
3S =1·32 + 3·33 + ....... + 97·350 + 99·351  cos θ cos φ   cos θ cos φ 
θ+φ θ− φ
⇒ 2 cos  cos 
– 2S = 1 · 3 + 2·32 + 2·33 + ...... + 2·350 – 99·351  2    2 

MATHEMATICS TODAY | MAY‘18 49


θ+φ θ−φ  4 9 5
= c1.2 sin  cos  we get Q =  − , 
 2   2   7 7 
θ+φ θ+φ \ Equation of second reflected ray is
⇒ cot  =c ⇒ = cot −1 c1
 2  1 2 9 5
+ 5
1 1dy y+ 5= 7 (x − 0) (pass through Q, S ′)
or + =0 \ O = 1, D = 1 4
2
(1 + x ) (1 + y ) dx 2 − −0
7
(D) y = c1e3x + c2e5x or c1e3x + c2e5x – y = 0 ...(i) or 4 5x + y + 5 = 0
dy (B) 4x2 + 9y2 = 36
\ = 3c1e3x + 5c2e5x
dx x2 y2
dy ...(ii) or + =1
or 3c1e3x + 5c2e5x – =0 9 4
dx
Again differentiating both sides w.r.t. x, i.e., ellipse along y
x-axis.
d2 y
then 9c1e3x + 25c2e5x – =0 ...(iii) If e be the
dx 2 eccentricity, then
Q

From Eqs. (i), (ii) and (iii) by eliminating c1, c2, c3, 4 = 9(1 – e2) x S S
(–3,0) (– 5, 0) ( 5, 0) (3,0)
x
we get
5
1 1 y or e =
3 P(0, –2)
dy Then,
3 5 =0
dx foci = (± 5 , 0) y
d2 y and P = (0, – 2)
x y
9 25 \ Equation of first reflected ray PQ is + =1
dx 2 − 5 −2
Expanding w.r.t. R1, then or 2 x + y 5 + 2 5 = 0 ...(ii)
 d2 y dy   d 2 y dy 
5 − 25 − 13 − 9  + y(75 − 45) = 0 Solving (ii) and 4x2 + 9y2 = 36,
 
 dx 2 dx   dx 2 dx   9 5 4
we get Q ≡  − , 
d2 y dy d2 y dy  7 7
⇒ 2 − 16 + 30 y = 0 ∴ − 8 + 15 y = 0 \ Equation of the second reflected ray is
2 dx 2 dx
dx dx
4
\ O = 2, D = 1 −0
y −0= 7 (x − 5 ) (Pass through Q, S)
(16 - 18) : 16. (b) 17. (b) 18. (d)
The correct set of combination from the above table is : −9 5
− 5
(A) - (III) - (ii); (B) - (I) - (iii); (C) - (II) - (i) 7
1
x2 y2 ⇒ y=− (x − 5 ) or x + 4 5 y − 5 = 0
(A) Given 9x2 + 4y2 = 36 or + =1 4 5
4 9
i.e., ellipse along y-axis. If e be the eccentricity, then x2 y2
4 = 9(1 – e2) (C) Q Given 4x2 + 5y2 = 100 or + =1
y 25 20
5 (0, 3)
⇒ e= i.e., ellipse along x-axis. y
Q
3 If e be the
(0, 5) P(0, 2 5 )
Then, foci = (0, ± 5 ) S eccentricity then
and P ≡ (2, 0) P 20 = 25 (1 – e2)
,0)

x x (–5, 0) ( 5, 0)
5

C (2, 0)
\ Equation of first (–2, 0) 4 C
(–

= 1 − e2
x
⇒ x S S (5, 0)
5)

reflected ray PQ is S 5
,–

Q
(0

x 5 + 2 y − 2 5 = 0 ...(i) 1 P(0, –2)


(0, -3) ∴ e=
Solving (i) and 9x2 + 4y2 = 36, y 5 y

50 MATHEMATICS TODAY | MAY‘18


Then, foci (± 5 , 0) and P ≡ (0, 2 5 ) ⇒ (b + c)2 – a2 = 2bc ⇒ b2 + c2 = a2
⇒ ∠A = 90° ⇒ ∠B = 30°
\ Equation of first reflected ray PQ is
x y 3. (c) : The director circle of given hyperbola
+ = 1 or 2x + y – 2 5 = 0 ...(iii)
5 2 5 x2 y2
− = 1 , is x2 + y2 = 5 and given point M(1, 2)
Now, solving (iii) and 4x2 + 5y2 = 100 12 7
5 5 4 5 lies on it.
we get Q ≡  ,− π
 3 3  ⇒ The angle between pair of tangents = .
\ Equation of the second reflected ray is 2
1 1 π
−4 5 As tan −1 + tan −1 = tan −1 1 =
−0 3 2 4
y −0= 3 (x + 5 ) (pass through Q, S)  1  1  π
5 5 ⇒ 2  tan −1   + tan −1    =
+ 5  3  2  2
3
4. (d) : Vector perpendicular to 2x – y – z = 4 is
1  ^ ^ ^
⇒ y = − (x + 5 ) n = 2i − j − k
2 
Also line is collinear with n .
or x + 2 y + 5 = 0 ∴ S : x + 2 y + 5 = 0
So, equation of line is
PAPER-2  ^ ^ ^ ...(1)
r = λ (2 i − j − k )
1. (c) : We have N = a b c d Now line (1) meet the plane (0, 0, 0)

First place a can be filled in 2 ways i.e. 4, 5  ^ ^ ^ n
r ⋅ (3 i − 5 j + 2 k) = 6
(4000 ≤ N < 6000) So, l(6 + 5 – 2) = 6 2x – y – z = 4
For b and c, total possibilities are '6' (3 ≤ b < c ≤ 6) 2
i.e. 34, 35, 36, 45, 46, 56 ⇒ λ=
Last place d can be filled in 2 ways i.e. 0, 5 (N is a 3
Hence P.V. of the point 'A' 3x – 5y + 2z = 6
multiple of 5) A
Hence total numbers = 2 × 6 × 2 = 24 2 ^ ^ ^
is (2 i − j − k)
2. (b) : We have ra + rb = 3R 3
∆ ∆ 3abc  abc  5. (d) : S = 2C0 + 3C1 + 4C2 + 5C3 + ......... + 99C97
⇒ + = 3R = QR = ⇒ S = 3 C0 + 3C1 + 4C2 + 5C3 + .......... + 99C97
s −a s −b 4∆  4 ∆  
∆(s − b + s − a) 3abc
⇒ = = 4 C1 + 4C2 + 5C3 + .......... + 99C97 = 5C2 + 5C3 +...........
(s − a)(s − b) 4∆  
c∆ 3abc ⇒ ∆2 3ab 100 ⋅ 99 ⋅ 98
⇒ = = \ S = 100C97 ⇒ S = = 161700
(s − a)(s − b) 4 ∆ (s − a)(s − b) 4 3 ⋅2
⇒ 4s(s – c) = 3ab ⇒ (a + b + c)(a + b – c) = 3ab 6. (d) : For the equation x2 – px + 1 = 0,
⇒ (a + b)2 – c2 = 3ab ⇒ a2 + b2 – c2 = ab the product of roots, ab2 = 1
and for the equation x2 – qx + 8 = 0,
⇒ c2 = a2 + b2 – ab
the product of roots a2b = 8
⇒ a2 + b2 – 2ab cos C = a2 + b2 – ab
Hence, (ab2) (a2b) = 8 ⇒ a3 b3 = 8 ⇒ ab = 2
(Q c2 = a2 + b2 – 2ab cos C)
1
1 \ From ab2 = 1, we have b = and from a2·b = 8,
⇒ cosC = ⇒ ∠C = 60° ...(1) 2
2 we have a = 4
Similarly from rb + rc = 2R
∆(2s − b − c) 2abc 1 17
∆ ∆ Now, p = a + b2 = 4 + =
⇒ + = 2R ⇒ = 4 4
s −b s −c (s − b)(s − c) 4 ∆ 1 33
and q = α2 + β = 16 + =
2∆ 2 2 2
⇒ = bc ⇒ 2s(s – a) = bc r
(s − b)(s − c) Q is arithmetic mean of p and q
⇒ (b + c + a)(b + c – a) = 2bc 8

MATHEMATICS TODAY | MAY‘18 51


r p+q  17 33  2010
{x + k} {x + 1} {x + 2}
∴ = ⇒ r = 4( p + q) = 4  +  = 17 + 66 = 83
8 2 4 2 (d) We have ∑ 2010
=
2010
+
2010
+ ......
k =1
7. (b) : We have ax3 + bx2 + cx + d = 0 {x + 2010} 2010 {x}
+ = = {x }
ax 4 bx 3 cx 2 2010 2010
Let f (x ) = + + + dx + e \ f(0) = e
4 3 2 10. (a, c, d) : I1 = ln(1 + 3 )
8b π
f(2) = 4a + + 2c + 2d + e I2 =
3 3
(12a + 8b + 6c + 6d )  
2 1  1
= + e = (6a + 4b + 3c + 3d ) + e = 0 3
3 3  dx dx 
I0 = Lim In = Lim  ∫ + ∫
n ∫
= dx = 1.
⇒ f(2) = e n→ ∞ n → ∞ 1 + xn 1 + x
\ By Rolle’s theorem, there exist atleast one value of 0 1  0
 zero 
x ∈ (0, 2) such that f ′(x) = 0  
⇒ The equation ax3 + bx2 + cx + d = 0 has atleast Hence I0 = 1. Now verify all alternatives.
y
one real root in [0, 2] 11. (c, d) : Any tangent to the (4,4)
2 (2,3)
8. (c, d) : We have f ′(x) = 5 sin4 x cos x – 5 cos4 x sinx parabola y = 4x, is
= 5 sinx cosx(sinx – cosx)(1 + sinx cosx) yt = x + t2 at (t2 , 2t)
(1,2)
π π If it passes through the x
\ f ′(x) = 0 at x = . Also f (0) = f   = 0 centre (2, 3) of the circle,
O
4 2
then t2 – 3t + 2 = 0 ⇒ t = 1, 2
 π
Hence $ some c ∈  0,  for which f ′(c) = 0 \ The point can be (1, 2) or (4, 4)
 2
(By Rolle's Theorem) (one circle will be of radius 2 and other will be of
 π π π radius 5 )
Also in  0,  f is decreasing and in  ,  f is
 4 4 2 12. (a, b, c, d) :
increasing −π π
π Q ≤ sin −1 x ≤ ∀ − 1 ≤ x ≤ 1
⇒ minimum at x = 2 2
4 3 π2
π
As f(0) = f   = 0 ⇒ 2 roots ∴ 0 ≤ (sin−1 x )2 + (sin −1 y )2 + (sin−1 z )2 ≤
2 4
2
−1 2 3π is possible
9. (a, d) : (a) We have, F (x ) = (1 − x 6/5 )
−1 2 −1 2
∴ (sin x ) + (sin y ) + (sin z ) =
4
1 if x, y, z ∈ {–1, 1}
Now, F ′(x ) = −6 x 5 exist ∀ x ∈(−1, 1) \ Possible values of x – y + z from the ordered triplet
5
(x, y, z) are as follows
Also, F(– 1) = 0 = F(1)
Hence Rolle's theorem is applicable to the function F(x). (x, y, z) x–y+z (x, y, z) x–y+z
(b) For domain of F(x), (–1, –1, –1) –1 (1, 1, 1) 1
5 – [x] + 1 – [x]2 > 0 and x2 + x – 2 ≠ 0 (–1, 1, 1) –1 (1, – 1, –1) 1
⇒ (x + 2) (x –1) ≠ 0 ⇒ x ≠ – 2, 1 (1, –1, 1) 3 (–1, 1, –1) –3
Now [x]2 + [x] – 6 < 0 ⇒ ([x] + 3) ([x] – 2) < 0 (1, 1, –1) –1 (–1, –1, 1 1
⇒ –3 < [x] < 2 ⇒ – 2 ≤ x < 2
\ Domain = (–2, 1) ∪ (1, 2) \ Set of values of x – y + z is {±1, ± 3)
1 13. (a, c, d) : Centre (3, 3) with radius = 3
(c) We have, F(q) = a sinq + sin 3q
3 and centre (–3, –3) with radius = 3.
π
As F(q) has an extremum at q = , so Y
3
π a
⇒ a cosq + cos 3q = 0 at q = ⇒ −1= 0
3 2 O X
a
⇒ −1 ⇒ a = 2
2

52 MATHEMATICS TODAY | MAY‘18


MATHEMATICS TODAY | MAY‘18 53
14. (b, d) : Equation of the line passing through Let x – 2 = t, then
P(1, 4, 3) and direction ratio's a, b & c is (t − 2)4 t 4 − 8t 3 + 24t 2 − 32t + 16
t∫ dt = ∫ dt
x −1 y − 4 z − 3 t4 t4
= = ...(1) ...(i)
a b c  8 24 32 16 
= ∫ 1 − + − +  dt
x −1 y + 3 z − 2  t t2 t3 t 4 
Since (i) is perpendicular to = =
2 1 4 24 16 16
x +2 y − 4 z +1 = t− 8 ln | t | − + − +C
and = =
3 2 −2 t t 2 3t 3
Hence 2a + b + 4c = 0 and 3a + 2b – 2c = 0 24 16 16
= x − 8 ln | x − 2 | − + − +C
a b c a b c x − 2 (x − 2)2
3(x − 2)3
∴ = = ⇒ = =
−2 − 8 12 + 4 4 − 3 −10 16 1 2β
Hence the equation of the line is x δ +1 − 5x γ +1 + 2β | x | +1
(ii) ∫ x 2 + 4β | x | +1
dx
x −1 y − 4 z − 3 ...(ii) 2α
= =
−10 16 1 (putting a, b, g and d)
1 5 3
Now any point Q on (2) can be taken as (1 – 10l, 16l x − 5x + | x | +1
= ∫ (Q x 5 − 5x 3 are odd function.)
+ 4, l + 3) 2
x + 2 | x | +1
−1
\ Distance of Q from P(1, 4, 3) = (10l)2 + (16l)2 + l2
1 1 1
= 357 | x | +1 dx dx
2
= ∫ 2
= 2∫
(| x | +1)
= 2∫
1+ x
⇒ (100 + 256 + 1)l = 357 −1 (| x | +1) 0 0
1
⇒ l = 1 or – 1 = 2[ln |(1 + x)|] = 2ln2
0
\ Q is (–9, 20, 4) or (11, – 12, 2)
(17 – 18) :17. (d) 18. (b)
Hence, a1 + a2 + a3 = 15 or 1
a + c  a
(15 – 16) : 15. (a) 16. (b) Given log  + log   = log 2
 a  b
Sum of all four roots = 6; Sum of two roots = 0
⇒ sum of other two roots = 6 a +c 
⇒ log  = log 2 ⇒ a + c = 2b ...(i)
 b 
The factors corresponding to these roots are of the type 2 2
Also, a – ax + 2bx + c + cx = 0 has equal roots
x2 – 0x + p and x2 – 6x + q i.e., x2 + p and x2 – 6x + q ⇒ (c – a)x2 + 2bx + (c + a) = 0 has equal roots
Thus, we have 4x4 – 24x3 + 31x2 + 6x – 8 ≡ 4(x2 + p) \ D = 0 ⇒ 4b2 – 4(c2 – a2) = 0
(x2 – 6x + q) \ b2 = c2 – a2 ...(ii)
⇒ Triangle is right angled at C ⇒ ∠C = 90°
Equating like powers of x, we get
From (i) and (ii), b2 = (c – a)(c + a) = (c – a) 2b
31 ⇒ 2(c – a) = b ...(iii)
31 = 4(p + q) ⇒ p + q =
4 As ∠C = 90° ⇒ ∠A + ∠B = 90°
Also, 6 = 4(–6p) ⇒ p = − 1 \q=8 From (iii) using Sine law,
4 2(sinC – sinA) = sinB ...(iv)
\ Equation f(x) = 0 can be written as Now, C = 90° ⇒ sinC = 1
 2 1 2 1 Also, A + B = 90° ⇒ B = 90° – A
 x − 4  (x − 6 x + 8) = 0 ⇒ x = ± or x = 2, 4 \ (iv) becomes 2(1 – sin A) = sin(90 – A) = cosA
2
Squaring both sides, we get
1 1
∴ α = − , β = , γ = 2, δ = 4 4(1 – sinA)2 = cos2A = (1 – sin2 A)
2 2 ⇒ 4(1 – sinA) = (1 + sinA) ⇒ 3 = 5 sinA
α +β + δ
1 1
− + +4 ⇒ sinA = 3/5
x−δ x−4 2 2 Now, B = 90 – A
(i) Now ∫  dx = ∫  dx
 x − γ   x − 2  ⇒ sinB = cosA = 4/5 and sinC = 1
4 3 4 12
x−4 ∴ sin A + sin B + sin C = + + 1 =
=∫ dx 5 5 5
 x − 2  

54 MATHEMATICS TODAY | MAY‘18


10 BEST
PROBLE
MS

Math Archives, as the title itself suggests, is a collection of various challenging problems related to the topics of
JEE Main & Advanced Syllabus. This section is basically aimed at providing an extra insight and knowledge to the
candidates preparing for JEE Main & Advanced. In every issue of MT, challenging problems are offered with detailed
solution. The reader’s comments and suggestions regarding the problems and solutions offered are always welcome.

1. If 1/6sinq, cosq, tanq are in G.P., then q is 6. The left hand derivative of f(x) = [x] sin(px) at
π π x = k, k is an integer is
(a) 2nπ ± , n ∈ I (b) 2nπ ± , n ∈ I
3 6 (a) (–1)k (k – 1)p (b) (–1)k–1 (k – 1)p
(c) (–1) kpk (d) (–1)k–1 k
nπ π
(c) nπ ± (−1) , n ∈ I (d) nπ ± , n ∈ I
3 3 7. A = {f, {f}} then power set of A is
2. P is a point on the line y + 2x = 1 and Q and R are (a) A (b) {f, {f}, A}
two points on the line 3y + 6x = 6 such that triangle (c) {f, {f}, {{f}}} (d) {f, {f}, {{f}}, A}
PQR is an equilateral triangle. The length of the side of 8. The rate of change of the volume of a sphere w.r.t
the triangle is its surface area, when the radius is 2 cm, is
2 3 4 1 (a) 1 (b) 2 (c) 3 (d) 4
(a) (b) (c) (d)
15 5 5 5 9. A tower stands at the centre of a circular park.
3. The equation of a circle which has normals (x – 1) ‘A’ and ‘B’ are two points on the boundary of the park
(y – 2) = 0 and a tangent 3x + 4y = 6 is such that AB (= a) subtends an angle 60° at the foot of
(a) x2 + y2 – 2x – 4y + 4 = 0 the tower, and angle of elevation of the top of the tower
(b) x2 + y2 – 2x – 4y + 5 = 0 from A and B is 30°. The height of the tower is
(c) x2 + y2 = 5 (d) (x – 3)2 + (y – 4)2 = 5 2a a
(a) (b) 2a 3 (c) (d) a 3
4. The mean of n items is X . If the first term, second 3 3
term, third term …. is increased by 1, 2, 3… and so on. sin8 x − cos8 x
10. ∫ dx is equal to
Then new mean is 1 − 2 sin2 x cos2 x
n 1
n +1 (a) sin2x + c (b) − sin 2 x + c
(a) X + (b) X + 2 2
2 1
n −1 (c) sin 2 x + c (d) –sin2x + c
(c) X + n (d) X + 2
2
5. Let u(x) and v(x) be differentiable functions such SOLUTIONS
u(x ) u ′( x )  u(x )  ′ p+q 1
that = 7. If = p and   = q, then 1.(a) : cos2 θ = sin θ tan θ
v(x ) v ′( x )  v( x )  p−q 6
has the value equal to (u′ is equal to derivative of ‘u’) ⇒ 6cos3q = 1 – cos2q
(a) 1 (b) 0 (c) 7 (d) –7 ⇒ 6cos3q + cos2q – 1 = 0
By : Prof. Shyam Bhushan, Director, Narayana IIT Academy, Jamshedpur. Mob. : 09334870021

MATHEMATICS TODAY | MAY‘18 55


⇒ (2cosq – 1)(3cos2q + 2cosq + 1) = 0 9. (c) :
1 π
⇒ cos θ = ⇒ θ = 2nπ ± , n ∈ I
2 3
2. (a) : P
y + 2x = 1

1/ 5
3y + 6x = 6
Q R
The given lines are y +2x = 1 and y + 2x = 2
2 −1 1 CD = height of tower = h.
The distance between the lines = =
5 5 AC = BC = AB = a = r
1 2 h a
Side length of the triangle = cosec 60° = tan 30° = ⇒ h=
5 15 a 3
3. (a) : The two normal’s are x =1 and y = 2 (sin4 x + cos 4 x )(sin4 x − cos 4 x )
Their point of intersection, ( 1,2) is the centre of the 10 (b) : ∫ (sin2 x + cos2 x )2 − 2 sin2 x cos2 x
dx
3+8−6
required circle radius =1 = ∫ (sin4 x − cos 4 x )dx
5
Therefore, required circle is (x – 1)2 + (y – 2)2 = 1 sin 2 x
i.e., x2 + y2 – 2x – 4y + 4 = 0 = − ∫ (cos2 x − sin2 x )dx = ∫ − cos 2 x dx = − +c
2
4 (a) : a1 + 1, a2 + 2, a3 + 3, ....., an + n  
(a + a + .... + an ) + (1 + 2 + ..... + n)
New mean = 1 2
n DELHI at
1 + 2 + 3 + ...... + n n(n + 1)
=X+ =X+ • Satija Book Depot - Kalu Sarai Ph: 27941152; Mob: 9868049598
n 2n • Lov Dev & Sons - Kalu Sarai Ph: 43215656, 43215650; Mob: 9811182352
5. (a) : u(x) = 7v(x) ⇒ u′(x) = 7v′(x) ⇒ p = 7 (given) • Mittal Books - Darya Ganj Ph: 011-23288887; Mob: 9899037390
u(x ) u(x ) • Janta Book Depot Pvt Ltd. - Daryaganj
Again =7⇒ =0 Ph: 23362985, 23369685; Mob: 9350257596
v(x ) v(x ) • Mittal Book Distributors - Daryaganj Mob: 9811468898, 9810565956
p+q 7+0 • R D Chawla & Sons - Daryaganj Ph: 23282360/61; Mob: 9810045752 / 50 / 56
⇒ q = 0. Now, = =1
p−q 7−0 • Ms Enterprises - Dwarka Mob: 9810257310
• Yours Books & Stationers - Dwarka Mob: 9810676100
6. (a) : If x is just less than k, then [x] = k – 1 • Naval Book Depot - Jasola Ph: 011-26175789 , 26102425
\ f(x) = (k – 1) sinpx • Raaj Book Point - Karkardooma Ph: 011-22371098, 22375017; Mob: 9811021981
(k − 1) sin πx − k sin πk • Budaniya Book Shop - Mayur Vihar Ph: 22759059; Mob: 9958129735
LHD of f (x ) = lim • Anand Book Corner - Mayur Vihar
x →k x−k Ph: 22751946, 47; Mob: 9868102255, 9868082201
(k − 1) sin πx • New Arihant Book Depot - Patparganj
= lim , where x = k − h Ph; 26524221, 65726582; Mob: 9811522520
x →k x−k
(k − 1) sin π(k − h) • Schoolkart Technologies Pvt. Ltd. - Patparganj Mob: 8800654410
= lim = (k – 1)(–1)kp • Budaniya Book Shop - Prashant Vihar
h→0 −h Ph: 47631039; Mob: 9910749282, 9212519280
7. (d) : Conceptual. • Kashyap Brothers - Punjabi Bagh Ph: 65196167; Mob: 9811744071/ 9212144072
• Lamba Book Depot - Tilak Nagar Mob: 9810156674, 7503222162, 9210575859
4 3
8. (a) : V = πr , S = 4 πr 2 • Raj Book Agency - Tilak Nagar Ph: 64534374; Mob: 9811234161
3 • Janta The Bookshop - Vikas Puri Ph: 24604413; Mob: 9311167890

dV dS dV dV / dr 4 πr 2 r • Mishra Book Depot - Wazir Nagar Ph: 26864637; Mob: 9313799595, 9818779375
= 4 πr 2 , = 8 πr ⇒ = = =
dr dr dS dS / dr 8 πr 2
dV 2
When r = 2, = =1
dS 2

56 MATHEMATICS TODAY | MAY‘18


ISI
Exam on
13th May
2018

MOCK TEST
PAPER
Indian Statistical Institute
 nπ  8. Let a ≥ 2 be a real number. Let a, b be roots of the
1. Consider an = sin   . Establish the limiting equation x2 – ax + 1 = 0 and Let Tn = an + bn, n = 1, 2, ....
 2 

behaviour of each of the following sequences.  Tn 
(i) (bn)n ≥ 1 with bn = n·an (i) Prove that the sequence   is decreasing.
T
 n + 1 n = 1
an
(ii) (cn) with cn = , n ≥1 T T T
n (ii) Find all 'a' such that 1 + 2 +.... + n > n − 1, for
 nπ  T2 T3 Tn+1
(iii) (dn) with dn = an cos   any n = 1, 2, ......
 2 
9. Let M be the centroid of DABC with ∠AMB = 2∠ACB.
2. Consider the function Prove that
α + x sin2 x , x ≤ 0 (i) AB4 = AC4 + BC4 – AC2·BC2
f (x ) =  (ii) ∠ACB ≥ 60°
−1
β[sin x + cos x] , x > 0
10. If a1, a2, ...... an are real numbers, show that
(i) Find the value of a, b ∈ R, if any, for which f has n n
a primitive function in [–p/2, p/3]. ∑ ∑ ij cos(ai − a j ) ≥ 0
(ii) Find the value of a, b ∈ R, if any, for which f is i =1 j =1
integrable in [–p/2, p/3]. ∞ n
Cr
3. What is the remainder when the given sum S is
11. Evaluate ∑ 2n−1
Cr
r =0
divided by 4 : S = 15 + 25 + 35 + ..... + 1005 12. Let (an) be the last digit of the number 11 + 22 + 33 + nn.
4. Let F be the midpoint of side BC of DABC. Prove that the sequence (an)n is periodic with period 100.
Construct isosceles right triangles ABD and ACE 13. The number 1, 2, ..... 5n are divided into two
externally on sides AB and AC respectively with 90° at disjoint sets. Prove that these sets contain atleast n pairs
D and E. Show that DDEF is an isosceles right triangle. (x, y), (x > y), such that the number x – y is also an
5. Find all real solutions of the equation element of the set which contains the pair.

x 2 − t + 2 x 2 − 1 = x for each real value of t. 14. Find the locus of point P in the plane of square
1
6. If C0, C1, C2, ...... Cn are real numbers satisfying ABCD such that max. {PA, PC} = (PB + PD).
2
C0 + C1 + C2 + ......+ Cn = 0. Prove that
15. Let M be a point in the interior of DABC. Lines
C0 + 2C1 + 3C2n2 + (n + 1)Cnxn = 0 has atleast one
AM, BM, CM intersect the sides BC, CA, AB at points
real root.
6N − 1 A1, B1, C1, respectively. Denote the D1, D2, D3, D4, D5, D6
 r r 
7. Find ∑ min.  , 
respectively the areas of triangles MA1B, MA1C, MB1C,
r =1
 3N 6N  ∆ ∆ ∆
MB1A, MC1A, MC1B. Prove that if 1 + 3 + 5 = 3
where <x> denotes min. (x – [x], [x] – x + 1), [·] ∆2 ∆ 4 ∆6
denotes G.I.F. then M is the centroid of DABC
By : Tapas Kr. Yogi, Visakhapatnam Mob : 09533632105

MATHEMATICS TODAY | MAY‘18 57


SOLUTIONS 6. Let f(x) = C0 + 2C1x + 3C2x2 + .... + (n + 1)Cnxn
1
1. (i) Consider the subsequence of bn corresponding
to odd integral, i.e. b2n + 1 = (–1)n (2n + 1). Then ∫ f (x )dx = C0 + C1 + C2 + ..... + Cn = 0
0
This does not have a limit. Hence bn also has no limit.
(ii) cn sequence is product of a bounded sequence (an) So, by MVT there exists a l on (0 < l < 1) such that
1 f(l) = 0. i.e. f(x) = 0 has atleast one real root.
with sequence   which tends to 0. So, lim cn = 0.
n n ≥1 n→∞ 3N
7. Middle term of the sequence is =0
 nπ  1 3N
(iii) dn = an cos   = sin(nπ) = 0 for all n ∈ N .
2 2 6N − r r 6N − r r
and = and =
So, (dn) is actually constant and converges to 0. 3N 3N 6N 6N
2. (i) The function f is continuous in [–p/2, p/3]– {0}. 3N − 1
 r r 
Further f(x)→ b as x → 0+ and f(x) → a as x → 0–. Thus So, required sum = 2 ∑ min. 
 3N
,
6

N 
if a = b then f is continuous in [–p/2, p/3] and in this r =1

interval has a primitive function. If a ≠ b, then f has  2N 3N − 1 


r r
a jump discontinuity at x = 0 and so, does not admit =2 ∑
 + ∑ 
r = 1 6N 2n + 1
3N 
primitive function in [–p/2, p/3].  
(ii) f is continuous in [–p/2, p/3] – {0} and bounded,  2N r N −1
(N − 1)N 
hence integrable in [–p/2, p/3] for every a and b. = 2 ∑ + ∑ =N
r = 1 6N r = 1 2 
3. Notice that 25 + 45 + ... + 1005 ≡ 0 (mod 4) ...(1)  
and 15 ≡ 1 (mod 4), 35 ≡ –1 (mod 4), 55 ≡ 1 (mod 4),
8. (i) Given, a ≥ 2, ab = 1, we have
75 ≡ –1 (mod 4).
So, 15 + 35 + 55 + .... + 995 = [1 + (–1) + 1 – 1 + ....] n−1
Tn − 1 Tn (αβ) (α − β)2
(mod 4) = 0 (mod 4) (50 terms) ...(2) − = ≥0
5 5 5 5 5 5
Tn Tn + 1 (αn + βn )(αn+1 + βn+1 )
Hence, 1 + 2 + 3 + 4 + .... + 99 + 100 = 0 (mod 4)
4. Using complex nos. E Tn − 1Tn  T 
O
A So, ≥ ⇒  n  seq. is decreasing.
b+c Tn Tn + 1 T
F≡ D  n + 1 
2 (ii) From the above relation, we have
d = b – ba, e = ca B C T T T T
n ⋅ 1 ≥ 1 + 2 + ..... + n > n − 1
F (c)
1+i (b)
T2 T2 T3 Tn − 1
where α = then d – f = i(e – f)
2
⇒ DDEF is right isosceles. T1 1 T
i.e., > 1 − As n → ∞, 1 ≥ 1
5. After simplifying, we have T2 n T2
16(x2 – t)(x2 – 1) = (4x2 – t – 4)2 a
So, ≥ 1 i.e., a ≤ 2 Q a ≤ 2 \ a = 2.
|t − 4 | 2
...(i) a −2
i.e. =x
16 − 8t x 2 + y 2 − c2
For x to exist, 16 – 8t > 0 i.e., t < 2 9. (i) In DAMB, cos 2C = ...(i)
2 xy
Putting (i) in place of x in equation, we have on
1 C
simplification |3t – 4| + 2 |t| = 4 – t and Ar .(∆AMB) = Ar .(∆ABC )
3
4 b M a
If t ≥ then equation becomes 6t = 8 ⇒ t = 4/3. 1 1 x 2C y
3 ⇒ xy sin 2C = ∆
4 2 3 c
If 0 ≤ t ≤ the equation becomes identity and for 2∆
A B
3 ⇒ xy = ...(ii)
t ≤ 0, we have t = 0. Hence, solution exists iff t ∈[0, 4/3]. 3 sin 2C

58 MATHEMATICS TODAY | MAY‘18


2 (x, y), x > y with the desired property. Let l be a given
And x = × (length of median from A) number l = 1, 2, ... n. If l and 2l are in the same set A
3
or B then so is 2l – l = l. The same can be said about
2 1
= × 2b2 + 2c 2 − a2 ...(iii) 2l or 4l. Consider the case when l and 4l are elements
3 2 of A and 2l is an element of B. If 3l is an element of A
2 1 ...(iv)
and y = × ⋅ 2c 2 + 2a2 − b2 then 4l – 3l = l ∈ A. Now if 5l ∈A then 5l – 4l = l ∈ A
3 2 and if 5l ∈B then 5l – 3l = 2l ∈B.
Simplifying (i), (ii), (iii) and (iv), we have So, among the numbers l, 2l, 3l, 4l, 5l there is
c4 = a4 + b4 – a2 b2 atleast a pair with the required property. Because
(ii) It is clear that a4 + b4 – a2b2 ≥ (a2 + b2 – ab)2 l = 1, 2, 3, .... it follows that there is atleast n pairs
So, a2 + b2 – 2ab cosC = c2 ≥ a2 + b2 – ab with the required property.
1
⇒ cos C ≤ i.e., C ≥ 60° 14. Let l be the side length of square ABCD. Assuming,
2 max. {PA, PC} = PA; we have, P
10. The given summation is equivalent to
2PA = PB + PD D C
n n
∑ ∑ (ij cos ai cos a j + ij sin ai sin a j ) l ⋅ 2PA = l ⋅ PB + l ⋅ PD
i =1 j =1 A B
n n n n So, BD·PA = AD·PB AB·PD
= ∑ i cos ai ∑ j cos a j + ∑ i sin ai ∑ j sin a j Hence from converse of ptolemy's theorem, if follows
that PDAB is cyclic quad. i.e., locus of P is circumcircle
i =1 j =1 i =1 j =1
of square ABCD.
= (∑ i cos ai ) + (∑ i sin ai )
2 2
≥0
15. Using, Ceva's theorem, we have
n n n
Cr + 1  C1 A A1B B1C
Cr  Cr  ⋅ ⋅ =1
11. Notice that =2 − C1B A1C B1 A
2n − 1
Cr  2n C 2n
C 
 r r +1 
∆ 5 ∆1 ∆ 3
n
i.e., ⋅ ⋅ =1

Cr ∞  nC n
Cr +1  ∆6 ∆2 ∆ 4
So, ∑ =2∑  r − 
2n−1 2n 2n
r =0 Cr r = 0  Cr Cr +1  So, given condition becomes
n ∆1 ∆ 3 ∆ 5 ∆1 ∆ 3 ∆ 5
C0
=2× =2 + + = 3⋅ 3 ⋅ ⋅
2n
C0 ∆2 ∆ 4 ∆6 ∆2 ∆ 4 ∆6
12. Let us denote the last digit of a number by l(n). The i.e., A.M. = G.M. ⇒ Nos. are all equal.
sequence {l(n)} is clearly periodic of period 10. Also, ∆ ∆ ∆ CA AB B1C
for a fixed a (∈N), the sequence {l(an)} is periodic and i.e., 1 = 3 = 5 = 1 i.e., 1 = 1 = =1
∆2 ∆ 4 ∆6 C1B A1C B1 A
period is equal to 1 if 'a' ends in (0, 1, 5, 6), 2 if 'a' ends
in (4, 9) and 4 if 'a' ends in (2, 3, 7, 8). Since period is i.e., A1, B1, C1 are midpoints and hence M is the centroid
either 1, 2 or 4 or 10. Now L.C.M. of 10 and 4 is 20. So, of the triangle.
if we set m = (n + 1)n+1 + (n + 2)n +2 + ..... + (n + 20)n + 20 
then l(m) does not depend on n.
So last digit of 11 + 22 + 33 + ..... + 2020 = 1 + 4 + 7 +
6 + 5 + 6 + ........ ≡ 4 on simplification GOA at
So, last of given sum of form • Golden Heart Emporium - Goa
Ph: 0832-2725208, 3257383, 2730874; Mob: 8322725208, 9370273479
(n + 1) n+1 + (n + 2) n+2 + ..... + (n + 100) n+100 is
• Universal Traders - Goa Ph: 0832-2315985; Mob: 9404150150
l(4·5) = l(20) = 0. • Success Stationers - Margao Mob: 9850398314
Hence {dn} sequence has periodic with period = 100
13. Let's prove this by contradiction. Suppose these are
two sets A and B such that A ∪ B = {1, 2, 3, ..... 5n}.
A ∩ B = f and the sets contain together less than n pairs

MATHEMATICS TODAY | MAY‘18 59


Y U ASK 2. If a, b, c are in G.P. and the equation ax2 + 2bx + c = 0
and dx2 + 2ex + f = 0 have a common root, then
show that d , e , f are in A.P.

WE ANSWER
a b c
(Priyansh Sharma, Delhi)
Ans. We have, b2 = ac ...(i)
Do you have a question that you just can’t get answered? The equation ax2 + 2bx + c = 0
Use the vast expertise of our MTG team to get to the bottom −2b ± 4b2 − 4ac −b
gives x = = [using (i)]
of the question. From the serious to the silly, the controversial 2a a
to the trivial, the team will tackle the questions, easy and tough. −e ± e 2 − df
and dx2 + 2ex + f = 0 ⇒ x=
The best questions and their solutions will be printed in this d
column each month. According to the given condition, we have
2
−b −e ± e − df e b
2
e 2 − df
1. There are n intermediate stations on a railway line = i.e.  −  =
a d d a d2
from one terminus to another. In how many ways
2be b2 f c f
can the train stop at 3 of these intermediate stations i.e. = + = + [using (i)]
if ad a2 d a d
(i) all the three stations are consecutive? i.e. 2be = d + f  d
multiplying both sides by c 
(ii) at least two of the stations are consecutive? ac a c
(iii) no two of these stations are consecutive? i.e. 2e d f ⇒ d e f are in A.P.
= + , ,
(Ritu Jain, Gujarat) b a c a b c
Ans. 3. Find the values of a, b, c if the following limit exists :
A B
S1 S2 S3 Sr–1 Sr Sr+1 Sn–2 Sn–1 Sn ae x − b ln(1 + x ) + cxe − x
Lim =2
(i) The number of triplets of consecutive stations, viz. x →0 x 2 sin x
(Rajnish, Jharkhand)
S1S2S3, S2S3S4, S3S4S5, ..........., Sn–2Sn–1Sn is (n – 2).
(ii) The total number of consecutive pair of stations, ae x − b ln(1 + x ) + cxe − x
Ans. Let L = Lim
viz. x →0 x 2 sin x
−x
S1S2, S2S3, .........., Sn–1 Sn is (n – 1). ae x + cxe − b ln(1 + x ) x
= Lim ⋅
Each of the above pair can be associated with a third x →0 x3 sin x
station in (n – 2) ways. Thus, choosing a pair of stations x −x
ae + cxe − b ln(1 + x )
and any third station can be done in (n – 1)(n – 2) ways. = Lim
x →0 x3
The above count also includes the case of three    
x2 x2
consecutive stations. However, we can see that each a 1 + x + + ...  + cx 1 − x + − ... 
 2!   2! 
such case has been counted twice. For example, the pair
S4S5 combined with S6 and the pair S5S6 combined with  x2 x3 
−b  x − + + ... 
S4 are identical.  2 3 
= Lim 3
Hence, subtracting the excess counting, the number of x →0 x 2
ways in which three stations can be chosen so that at x  a c b
a + (a + c − b)x + (a − 2c + b) +  + −  x 3 + ...
least two of them are consecutive is 2  3! 2 ! 3 
= Lim
(n – 1)(n –2) – (n – 2) = (n – 2)2. x →0 x3
For a finite non-zero limit to exist, we have
(iii) Without restriction, the train can stop at any three
a=0 ...(i) a+c–b=0 ...(ii)
stations in nC3 ways. Hence, the number of ways the a – 2c + b = 0 ...(iii)
train can stop so that no two stations are consecutive is a c b
and + − = 2 [according to the given condition]
nC – (n – 2)2 = n(n − 1)(n − 2) – (n – 2)2 6 2 3
3 i.e. a + 3c – 2b = 12 ...(iv)
1⋅ 2 ⋅ 3
 n − n − 6n + 12  (n − 2)(n − 3)(n − 4)
2 Solving (i), (ii), (iii) and (iv), we can see that there is no
= (n – 2)   = = n–2C3. value of a, b, c which satisfies all the equations.
 6 6 

60 MATHEMATICS TODAY | MAY‘18


MATHEMATICS TODAY | MAY‘18 61
But f(0) = 0 ⇒ both function g1 and g2 have a value
zero at x = 0.
7. (d) : f1oF = f4 ⇒ F = f1–1 of4 = f1of4 ⇒ F = f4
SOLUTION SET-184 8. (c) : f3oJof2 = f4 ⇒ J = f3–1 of4of2–1 = f3of4of2
1. (c) : Q a, b, c are in H.P. \ b = 2ac/(a + c) ...(i)  1  1 
\ J (x ) = f3of 4   = f3
b, c, d are in G.P. \ c2 = bd ...(ii)  x  1
 1 − 
and c, d, e are in A.P. \ d = (c + e)/2 ...(iii) x
From (i), ab + bc = 2ac ⇒ c = ab/(2a–b) ...(iv)  x   x 
= f3   = 1 −  x − 1 = f 4 ( x )
1  ab   x − 1  
Also, d =  + e (Using (iii) and (iv)) ...(v)
2  2a − b 
 t − tt 
On putting the values of c and d from (iv) and (v) 9. (2) : Let sin x = t ⇒ L = lim  
in (ii), we get t → 1  1 − t + ln t 
t
1 − t (1 + ln t )
a 2 b2 b  ab  ab2 = lim
1
(By L.H. Rule)
=  + e ⇒ e = t →1
−1 +
(2a − b)2 2  2a − b  (2a − b)2 t
−t t (1 + ln t ) − t
2 t −1
2. (d) : Every number between 100 and 1000 −1 − 1
= lim = =2
is a 3-digit number. We first have to count the t →1 1 − 1
− 2
permutations of 6 digits taken 3 at a time. This number t
would be 6P3. But, these permutations will include
10. (b) : (P) → 3; (Q) → 1; (R) → 4; (S) → 2
those numbers also where 0 is at the 100th place.
^ ^ ^
To get the number of such numbers, we fix 0 at the 100th    i j k ^ ^ ^
place and rearrange the remaining 5 digits taking 2 at a P. Here, (c − a) × b = −5 −3 4 = 2 i + 14 j + 13 k
time. This number is 5P2.
6 ! 5! 6 1 −2
So, the required number = 6P3 − 5P2 = − = 100
3! 3! ^ ^ ^
| 2 i + 14 j + 13 k | 369
3. (d) : z = ki, k ∈R ⇒ –(cosa + isina)k2 + ki + 1 = 0 ∴ d= = =3
⇒ k2cosa = 1, ksina = 1. ^ ^ ^ 41
5 −1 5 +1 | 6 i + j− 2 k |
Eliminating k, we get cos α = , tan α = Q. z = w, w2
2 2
6 2
Put x = z 
2  r 1 
\ ∑  ω + r  = 1 + 1 + 4 + 1 + 1 + 4 = 12
2
4 2e sin x 16 e sin z
4. (c) : ∫ dx = ∫ dz  
⇒ 2dx = dz  r =1  ω 
1 x 1 z
 x  3 1 (2 x − 1 − 3)
 d R. = −
16 e sin x 
1+ x 3 x +1
2(x 2 − x + 1)
= ∫ d(F (z ))  F (x ) = , x > 0
1  dx x  1
3dx x +1 3 2
1
 2x −1 
1
16 ∴∫ = ln + × tan −1  
= [F (z )]1 = F (16) − F (1) \ F(16) = F(k) 3
0 1+ x x2 − x +1 0 2 3  3  0
Hence, one of the possible values of k = 16.
π π
5. (d) : Let g(x) = f(x) – x2 = ln 2 + 3 = ln 2 +
3 3
Using Rolle's Theorem, we get
g′(x) = 0 for at least one x ∈ (1, 2) S. t = sin 2x + 2cos2x ⇒ 1 – sin 2x + 2sin2x = 3 – t
27
Also, g′(x) = 0 for at least one x ∈ (2, 3) The equation becomes 3t + = 28 ⇒ 3t = 1, 27
\ g′′(x) = 0 for at least one x ∈ (1, 3) 3t
\ f ′′(x) = 2 for some x ∈ (1, 3) \ t = 0, 3. Now sin 2x + 2cos2x < 3
6. (b) : (f ′(x))2 – k2(f(x))2 ≤ 0 \ sin 2x + 2cos2x = 0 ⇒ sin 2x + cos 2x = –1
⇒ (f′(x) – kf(x))(f ′(x) + kf(x)) ≤ 0  π 3π
cos  2 x −  = cos
⇒ (f(x)e–kx)(f(x)ekx) ≤ 0  4 4
⇒ Exactly one of the functions g1(x) = f(x)e–kx or π 3π π 3π 3π 7 π
∴ 2 x − = 2nπ ± ⇒ x= , , ,
g2(x) = f(x)ekx is non decreasing. 4 4 2 2 4 4

62 MATHEMATICS TODAY | MAY‘18


Units
2018
VSA(1 mark) SA(2 marks) VBQ(4 marks) LA I(4 marks) LA II(6 marks) Total
Relations and Functions 1(1) --- --- --- 6(1)* 7(2)
Inverse Trigonometric Functions 1(1) 2(1) --- --- --- 3(2)
Matrices 1(1) --- --- --- --- 1(1)
Determinants --- 2(1) --- 4(1) 6(1)* 12(3)
Continuity and Differentiability --- 2(1) --- 8(2)* --- 10(3)
Application of Derivatives --- 2(1) 4(1) 4(1)* --- 10(3)
Integrals --- 2(1) --- 4(1) 6(1)* 12(3)
Application of Integrals --- --- --- --- 6(1) 6(1)
Differential Equations --- 2(1) --- 4(1)* --- 6(2)
Vector Algebra 1(1) 2(1) --- 4(1) --- 7(3)
Three Dimensional Geometry --- --- --- 4(1) 6(1) 10(2)
Linear Programming --- --- --- --- 6(1) 6(1)
Probability --- 2(1) --- 8(2) --- 10(3)
Total 4(4) 16(8) 4(1) 40(10) 36(6) 100(29)
* It is choice based question.
Time Allowed : 3 hours Maximum Marks : 100
GENERAL INSTRUCTIONS
(i) All questions are compulsory.
(ii) This question paper contains 29 questions.
(iii) Questions 1-4 in Section-A are very short-answer type questions carrying 1 mark each.
(iv) Questions 5-12 in Section-B are short-answer type questions carrying 2 marks each.
(v) Questions 13-23 in Section-C are long-answer-I type questions carrying 4 marks each.
(vi) Questions 24-29 in Section-D are long-answer-II type questions carrying 6 marks each.

SECTION - A 3. Find the


 magnitude of each of the two vectors

a and b , having the same magnitude such that the
1. Find the value of tan −1 3 − cot −1 (− 3 ) .
angle between them is 60° and their scalar product
0 a −3 9
is .
2. If the matrix A = 2 0 −1 is skew symmetric, 2
4. If a * b denotes the larger of ‘a’ and ‘b’ and if
b 1 0 
a o b = (a * b) + 3, then write the value of
find the values of ‘a’ and ‘b’. (5) o (10), where * and o are binary operations.

MATHEMATICS TODAY | MAY‘18 63


SECTION - B OR
 1 1 Find the intervals in which the function
5. Prove that : 3sin–1 x = sin–1(3x –4x3), x ∈  − ,  .
 2 2
x4
 2 −3 f (x ) = − x 3 − 5x 2 + 24 x + 12 is
6. Given A =  –1 4
−4 7  , compute A and show that
  (a) strictly increasing, (b) strictly decreasing
2A–1 = 9I – A.
17. An open tank with a square base and vertical sides
−1  1 + cos x  is to be constructed from a metal sheet so as to
7. Differentiate tan  with respect to x.
 sin x  hold a given quantity of water. Show that the cost
8. The total cost C(x) associated with the production of material will be least when depth of the tank is
of x units of an item is given by C(x) = 0.005x3 – half of its width. If the cost is to be borne by nearby
0.02x2 + 30x + 5000. Find the marginal cost when settled lower income families, for whom water will
3 units are produced, where by marginal cost we be provided, what kind of value is hidden in this
mean the instantaneous rate of change of total cost question?
at any level of output.
2 cos x
18. Find : ∫ dx
cos 2 x + 2 sin2 x (1 − sin x )(1 + sin2 x )
9. Evaluate : ∫ dx
cos2 x
19. Find the particular solution of the differential
10. Find the differential equation representing the
equation ex tan y dx + (2 – ex) sec2y dy = 0, given
family of curves y = aebx + 5, where a and b are
π
arbitrary constants. that y = when x = 0.
4
11. If q is the angle between two vectors i − 2 j + 3k and OR
3i − 2 j + k , find sin q. Find the particular solution of the differential
12. A black and a red die are rolled together. Find the dy
conditional probability of obtaining the sum 8, equation + 2 y tan x = sin x , given that y = 0 when
dx
given that the red die resulted in a number less than 4. π
x= .
SECTION - C 3
 
13. Using properties of determinants, prove that 20. Let a = 4i + 5j − k , b = i − 4 j + 5k and
 
1 1 1 + 3x c = 3i + j − k . Find a vector d which is
1+ 3y 1 1 = 9(3xyz + xy + yz + zx )    
perpendicular to both c and b and d ⋅ a = 21 .
1 1 + 3z 1
21. Find the shortest distance between the lines

14. If (x2 + y2)2 = xy, find
dy r = (4i − j) + λ (i + 2 j − 3k ) and
dx 
r = (i − j + 2k ) + µ (2i + 4 j − 5k ).
OR
dy 22. Suppose a girl throws a die. If she gets 1 or 2, she
If x = a(2q – sin 2q) and y = a(1 – cos2q), find tosses a coin three times and notes the number of
dx
π
when θ = . tails. If she gets 3, 4, 5 or 6, she tosses a coin once
3
and notes whether a ‘head’ or ‘tail’ is obtained. If she
15. If y = sin(sinx), prove that obtained exactly one ‘tail’, what is the probability
d2 y dy that she threw 3, 4, 5 or 6 with the die?
+ tan x + y cos2x = 0.
dx 2 dx 23. Two numbers are selected at random (without
16. Find the equations of the tangent and the replacement) from the first five positive integers. Let
normal, to the curve 16x2 + 9y2 = 145 at the point X denote the larger of the two numbers obtained.
(x1, y1), where x1 = 2 and y1 > 0. Find the mean and variance of X.

64 MATHEMATICS TODAY | MAY‘18


SECTION - D screws ‘A’ while it takes 6 minutes on the automatic
24. Let A = {x ∈ Z : 0 ≤ x ≤ 12}. Show that and 3 minutes on the hand-operated machine to
manufacture a packet of screws ‘B’. Each machine
R = {(a, b) : a, b ∈ A, |a – b| is divisible by 4} is
is available for at most 4 hours on any day. The
an equivalence relation. Find the set of all elements
manufacturer can sell a packet of screws ‘A’ at
related to 1. Also write the equivalence class [2].
a profit of 70 paise and screws ‘B’ at a profit of
OR ` 1. Assuming that he can sell all the screws be
Show that the function f : R → R defined by manufactures, how many packets of each type
x should the factory owner produce in a day in order
f (x ) = , ∀ x ∈ R is neither one-one nor onto.
x2 + 1 to maximize his profit? Formulate the above LPP
Also, if g : R → R is defined as g(x) = 2x – 1, find and solve it graphically and find the maximum
profit.
fog(x).
SOLUTIONS
2 −3 5 
 
25. If A = 3 2 −4  , find A–1. Use it to solve the 1. Since, tan −1 3 − cot −1( − 3 )
1 1 −2 
= tan −1 3 − ( π − cot −1 3 )
system of equations π π = π−π=−π
2x – 3y + 5z = 11 = tan −1 3 − π + cot −1 3 = −π+
3 6 2 2
3x + 2y – 4z = –5
2. A square matrix A is said to be skew symmetric
x + y – 2z = –3. matrix if A = –A′ ...(i)
OR 0 a −3  0 2 b
Using elementary row transformations, find the    
Now, A = 2 0 −1 ∴ A′ =  a 0 1
inverse of the matrix b 1 0 
1 2 3  −3 −1 0
  From (i), A + A′ = O
A= 2 5 7
 −2 −4 −5 0 a −3  0 2 b 
   
⇒ 2 0 −1 +  a 0 1 = O
26. Using integration, find the area of the region in the b 1 0   −3 −1 0
first quadrant enclosed by the x-axis, the line y = x
 0 2 + a b − 3
and the circle x2 + y2 = 32.  
⇒ a + 2 0 0  =O
π/ 4
sin x + cos x b − 3 0 0 
27. Evaluate : ∫ dx
0
16 + 9 sin 2 x
⇒ a + 2 = 0 & b –3 = 0
OR
\ a = –2 & b = 3
3    9
Evaluate : ∫ (x
2
+ 3x + e x ) dx , as the limit of the 3. We are given that, | a | = | b |, θ = 60° and a ⋅b =
2
sum. 1  
a ⋅b 9/2 1 9/2
Now, cos θ =   ⇒ cos 60° =  ⇒ =
28. Find the distance of the point (–1, –5, –10) | a || b | | a |2 2 | a |2
from the point of intersection of the line   

r = 2i − j + 2k + λ(3i + 4 j + 2k ) and the plane ⇒ | a |2 = 9 ⇒ | a | = 3 So, | b | = 3

r ⋅ (i − j + k ) = 5 . 4. (5) o (10) = (5 * 10) + 3
29. A factory manufactures two types of screws A and = larger of (5, 10) + 3 = 10 + 3 = 13
B, each type requiring the use of two machines, an 5. Put sin–1x = q. Then x = sinq
automatic and a hand-operated. It takes 4 minutes Now, sin3q = (3sinq – 4sin3q) = (3x – 4x3)
on the automatic and 6 minutes on the hand- ⇒ 3q = sin–1(3x – 4x3)
operated machines to manufacture a packet of ⇒ 3sin–1x = sin–1(3x – 4x3) [Q q = sin–1x]

MATHEMATICS TODAY | MAY‘18 65


2 −3 From (ii) & (iii), we have
6. | A | = = 14 – 12 = 2 ≠ 0 1 dy 1 d y
2
d2 y dy
−4 7 = ⇒ =b ...(iv)
So, A is a non-singular matrix and therefore, it is invertible. ab dx ab2 dx 2 dx 2 dx
 7 3 1 dy
From (i) and (ii), we have =b ...(v)
Adj ( A) =   y dx
 4 2 2
d 2 y 1  dy 
1 1  7 3 Solving (iv) and (v), we get =  
Hence, A−1 = Adj ( A) =   dx 2 y  dx 
| A| 2  4 2
 
 7 3 11. Let a = i − 2 j + 3k , b = 3i − 2 j + k
⇒ 2 A−1 =  ...(i)    

 4 2 Now, a ⋅ b = | a || b | cos θ
1 0  2 −3 ⇒ (i − 2 j + 3k ) ⋅ (3i − 2 j + k ) = (1)2 + (−2)2 + (3)2
Now, 9I – A = 9  − 
0 1  −4 7 
× (3)2 + (−2)2 + (1)2 cos θ
9 0  2 −3 7 3 –1
= − =  = 2A [From (i)] 10
0 9  −4 7   4 2 ⇒ 3 + 4 + 3 = 14 × 14 cosθ ⇒ cosθ =
14
 2 cos2 x 
−1  1 + cos x  −1 
2  100 96 2 6
7. Let y = tan   = tan  ∴ sin θ = 1 − cos2 θ = 1 − = =
sin x  x x 196 196 7
 2 sin cos 
2 2
12. E : ‘a total of 8’ and F : ‘red die resulted in a number
−1  x −1 π x π x
= tan  cot  = tan tan  −  = − less than 4’
 2 2 2 2 2
i.e., E = {(2, 6), (3, 5), (4, 4), (5, 3), (6, 2)}
dy d  π x  1 and F = {(x, y) : x ∈ {1, 2, 3, 4, 5, 6}, y ∈ {1, 2, 3}}
∴ =  −  = − .

dx dx 2 2 2 i.e., F = {(1, 1), (1, 2), (1, 3), (2, 1), (2, 2),
8. We have C(x) = 0.005x3 – 0.02x2 + 30x + 5000 ...(i) (2, 3), (3, 1), (3, 2), (3, 3), (4, 1), (4, 2), (4, 3), (5, 1),
(5, 2), (5, 3), (6, 1), (6, 2), (6, 3)}
Differentiating (i) w.r.t. x, we have
Hence, E ∩ F = {(5, 3), (6, 2)}, P(E) = 5/36,
dC(x ) d(0.005x 3 − 0.02 x 2 + 30 x + 5000) P(F) = 18/36, P(E ∩ F) = 2/36
MC = =
dx dx \ Required probability = P (E | F)
2
= 3 × 0.005x – 2 × (0.02)x + 30 P( E ∩ F ) 2 / 36 1 .
= = =
= 0.015x2 – 0.04x + 30 ...(i) P( F ) 18 / 36 9
dc
 
\   = 0.015(3)2 – 0.04(3) + 30 = 30.015 1 1 1 + 3x
 dx  x =3
Hence, required marginal cost when x = 3 is 30.015 13. L.H.S. = 1 + 3 y 1 1
1 1 + 3z 1
cos 2 x + 2 sin2 x
9. Let I = ∫ dx
cos2 x Applying C2 → C2 – C1, C3 → C3 – C1, we get
2 2 2 2 2
cos x − sin x + 2 sin x cos x + sin x 1 0 3x
=∫ dx = ∫ dx
2
cos x cos2 x 
L.H.S. = 1 + 3 y −3 y −3 y
1 1 3z 0
=∫ dx = ∫ sec2 x dx = tan x + C
cos2 x
Taking out 3 common from both C2 and C3, we get
10. We have, y = a ebx + 5 ...(i)
1 0 x
dy 1 dy
∴ = aebx +5 ⋅ b ⇒ ebx +5 = ...(ii) L.H.S. = 9 1 + 3 y − y − y
dx ab dx
Differentiating (ii) w.r.t. x, we get 1 z 0
2 2
1 d y 1 d y Expanding along R1, we get
b ⋅ ebx +5 = ⇒ ebx +5 = ...(iii)
ab dx 2 ab2 dx 2 L.H.S. = 9[1(0 + zy) – 0 + x(z + 3yz + y)]

66 MATHEMATICS TODAY | MAY‘18


= 9[zy + xz + 3xyz + xy] 16. Given curve is, 16x2 + 9y2 = 145 ...(i)
= 9(3xyz + xy + yz + zx) = R.H.S. Hence proved Differentiating (i) both sides w.r.t. x, we get
14. We have, (x2 + y2)2 = xy −32 x −16 x
32x + 18yy′ = 0 ⇒ y ′ = = ...(ii)
Differentiating both sides, we get 18 y 9 y
Now, (i) passes through (x1, y1)
 dy  dy
2(x 2 + y 2 ) 2 x + 2 y  = x + y ⋅ 1 \ 16x12 + 9y12 = 145
 dx  dx ⇒ 16(2)2 + 9y12 = 145 (Q x1 = 2)
 dy  dy ⇒ y12 = 9 ⇒ y1 = ±3
⇒ 4(x 2 + y 2 )  x + y  = x + y
 dx  dx But y1 > 0 \ y1 = 3
dy \ From (ii), we get
⇒ 4x(x2 + y2) – y = (x – 4y(x2 + y2)) −16 x1 −16 2 −32
dx y′ = ⇒ y′ = × =
dy 4 x 3 + 4 xy 2 − y 9 y1 9 3 27
∴ =
dx x − 4 yx 2 − 4 y 3 So, equation of tangent passing through (2, 3) and having
OR  −32 
slope  is
We have, x = a(2q – sin2q) ...(i)  27 
and y = a(1 – cos2q) ...(ii) −32
y −3= (x − 2) ⇒ 32x + 27y – 145 = 0
Differentiating (i) and (ii) w.r.t. q, we get 27
Also, equation of normal passing through (2, 3) is (y – 3)
dx dy
= a(2 –2cos2q), = 2a sin2q −1
dθ dθ = (x – 2) ⇒ 27x – 32y + 42 = 0
 −32 
dy dy / dθ 2a sin 2θ sin 2θ  27 
Now, = = =
dx dx / dθ a(2 − 2 cos 2θ) 1 − cos 2θ OR
4
x
π
sin 2  
π
sin  π −  We have f(x) = – x3 – 5x2 + 24x + 12 ...(i)
dy 3  3 4
∴ = = f ( x ) being a polynomial function is continuous and
dx 2π π
1 − cos  π − 
π
θ= 1 − cos derivable on R.
3 3  3
Differentiating (i) w.r.t. x, we get
π
sin   3 4x3
3 f ′( x ) = − 3x 2 − 10 x + 24 = x3 – 3x2 – 10x + 24
2 3 1 4
= = = =
 π  1+ 1 3 3 = (x – 2) (x2 – x – 12) = (x – 2) (x – 4) (x + 3)
1 + cos
3 2 (a) For increasing, f ′(x) > 0
15. We have, y = sin(sinx) ⇒ (x – 2) (x – 4) (x + 3) > 0
Differentiating both sides w.r.t. x, we get ⇒ x ∈(4, ∞) ∪ (–3, 2)
dy (b) For decreasing, f ′(x) < 0
= cos(sin x ) ⋅ cos x ⇒ (x – 2) (x – 4) (x + 3) < 0
dx
⇒ x ∈ (2, 4) ∪ (–∞, –3)
Again differentiating w.r.t. x both sides, we get
d2 y For complete solutions refer to MTG CBSE Champion
= –sin(sinx) · cosx. cosx + (–sinx) cos(sinx)
2 Mathematics
dx
= – cos2x · sin(sinx) – sinx · cos(sinx)
d2 y dy 
Now, L.H.S. = + y cos2 x
+ tan x
dx 2 dx MPP-1 CLASS XI ANSWER KEY
= –cos2x · sin(sinx) – sinx · cos(sinx) 1. (d) 2. (c) 3. (b) 4. (d) 5. (b)
+ tanx(cosx · cos(sinx)) + cos2x · sin(sinx)
6. (c) 7. (a, c) 8. (a) 9. (c) 10. (a, c, d)
= – cos2x · sin(sinx) –sinx · cos(sinx) + sinx · cos(sinx)
11. (a, b) 12. (a, d) 13. (b, c) 14. (d) 15. (c)
+ cos2x · sin(sinx)
= 0 = R.H.S. 16. (c) 17. (4) 18. (1) 19. (6) 20. (7)

MATHEMATICS TODAY | MAY‘18 67


CLASS XI Series 1

CBSE
Sets
Set is the well defined collection of objects. INTERVALS AS SUBSETS OF R
Let a, b ∈R and a < b, then
REPRESENTATION • Closed interval : [a, b] = {x : a ≤ x ≤ b}
OF SETS • Semi closed or semi open interval :
[a, b) = {x : a ≤ x < b} and (a, b] = { x : a < x ≤ b}
Roster/Tabular form: The elements are • Open interval : (a, b) = {x : a < x < b}
separated by commas and are enclosed within
braces { }. POWER SET
• Set of all the subsets of a set is called power set of a
Set builder form : The elements are set which is denoted by P(A) i.e., P(A) = {S : S ⊂ A},
represented by using a variable x followed by where S is subset of set A.
‘:’ and after ‘:’ we write the characteristic or UNIVERSAL SET
property possessed by the elements. The whole
• If all sets under consideration are subsets of a
description is enclosed within braces { }.
larger set, then this larger set is called universal set,
denoted by U.
TYPES OF SETS
COMPLEMENT OF A SET
• Empty Set - Set which does not contain any element.
It is denoted by f or { }. • The complement of a given set is the set which
contains all those members of the universal set that
• Finite Set - Set which is empty or having finite does not belong to the given set.
number of elements. • The complement of the set A is denoted by A′ or by
• Infinite Set - Set which is not a finite set. Ac = {x : x ∈ U, x ∉ A}.
• Equal Sets - Two given sets having exactly the same OPERATIONS ON SETS
elements.
• Union of Sets : For two sets A and B, union of sets
SUBSET written as A ∪ B is the set of all those elements
For any two sets P and Q, which belong to A or B or both.
Symbolically, A ∪ B = {x : x ∈ A or x ∈ B}
• P is said to be the subset of Q, i.e. P ⊆ Q if every • Intersection of Sets : For two sets A and B,
element of P is also an element of Q. intersection of sets written as A ∩ B is the set of all
• If P ⊂ Q and P ≠ Q, then P is called proper subset of those elements which belong to A and B both.
Q and Q is called superset of P. Symbolically, A ∩ B = {x : x ∈ A and x ∈ B}

68 MATHEMATICS TODAY | MAY‘18


• Difference of Sets : For two sets A and B, difference 2. If X and Y are two sets such that n(X) = 17,
of sets written as A – B is the set of elements which n(Y) = 23 and n(X ∪ Y) = 38, find n(X ∩ Y).
are in A but not in B. 3. Are the sets A = {x : x is a letter in the word "LOYAL"},
Symbolically, A – B = {x : x ∈ A and x ∉ B} B = {x : x is a letter of the word "ALLOY"} equal?
Similarly, B – A = {x : x ∈ B and x ∉ A}
4. List all the elements of sets:
IMPORTANT PROPERTIES (i) C = {x : x = 2n, n ∈ N and n ≤ 5}
• Properties of union (ii) H = {x : x = n2, n ∈ N, 2 ≤ n ≤ 5}
(i) A ∪ B = B ∪ A (Commutative law)
(ii) (A ∪ B) ∪ C = A ∪ (B ∪ C) (Associative law)
(iii) A ∪ f = A (Law of identity element)
{
1 1 1 1
}
5. Write the set X = 1, , , , , ..... in the set-
builder form.
4 9 16 25

(iv) A ∪ A = A (Idempotent law) SHORT ANSWER TYPE


(v) U ∪ A = U (Law of U) 6. State which of the following sets are finite and
• Properties of intersection which are infinite:
(i) A ∩ B = B ∩ A (Commutative law) (i) A = {x : x ∈ Z and x2 – 5x + 6 = 0}
(ii) (A ∩ B) ∩ C = A ∩ (B ∩ C) (Associative law) (ii) B = {x : x ∈ Z and x2 is even}
(iii) f ∩ A = f, U ∩ A = A (Law of f and U) (iii) C = {x : x ∈ Z and x2 = 36}
(iv) A ∩ A = A (Idempotent law) (iv) D = {x : x ∈ Z and x > –10}
• Distributive laws 7. Express each of following sets as an interval:
(i) A ∩ (B ∪ C) = (A ∩ B) ∪ (A ∩ C) (i) A = {x : x ∈ R, –4 < x < 0}
(ii) A ∪ (B ∩ C) = (A ∪ B) ∩ (A ∪ C) (ii) B = {x : x ∈ R, 0 ≤ x < 3}
• Complement laws (iii) C = {x : x ∈ R, 2 < x ≤ 6}
(i) A ∪ A′ = U (iv) D = {x : x ∈ R, –5 ≤ x ≤ 2}
(ii) A ∩ A′ = f 8. Describe the following sets in Roster form:
• De Morgan’s law (i) The set of all letters in the word 'MATHEMATICS'
(i) (A ∪ B)′ = A′ ∩ B′ (ii) The set of squares of integers.
(ii) (A ∩ B)′ = A′ ∪ B′ 9. A survey shows that 73% of the Indians like apples,
• Law of double complementation whereas 65% like oranges. What percentage of
(A′)′ = A Indians like both apples and oranges?
• Laws of empty set and universal set 10. Write the following sets in Roster form :
f′ = U and U′ = f (i) A = {an : n ∈ N, an + 1 = 3an and a1 = 1}
(ii) B = {an : n ∈ N, an + 2 = an + 1 + an, a1 = a2 = 1}
VENN DIAGRAM
LONG ANSWER TYPE - I
• A diagram used to illustrate relationship between the
11. Write the following subsets of R as intervals:
sets.
(i) {x : x ∈ R, –4 < x ≤ 6}
IMPORTANT FORMULAE BASED ON NUMBER OF (ii) {x : x ∈ R, –12 < x < –10}
ELEMENTS IN SETS (iii) {x : x ∈ R, 0 ≤ x < 7}
(iv) {x : x ∈ R, 3 ≤ x ≤ 4}
For two finite sets A and B, we have Also, find the length of each interval.
• n(A ∪ B) = n(A) + n(B) – n(A ∩ B), if A ∩ B ≠ f
12. For any sets A and B, show that A – B = A ∩ B′
• n(A ∪ B) = n(A) + n(B), if A ∩ B = f
• n(A ∪ B ∪ C) = n(A) + n(B) + n(C) – n(A ∩ B) 13. Let A, B and C be the sets such that A ∪ B = A ∪ C
and A ∩ B = A ∩ C. Show that B = C.
– n(B ∩ C) – n(C ∩ A) + n(A ∩ B ∩ C)
14. In a survey of 100 students, the number of students
WORK IT OUT
studying the various languages is found as: English
VERY SHORT ANSWER TYPE only 18; English but not Hindi 23; English and
1. Write the set A = {x : x ∈ Z, x2 < 20} in the roster Sanskrit 8; Sanskrit and Hindi 8; English 26;
form. Sanskrit 48 and no language 24.

MATHEMATICS TODAY | MAY‘18 69


(i) How many students are studying Hindi? 3. We have, A = {L, O, Y, A, L} = {L, O, Y, A}
(ii) How many students are studying English and and B = {A, L, L, O, Y} = {L, O, Y, A}
Hindi both? Clearly, A = B.
15. If A = {x : x = N, x is a factor of 6} and B = {x ∈ N : 4. (i) C = {2, 4, 6, 8, 10}
x is a factor of 8}, then find (ii) H = {4, 9, 16, 25}
(i) A ∪ B (ii) A ∩ B 5. The elements of set X are the reciprocals of the
(iii) A – B (iv) B – A squares of all natural numbers. So, the set X in set
LONG ANSWER TYPE - II 1 
builder form is X =  2 : n ∈ N .
16. A college awarded 38 medals for Honesty, 15 for n 
Punctuality and 20 for Obedience. If these medals 2
6. (i) A = {x : x ∈ Z and x – 5x + 6 = 0} = {2, 3}
were bagged by a total of 58 students and only 3 So, A is a finite set.
students got medals for all three values, how many (ii) B = {x : x ∈ Z and x2 is even}
students received medals for exactly two of the = {..., –6, –4, –2, 0, 2, 4, 6, ...}
three values? Clearly, B is an infinite set.
17. There are 240 students in class XI of a school, 130 (iii) C = {x : x ∈ Z and x2 = 36} = {6, –6}
play cricket, 100 play football, 75 play volleyball, 30 Clearly, C is a finite set.
of these play cricket and football, 25 play volleyball (iv) D = {x : x ∈ Z and x > –10} = {–9, –8, –7, ...}
and cricket, 15 play football and volleyball. Also Clearly, D is an infinite set.
each student plays atleast one of the three games. 7. (i) A = (–4, 0) (ii) B = [0, 3)
How many students play all the three games? (iii) C = (2, 6] (iv) D = [–5, 2]
18. In a town of 10000 families, it was found that 8. (i) We observe that distinct letters in the word
40% families buy newspaper A, 20% families buy 'MATHEMATICS' are:
newspaper B and 10% families buy newspaper M, A, T, H, E, I, C, S
C, 5% families buy A and B, 3% buy B and C and Since the order in which the elements of a set are
4% buy A and C. If 2% families buy all the three written is immaterial and the repetition of elements
newspaper, find the number of families which buy has no effect. So, required set can be described as
(i) A only (ii) B only follows :
(iii) None of A, B and C. {M, A, T, H, E, I, C, S}.
19. A survey of 500 television viewers produced the (ii) Since square of a negative integer is same as
following information; 285 watch football, 195 the square of its absolute value. Therefore, squares
watch hockey, 115 watch basketball, 45 watch of integers are 0, 1, 4, 9, 16, 25,...... Hence, required
football and basketball, 70 watch football and set is {0, 1, 4, 9, 16,....}.
hockey, 50 watch hockey and basketball, 50 do not 9. Let A = set of Indians who like apples
watch any of the three games. How many watch all and B = set of Indians who like oranges.
the three games? How many watch exactly one of Then, n(A) = 73, n(B) = 65 and n(A ∪ B) = 100
the three games? n(A ∩ B) = n(A) + n(B) – n(A ∪ B)
20. A survey shows that 63% of the Americans like = 73 + 65 – 100 = 38.
cheese whereas 76% like apples. If x% of the Hence, 38% of the Indians like both apples and oranges.
Americans like both cheese and apples, find the 10. (i) We have, a1 = 1 and an + 1 = 3an for all n ∈ N
value of x. Putting n = 1, 2, 3, 4, 5,......... in an + 1 = 3an, we get
SOLUTIONS a2 = 3a1 = 3 × 1 = 3 [ a1 = 1]
1. The integers whose squares are less than 20 are 0, a3 = 3a2 = 3 × 3 = 32 [ a2 = 3]
± 1, ± 2, ± 3, ± 4. a4 = 3a3 = 3 × 32 = 33 [ a3 = 32]
\ Set A in roster form is A = {–4, –3, –2, –1, 0, 1, 2, 3, 4}. a5 = 3a4 = 3 × 3 = 3 , a6 = 3a5 = 3 × 34 = 35 and
3 4

2. We have, n(X ∪ Y) = n(X) + n(Y) – n(X ∩ Y) so on.


⇒ 38 = 17 + 23 – n(X ∩ Y) Hence, A = {a1, a2, a3, a4, a5, a6, ...}
⇒ n(X ∩ Y) = 40 – 38 = 2 = {1, 3, 32, 33, 34, 35, ........}

70 MATHEMATICS TODAY | MAY‘18


MATHEMATICS TODAY | MAY‘18 71
(ii) We have, a1 = 1, a2 = 1 and an + 2 = an + 1 + an. (i) Number of students studying Hindi
Putting n = 1, 2, 3, 4, ..... in an + 2 = an + 1 + an, we get = (b + c + e + f) = (0 + 3+ 10 + 5) = 18.
a3 = a2 + a1 = 1 + 1 = 2; a4 = a3 + a2 = 2 + 1 = 3; (ii) Number of students studying English and
a5 = a4 + a3 = 3 + 2 = 5; a6 = a5 + a4 = 5 + 3 = 8 and Hindi both = (b + c) = (0 + 3) = 3.
so on. 15. We have,
Hence, B = {a1, a2, a3, a4, a5, a6, .......} A = {x : x ∈ N, x is a factor of 6} = {1, 2, 3, 6}
= {1, 1, 2, 3, 5, 8, ........} and B = {x : x ∈ N, x is a factor of 8} = {1, 2, 4, 8}
11. (i) {x : x ∈ R, –4 < x ≤ 6} = (–4, 6] (i) A ∪ B = {1, 2, 3, 6} ∪ {1, 2, 4, 8} = {1, 2, 3, 4, 6, 8}
Length = 6 – (–4) = 10 (ii) A ∩ B = {1, 2, 3, 6} ∩ {1, 2, 4, 8} = {1, 2}
(ii) {x : x ∈ R, –12 < x < –10} = (–12, –10) (iii) A – B = {1, 2, 3, 6} – {1, 2, 4, 8} = {3, 6}
Length = –10 – (–12) = 2 (iv) B – A = {1, 2, 4, 8} – {1, 2, 3, 6} = {4, 8}
(iii) {x : x ∈ R, 0 ≤ x < 7} = [0, 7) 16. Let H, P and O be the sets of students who received
Length = 7 – 0 = 7 medals in Honesty, Punctuality and Obedience
(iv) {x : x ∈ R, 3 ≤ x ≤ 4} = [3, 4] respectively. Then,
Length = 4 – 3 = 1 n(H) = 38, n(P) = 15, n(O) = 20, n(H ∪ P ∪ O) = 58
12. Let x ∈ A – B. Then, and n(H ∩ P ∩ O) = 3.
x ∈ A – B ⇒ x ∈ A and x ∉ B Now, n(H ∪ P ∪ O) = n(H) + n(P) + n(O)
⇒ x ∈ A and x ∈ B′ ⇒ x ∈ A ∩ B′. – n(H ∩ P) – n(P ∩ O) – n(H ∩ O) + n(H ∩ P ∩ O)
\ (A – B) ⊆ (A ∩ B′) ...(i) ⇒ 58 = 38 + 15 + 20 – n(H ∩ P) – n(P ∩ O)
Again, let y ∈ (A ∩ B′). Then, – n(H ∩ O) + 3
y ∈ (A ∩ B′) ⇒ y ∈ A and y ∈ B′ ⇒ n(H ∩ P) + n(P ∩ O) + n(H ∩ O) = 18 ...(i)
⇒ y ∈ A and y ∉ B In the adjoining Venn diagram, H
⇒ y ∈ (A – B). let x denote the number of
\ (A ∩ B′) ⊆ (A – B) ...(ii) students who got medals in
From (i) and (ii), we get (A – B) = (A ∩ B′). Honesty and Punctuality only, y z
3
x
denote the number of students y P
13. We have, A ∪ B = A ∪ C who got medals in Punctuality O
⇒ (A ∪ B) ∩ C = (A ∪ C) ∩ C and Obedience only, z denote the number of
⇒ (A ∩ C) ∪ (B ∩ C) = C [ (A ∪ C) ∩ C = C] students who got medals in Honesty and Obedience
⇒ (A ∩ B) ∪ (B ∩ C) = C ...(i) only. 3 students got medals in all the three values.
Again, A ∪ B = A ∪ C Using (i) and Venn diagram, we get
⇒ (A ∪ B) ∩ B = (A ∪ C) ∩ B (x + 3) + (y + 3) + (z + 3) = 18
⇒ B = (A ∩ B) ∪ (C ∩ B) [ (A ∪ B) ∩ B = B] ⇒ x + y + z = 9.
⇒ B = (A ∩ B) ∪ (B ∩ C) ...(ii) Hence, the number of students who received medals
From (i) and (ii), we get B = C. in exactly two values out of the three values = 9.
14. Let E, H, S denote the sets of students studying 17. Let C, F and V be the sets C F
English, Hindi and Sanskrit respectively. of students who play cricket,
75 + x 55 + x
30 – x

In the adjoining Venn diagram, let a, b, c, d, e, f and football and volleyball


g denote the number of students in the respective respectively. –x x
15

25
–x

regions. According to given E H Let x be the number of


35 + x
information, we have students who play all the three
a b e V
a = 18, a + d = 23, c + d = 8, games, then the number of
c
c + f = 8, a + b + c + d = 26, d f students according to given information in the
c + d + f + g = 48, g question are shown in different regions of the
a+b+c+d+e+f+g S adjoining Venn diagram.
= 100 – 24 = 76. As each student plays atleast one of three games,
\ a = 18, d = 5, c = 3, n(C ∪ F ∪ V) = 240.
f = 5 and b = 0. From the Venn diagram, we have
\ g = 48 – (3 + 5 + 5) = 35 (75 + x) + (30 – x) + (55 + x) + (15 – x) + (35 + x) +
and e = 76 – (18 + 0 + 3 + 5 + 5 + 35) = 10. (25 – x) + x = 240

72 MATHEMATICS TODAY | MAY‘18


⇒ 75 + 30 + 55 + 15 + 35 + 25 + x = 240 Now, n(A ∩ B) = n(A) + n(B) – n(A ∪ B)
⇒ 235 + x = 240 ⇒ x = 5. ⇒ n(A ∩ B) = 63 + 76 – n(A ∪ B) = 139 – n(A ∪ B)
Hence, 5 students play all the three games. But, n(A ∪ B) ≤ 100.
18. Let P, Q and R denote the P ⇒ –n(A ∪ B) ≥ – 100
Q
sets of families who buy ⇒ 139 – n(A ∪ B) ≥ 139 – 100
newspapers A, B and C 3300 1400 ⇒ 139 – n(A ∪ B) ≥ 39

300
respectively, then 200 ⇒ n(A ∩ B) ≥ 39 ...(i)
0 100
n(P) = 40% of 10000 = 4000, 20 Now, A ∩ B ⊆ A and A ∩ B ⊆ B
n(Q) = 20% of 10000 = 2000, 500
⇒ n(A ∩ B) ≤ n(A) and n(A ∩ B) ≤ n(B)
R
n(R) = 10% of 10000 = 1000, ⇒ n(A ∩ B) ≤ 63 and n(A ∩ B) ≤ 76
n(P ∩ Q) = 5% of 10000 = 500, ⇒ n(A ∩ B) ≤ 63 ...(ii)
n(Q ∩ R) = 3% of 10000 = 300, From (i) and (ii), we obtain
n(P ∩ R) = 4% of 10000 = 400 and 39 ≤ n(A ∩ B) ≤ 63
n(P ∩ Q ∩ R) = 2% of 10000 = 200 ⇒ 39 ≤ x ≤ 63.
From the Venn diagram, we get
(i) Number of families who buy newspaper A
only = 3300.
(ii) Number of families who buy newspaper B
INDIA’S BEST EDUCATORS
only = 1400. The Indian Institute of Science, Bangalore, has again been
(iii) Number of families who buy newspaper A, B declared the best educational Institution by the HRD
ministry’s National Institutional Ranking Framework. India
or C i.e. atleast one of the newspaper Rankings 2018 show Delhi’s Miranda House is the best college
= 4000 + 1400 + 100 + 500 = 6000
\ Number of families who do not buy any of Rank went up Rank slipped Same rank as 2017

newspaper A, B and C = 10000 – 6000 = 4000. OVERALL RANKINGS


Rank Rank Participation must for public institutes
2018 Institution 2017
19. Let N = Total number of television viewers = 500, 1 Indian Institute of 1
 2,809 institutions
participated in nine categories.
 The HRD ministry has made
participation by public
n(F) = 285, n(H) = 195, n(B) = 115, n(F ∩ B) = 45, n(F Science, Bangalore
2 IIT Madras 2
Some colleges, including institutes mandatory from
St. Stephen’s and Hindu College, 2019.
∩ H) = 70, n(H ∩ B) = 50, n(F′ ∩ H′ ∩ B′) = 50 3 IIT Bombay 3 did not participate in the

Now, n(F′ ∩ H′ ∩ B′) = 50 4 IIT Delhi 5 ranking process last year but
are part of the latest edition
5 IIT Kharagpur 4
⇒ n[(F ∪ H ∪ B)′] = 50 6 Jawaharlal Nehru 6
University
⇒ N – n(F ∪ H ∪ B) = 50 Methodology
7 IIT Kanpur 7 The rankings are ascertained on the basis of teaching, learning
⇒ 500 – [n(F) + n(H) + n(B) – n(F ∩ H) 8 IIT Roorkee 9 and resources, research and professional practices, graduation
9 Banaras Hindu 10 outcomes, outreach and inclusivity and perception.
– n(F ∩ B) – n(H ∩ B) + n(F ∩ H ∩ B)] = 50 University, Varanasi The system of announcing the rankings every year was

⇒ n(F ∩ H ∩ B) = 500 – 285 – 195 – 115 + 70 10 Anna University, 13 introduced by the HRD ministry in 2016
Chennai
+ 50 + 45 – 50 = 20. UNIVERSITIES COLLEGES ENGINEERING
Hence, number of viewers who watch all the three Rank
2018 Institution 2017
Rank Rank
2018 Institution 2017
Rank COLLEGES
Rank Rank
games = 20 1 Indian Institute of
Science, Bangalore
1 1 Miranda House,
Delhi University
1 2018 Institution 2017
1 IIT Madras 1
Number of viewers who watch exactly one of the 2 Jawaharlal Nehru 2 2 St. Stephens, Delhi n/a
University, New Delhi University 2 IIT Bombay 2
three games = n(F ∩ H′ ∩ B′) + n(F′ ∩ H′ ∩ B) 3 Banaras Hindu 3 3 Bishop Heber College, 4
University, Varanasi 3 IIT Delhi 4
+ n(F′ ∩ H ∩ B′) 4 Anna University, 6
Tiruchirappalli
4 Hindu College, Delhi n/a 4 IIT Kharagpur 3
= n(F) + n(H) + n(B) – 2[n(F ∩ H) + n(H ∩ B) Chennai
5 Presidency College, n/a
5 University of 7 Chennai 5 IIT Kanpur 5
+ n(B ∩ F)] + 3n(F ∩ H ∩ B) Hyderabad
6 Loyola College, 2
6 Jadavpur University, 5 6 IIT Roorkee 6
= 285 + 195 + 115 – 2 (70 + 50 + 45) + 3(20) Kolkata
Chennai

7 University of Delhi, 8 7 Shri Ram College for 3 7 IIT Guwahati 7


= 595 – 2(165) + 60 = 595 – 330 + 60 = 325 New Delhi Commerce, Delhi

8 Amrita Vishwa 9 8 Lady Shri Ram College 7 8 Anna University, 8


20. Let A denote the set of Americans who like cheese, Vidyapeetham,
Coimbatore
for Women, Delhi Chennai
9 Ramakrishna Mission n/a
9 10
B denote those who like apples and x denote those 9 Savitribai Phule Pune 10 Vidyamandira,
Howrah
IIT Hyderabad
University
who like both cheese and apples. Let the population 10 Institute of 14
10 Aligarh Muslim 11 10 Madras Christian 12 Chemical
University, Aligarh College, Chennai Technology, Mumbai
of America be 100. Then, n(A) = 63, n(B) = 76.

MATHEMATICS TODAY | MAY‘18 73


MPP-1 Class XI

T his specially designed column enables students to self analyse


their extent of understanding of specified chapters. Give yourself
four marks for correct answer and deduct one mark for wrong answer.
Self check table given at the end will help you to check your
readiness.

Sets, Relations and Functions


Total Marks : 80 Time Taken : 60 Min.
Only One Option Correct Type 6. If R = {(x, y): x, y ∈ I, x2 + y2 ≤ 4} is a relation in I, then
domain of R is
1
1. If f(x) = 4x3 + 3x2 + 3x + 4, then x 3 f   is equal to (a) {0, 1, 2} (b) {–2, –1, 0}
x
(c) {–2, –1, 0, 1, 2} (d) None of these
1
(a) f(–x) (b)
f (x ) One or More Than One Option(s) Correct Type
2
  1  7. If X ∪ {1, 2} = {1, 2, 3, 5, 9}, then
(c)  f    (d) f(x)
  x  (a) the smallest set X is {3, 5, 9}
2. Let A = {1, 2, 3} and B = {2, 3, 4}, then which of the (b) the smallest set X is {2, 3, 5, 9}
following is a function from A to B? (c) the largest set X is {1, 2, 3, 5, 9}
(d) the largest set X is {2, 3, 4, 9}
(a) {(1, 2), (2, 3), (3, 4), (2, 2)}
(b) {(1, 2), (2, 3), (1, 3)} 8. The set (A ∪ B ∪ C) ∩ (A ∩ B′ ∩ C′)′ ∩ C′ is equal to
(c) {(1, 3), (2, 3), (3, 3)} (a) B ∩ C′ (b) A ∩ C
(d) {(1, 1), (2, 3), (3, 4)} (c) B′ ∩ C′ (d) None of these

3. If f (x) is a polynomial function satisfying f(x) f(y) 1


9. If f (x ) + 2 f   = 3x , x ≠ 0, and
= f(x) + f (y) + f (xy) – 2 for all real x and y and x
f(3) =10, then f(4) is equal to S = {x ∈ R : f (x) = f (–x)}; then S
(a) 16 (b) 17 (c) 18 (d) 19 (a) is an empty set
(b) contains exactly one element
1 x 
(c) contains exactly two elements
4. If f(x) = cos (logx), then f(x) f(y) −  f  y  + f (xy )
2  (d) contains more than two elements
has the value
1 10. In a group of 50 students, the number of students
(a) –1 (b) (c) –2 (d) 0 studying French, English, Sanskrit were found to be
2
as follows.
5. In a survey of 200 people of a town, it was found French = 17, English = 13, Sanskrit = 15
that 120 like tea, 90 like coffee and 70 like cold- French and English = 09, English and Sanskrit = 4
drink, 40 like tea and coffee, 30 like coffee and cold- French and Sanskrit = 5, English, French and
drink, 50 like cold-drink and tea and 20 none of Sanskrit = 3. The number of students who study
these beverages. The number of people who like all (a) French only is 6
the three beverages is (b) Sanskrit only is 8
(a) 30 (b) 20 (c) French and Sanskrit but not English is 2
(c) 22 (d) 25 (d) atleast one of the three languages is 30

74 MATHEMATICS TODAY | MAY‘18


11. The domain and range of the function f given by
Q. The domain of the function
f(x) = 2 – |x – 5| is
f defined by
(a) Domain = R (b) Range = (– ∞, 2] 1 2. R – {2, 6}
(c) Domain = R – {0} (d) Range = (– ∞, 2) f(x) = 4 − x +
x2 − 1
1 is equal to
12. For the function f (x ) =   , where [x] denotes
[ x ]  R. If R is the set of all
the greatest integer less than or equal to x, which of real numbers and if
the following statements is/are false? f : R – {2} → R is defined by
(a) The domain is (–∞, ∞).  3 5
2+x 3.  2 , 2 
(b) The range is {0} ∪ {–1} ∪ {1}. f (x ) = for 
2−x
(c) The domain is (–∞, 0) ∪ [1,∞). x → R – {2}, then the range
(d) The range is {0} ∪ {1}. of f is
13. Consider the following relations :
The domain of the function
I. A – B = A – (A ∩ B) S.
II. A = (A ∩ B) ∪ (A – B) x 2 + 2x + 1
f (x ) = 2 is 4. R – {–1}
III. A – (B ∪ C) = (A – B) ∪ (A – C) x − 8 x + 12
Which of these is/are correct?
(a) I and III (b) II
P Q R S
(c) I (d) II and III
(a) 1 3 2 4
Comprehension Type (b) 2 1 3 4
Out of 800 boys in a school, 224 played Cricket, 240 (c) 3 1 4 2
played Hockey and 336 played Basketball. Of the total, (d) 3 4 1 2
64 played both Basketball and Hockey; 80 played Cricket Integer Answer Type
and Basketball and 40 played Cricket and Hockey; 24
played all the three games.  5x − x 2 
17. If domain of log  is [1, x] then x is equal
14. The number of boys who did not play any game is  4 
(a) 128 (b) 216 (c) 240 (d) 160 to
15. The number of boys who played only Cricket is 18. If F is function such that F(0) = 2, F(1) = 3,
(a) 240 (b) 220 (c) 128 (d) 320
F(x + 2) = 2F(x) – F(x + 1) for x > 0, then unit place
Matrix Match Type in F(5) is equal to
16. Match the following. 19. Let Z denote the set of all integers and
Column I Column II A = {(a, b) : a2 + 3b2 = 28, a, b ∈ Z} and
P. The domain of definition of the B = {(a, b) : a > b, a, b ∈ Z}, then the number of
function elements in A ∩ B is
(–∞,–1) ∪
log(2 x − 3) 1.
f (x ) = + 5 − 2 x is (1, 4] 20. If A = {x : x = n2, n = 1, 2, 3}, then number of proper
x −1 subsets is

Keys are published in this issue. Search now! J
Check your score! If your score is
> 90% EXCELLENT WORK ! You are well prepared to take the challenge of final exam.

No. of questions attempted …… 90-75% GOOD WORK ! You can score good in the final exam.

No. of questions correct …… 74-60% SATISFACTORY ! You need to score more next time.
Marks scored in percentage …… < 60% NOT SATISFACTORY! Revise thoroughly and strengthen your concepts.

MATHEMATICS TODAY | MAY‘18 75


76 MATHEMATICS TODAY | MAY‘18
CLASS XII Series 1

CBSE
Relations and Functions
RELATIONS The element y ∈ B is called the image of x under f and
x is called the pre-image of y under f.
Let A and B be two non-empty sets. The relation R
between A and B is a subset of A × B. Symbolically, we Domain, Co-domain, Range of Function f(x)
write the relation between A and B as R : A → B if and
only if R ⊆ A × B The set A is called the domain of
Domain
function f.

Types of Relations The set B is called the co-domain of


Co-domain
function f.
The set {f(x) : x ∈ A and f(x) ∈ B}
Reflexive Relation : A relation R on a set A
for all values taken by f is called the
is reflexive, if (a, a) ∈R, ∀ a ∈A. Range
range of f. Obviously, it is a subset
of set B.
Symmetric Relation : A relation R on a set
A is symmetric if (a1, a2) ∈R ⇒ (a2, a1)∈R ∀ Types of Functions
a1, a2 ∈A
Type Definition Representation
Transitive Relation : A relation R on a set One-one A function f : X → Y f
A is transitive if (a1, a2) ∈R and (a2, a3) ∈R (Injective) is one-one, if different X Y
⇒ (a1, a3) ∈ R ∀ a1, a2, a3 ∈ A. Function elements of X have
different images in Y
Equivalence Relation : A relation R is an under f.
equivalence relation if R is reflexive, symmetric
and transitive. Onto A function f : X → Y is X f Y
(Surjective) onto, if every element of
Function Y is the image of some
FUNCTIONS element of X under f.
Let A and B be two non-empty sets. If there exists a Many-one A function f : X → Y
correspondence by which each element x ∈ A is related Function is many-one, if two or
to a unique element y ∈ B, then such correspondence is more than two elements
called the function from A to B. It is written as of X have the same
f : A → B such that x ∈ A and y ∈ B, where y = f(x). image in Y.

MATHEMATICS TODAY | MAY‘18 77


Into A function f : X → Y is X f 2. Let R be the set of all real numbers. Let f : R → R
Function into, if there exists a single
Y such that f(x) = sin x and g : R → R such that g(x) = x2.
Prove that gof ≠ fog.
element in Y having no
pre-image in X. 3. Let N be the set of natural numbers and relation R
on N be defined by
Bijective A function f : X → Y f R = {(x, y); x, y ∈ N, x + 4y = 10}
X Y
Function is bijective, if it is both Determine whether the above relation is reflexive,
one-one and onto. symmetric.
4. Let f : R → R be defined as f(x) = x2 + 1. Find :
(i) f –1(–5) (ii) f –1{10, 37}
COMPOSITION OF FUNCTIONS 5. If the binary operation ‘*’ on I is defined by
For two functions f : X → Y and g : Y → Z, composition a * b = 2a – 3b, then find
of functions is denoted by gof defined as
(i) 3 * 7 (ii) 7 * 3
gof (x) = g(f(x)) ∀ x ∈ X.
g
SHORT ANSWER TYPE
X f Y Z
1 1
f(x)}
6. If f (x ) = , x ≠ − , then show that
x f(x) 2x + 1 2
2x + 1 1 3
f ( f (x )) = , provided that x ≠ − , − .
gof(x) 2x + 3 2 2
7. Show that the function f : N → N defined by
INVERSE OF A FUNCTION f(x) = x3 is injective but not surjective.
• Inverse of a function f : A → B is defined as f –1 : B → A 8. Let ‘*’ be a binary operation on N given by
s.t. f –1(y) = x ⇔ f(x) = y a * b = HCF (a, b) for all a, b ∈ N.
• A function f is invertible if and only if f is one-one onto. (i) Find 12 * 4, 18 * 24, 7 * 5.
(ii) Check the commutativity and associativity of
BINARY OPERATION ‘*’ on N.
Let A be a non-empty set, then the function from A × A 9. Is f : R → R defined by f(x) = |x| + x is one-one or
into A is called a binary operation. Symbolically, a onto? Also find the range of f.
function '*' which is * : A × A → A is called a binary
operation. The image of any (a, b) ∈ A × A under '*' is 10. Prove that the inverse of a bijection is unique.
denoted by a * b. LONG ANSWER TYPE - I
(i) A binary operation '*' over a set A is said to be 11. Let A = Q × Q and let * be a binary operation on
commutative if a * b = b * a ∀ a, b ∈ A A defined by (a, b) * (c, d) = (ac, b + ad) for (a, b),
(ii) A binary operation '*' over a set A is said to be (c, d) ∈ A. Then with respect to * on A.
associative if (a * b) * c = a * (b * c) ∀ a, b, c ∈ A (i) Find the identity element in A.
(iii) Let * be a binary operation on a set A. An element (ii) Find the invertible elements of A.
e ∈ A is said to be an identity element for the
binary operation * if a * e = a = e * a ∀ a ∈ A 12. Let A be the set of all lines in a plane and let R be a
(iv) Let * be a binary operation on a set A and let relation in A defined by
e be the identity element of the set A for this R = {(L1, L2) : L1 ⊥ L2}.
operation *. An element b ∈ A is said to be the Show that R is symmetric but neither reflexive nor
inverse of an element a ∈ A if a * b = e = b * a. transitive.
WORK IT OUT 4x + 3 2
13. If f(x) = , x ≠ , show that (fof )(x) = x for all
VERY SHORT ANSWER TYPE 6x − 4 3
2
x ≠ . What is the inverse of f ?
1. On Q, the set of all rational numbers, a binary 3
ab 14. Let R+ be the set of all positive real numbers. Let
operation * is defined by a * b = for all a, b ∈ Q. f : R+ → [4, ∞ ) such that f (x) = x2 + 4. Show that
5
Find the identity element for * in Q. f is invertible and find f –1.

78 MATHEMATICS TODAY | MAY‘18


15. Let N be the set of all natural numbers and let R be 3. Given, R = {(x, y); x, y ∈ N, x + 4y = 10 }
a relation in N, defined by \ R = {(2, 2), (6, 1)}
R = {(a, b) : a is a multiple of b} R is not reflexive because (1, 1) ∉ R.
Show that R is reflexive and transitive but not R is not symmetric, because (6, 1) ∈ R but (1, 6) ∉ R.
symmetric. 4. (i) Let f –1(–5) = x. Then,
LONG ANSWER TYPE - II f(x) = –5 ⇒ x2 + 1= –5 ⇒ x2 = –6 ⇒ x = ± −6 ,
16. Let S be the set of all points in a plane and R be a which is not in R.
relation on S defined as So, f –1(–5) = f.
R = {(P, Q) : Distance between P and Q is less than 2 units}. (ii) f –1{10, 37} = {x ∈ R : f(x) = 10 or f(x) = 37}
Show that R is reflexive and symmetric but not = {x ∈ R : x2 + 1 = 10 or x2 + 1 = 37}
transitive. = {x ∈ R : x2 = 9 or x2 = 36} = {3, –3, 6, –6}
5. (i) 3 * 7 = 2(3) – 3(7) = 6 – 21 = –15
17. Show that f : N → N, defined by
(ii) 7 * 3 = 2(7) – 3(3) = 14 – 9 = 5
n + 1
 2 , if n is odd 1
f (n) =  6. We have, f (x ) =
2x + 1
 n , if n is even
 2
is a many-one onto function.
{ }
Clearly, domain ( f ) = R − −
1
1
2
Let y = ,
(n − 1), when n is odd 2x + 1
18. Let f : W → W : f(n) =  1 1− y
(n + 1), when n is even. ⇒ 2x + 1 = ⇒ x =
Show that f is invertible. Find f –1. y 2y
1
Since x is a real number distinct from − .
3 7  2
19. Let A = R –   and B = R −   . Therefore, y can take any non-zero real value. So,
5 5
Range (f ) = R – {0}.  1
7x + 4 ⇒ Range (f ) = R – {0} Domain (f ) = R – − 
Let f : A → B : f(x) = and
5x − 3  2
3y + 4 \ Domain (fof) = {x : x ∈ Domain (f ) and
g : B → A : g ( y) = .
5y − 7 f (x) ∈ Domain (f )}
Show that (gof) = IA and (fog) = IB.  1 1 1
⇒ Domain (fof ) =  x : x ≠ − and ≠− 
 2 2x + 1 2
20. Let X = {1, 2, 3, 4, 5, 6, 7, 8, 9}. Let R1 be a relation
on X given by R1 = {(x, y) : x – y is divisible by 3} and  1 3 
⇒ Domain (fof ) =  x : x ≠ − and x ≠ − 
R2 be another relation on X given by R2 = {(x, y) :  2 2
{x, y} ⊂ {1, 4, 7} or {x, y} ⊂ {2, 5, 8} or {x, y} ⊂ {3, 6, 9}}.  1 3
Show that R1 = R2. = R − − , − 
 2 2
SOLUTIONS  1 
Also, fof (x) = f(f(x)) = f 
1. Let e be the identity element. Then,  2 x + 1 
a * e = a = e * a , for all a ∈ Q 1 2x + 1
ae ea = =
⇒ = a and = a , for all a ∈ Q  1  2x + 3
2 +1
5 5  2 x + 1 
⇒ e=5
Thus, 5 is the identity element for the binary  1 3
Thus, fof : R − − , −  → R is defined by
operation * defined on Q.  2 2
2x + 1
2. Let x be an arbitrary real number, then fof (x ) = .
(gof ) (x) = g{f(x)} = g(sin x) = (sin x)2 2x + 3
(fog)(x) = f{g(x)} = f(x2) = sin x2 7. Let x1, x2 ∈ N be such that f(x1) = f(x2)
Clearly, (sin x)2 ≠ sin x2. ⇒ x13 = x23 ⇒ x13 – x23 = 0
Hence, gof ≠ fog. ⇒ (x1 – x2)(x12 + x1x2 + x22) = 0

MATHEMATICS TODAY | MAY‘18 79


⇒ x1 – x2 = 0 ( x1, x2 ∈ N, so x12 + x1x2 + x22 > 0) Clearly, (1, 0) ∈ Q × Q = A
⇒ x1 = x2. So, (1, 0) is the identity element in A.
Thus, f(x1) = f(x2) ⇒ x1 = x2 ⇒ f is one-one i.e., (ii) Let (a, b) be an invertible element of A. Then
injective. there exists (c, d) ∈ A such that
Since, 2 ∈ N (codomain of f ) and there does not exist (a, b) * (c, d) = (1, 0) = (c, d) * (a, b)
any x ∈ N (domain of f ) such that f(x) = 2 i.e., x3 = 2. ⇒ (ac, b + ad) = (1, 0) and (ca, d + bc) = (1, 0)
So, f is not onto i.e., f is not surjective. ⇒ ac = 1, b + ad = 0 and ca = 1, d + bc = 0
8. (i) 12 * 4 = HCF(12, 4) = 4, 18 * 24 = HCF(18, 24) = 6 1 b
⇒ c = and d = − , if a ≠ 0.
and 7 * 5 = HCF(7, 5) = 1 a a
(ii) Commutativity : For any a, b ∈ N, we have Thus, (a, b) is an invertible element of A, if a ≠ 0
a * b = HCF(a, b) = HCF(b, a) = b * a  1 −b 
and in such a case the inverse of (a, b) is  , .
So, ‘*’ is commutative on N.  a a 
Associativity: For any a, b, c ∈ N, we have 12. Clearly, any line L cannot be perpendicular to itself.
(a * b) * c = HCF(a, b) * c = HCF(a, b, c) \ (L, L) ∉ R for any L ∈ A.
and a * (b * c) = a * HCF(b, c) = HCF(a, b, c) So, R is not reflexive.
L3
\ (a * b) * c = a * (b * c) for all a, b, c ∈N. Again, let L1, L2 ∈ A. Then,
L2
So, ‘*’ is associative on N. (L1, L2) ∈ R ⇒ L1 ⊥ L2
L1
9. When x ≥ 0, |x| = x ⇒ f(x) = x + x = 2x ; ⇒ L2 ⊥ L1 ⇒ (L2, L1) ∈ R.
when x < 0, |x| = –x ⇒ f(x) = –x + x = 0 \ R is symmetric.
Since, f(x) = 0 for all x < 0 ⇒ f(–1) = 0 and f(–2) = 0 Now, let L1, L2, L3 ∈ A such that L1 ⊥ L2 and L2 ⊥ L3.
So, the two different elements –1, –2 ∈ R (domain Clearly L1 is not perpendicular to L3.
of f ) have same image. Thus, (L1, L2) ∈ R and (L2, L3) ∈ R, but (L1, L3) ∉ R.
⇒ f is not one-one. \ R is not transitive.
When x ≥ 0, f(x) = 2x ≥ 0 and when x < 0, f(x) = 0 Hence, R is symmetric but neither reflexive nor
⇒ Range of f = [0, ∞). transitive.
Since, Range of f = [0, ∞), which is a proper subset 4x + 3 2
of R (codomain of f ) 13. Given f (x ) = , x≠
6x − 4 3
⇒ f is not onto. 2
⇒ Domain of function f = R –   .
10. Let f : A → B be a bijection. If possible, let g : B → A 3
4x + 3
and h : B → A be two inverses of f. Let y = ⇒ 6 xy − 4 y = 4 x + 3
We have to prove that g = h. In order to prove this 6x − 4
it is sufficient to show that g(y) = h(y) for all y ∈ B. 4y + 3
⇒x= but x ∈ R ⇒ 6y – 4 ≠ 0
Let y be an arbitrary element of B. 6y − 4
Let g(y) = x1 and h(y) = x2. Then, 2 2
⇒ y ≠ ⇒ range of f = R –  
g(y) = x1 ⇒ f(x1) = y [ g is inverse of f] 3 3
and h(y) = x2 ⇒ f(x2) = y [ h is inverse of f] 2 2
\ f(x1) = f(x2) ⇒ x1 = x2 [ f is one-one] Thus, f is a function from R −   to R −   .
⇒ g(y) = h(y) 3 3
Therefore, the composite function fof exists.
Thus, g(y) = h(y) for all y ∈ B. Hence, g = h.
2 2
11. (i) Let (x, y) be the identity element in A. Then, fof : R −   → R −   and
(a, b) * (x, y) = (a, b) = (x, y) * (a, b) 3 3
for all (a, b) ∈ A  4x + 3 
( fof )(x ) = f ( f (x )) = f 
⇒ (ax, b + ay) = (a, b) = (xa, y + bx)  6 x − 4 
4x + 3
for all a, b ∈ A 4⋅ +3
16 x + 12 + 18 x − 12 34 x
⇒ (ax, b + ay) = (a, b) and (a, b) = (xa, y + bx) = 6x − 4 = = =x
4x + 3 24 x + 18 − 24 x + 16 34
for all a, b ∈ A 6⋅ −4
6x − 4
⇒ ax = a and b + ay = b and xa = a, y + bx = b 2
for all a, b ∈ A ⇒ fof = Identity function on R –  
⇒ f is invertible and f –1 = f. 3
⇒ x = 1, y = 0

80 MATHEMATICS TODAY | MAY‘18


14. Let f(x1) = f(x2) ⇒ x12 + 4 = x22 + 4 (iii) Transitivity : Consider points P, Q and T
⇒ x12 = x22 ⇒ x12 – x22 = 0 having coordinates (0, 0), (1.5, 0) and (3.2, 0). Now,
⇒ (x1 – x2)(x1 + x2) = 0 ⇒ x1 – x2 = 0 the distance between P and Q is 1.5 units which is
[ (x1 + x2) ≠ 0] less than 2 units and the distance between Q and
⇒ x1 = x2. T is 1.7 units which is also less than 2 units. But,
\ f is one-one. the distance between P and T is 3.2 units which
Now, y = x2 + 4 ⇒ x = y − 4 is not less than 2 units. Thus, (P, Q) ∈ R and
(Q, T) ∈ R but (P, T) ∉ R. So, R is not transitive
For each y ∈ [4, ∞) there exists x = y − 4 in
on S.
R+ such that f (x ) = f ( y − 4 ) = ( y − 4 )2 + 4 17. We have
= (y – 4) + 4 = y.
(1 + 1) 2 2
\ f is onto. f (1) = = = 1 and f (2) = = 1
Thus, f is one-one onto and therefore invertible. 2 2 2
Now, y = f(x) ⇒ y = x2 + 4 Thus, f(1) = f(2), while 1 ≠ 2.
⇒ x = y−4 \ f is many-one.
In order to show that f is onto, consider an arbitrary
⇒ f −1 ( y ) = y − 4 . element n ∈ N.
If n is odd, then (2n – 1) is odd and
\ f –1 : [4, ∞) → R+ such that f –1(y) = y − 4.
(2n − 1 + 1) 2n
15. (i) Reflexivity : Let a be an arbitrary element of N. f (2n − 1) = = = n.
2 2
Then, a = (a × 1) shows that a is a multiple of a. If n is even, then 2n is even and
\ (a, a) ∈ R ∀ a ∈ N.
2n
So, R is reflexive. f (2n) = = n.
(ii) Symmetry : Clearly, 6 and 2 are natural 2
Thus, for each n ∈ N (whether even or odd) there
numbers and 6 is a multiple of 2.
\ (6, 2) ∈ R. exists its pre-image in N.
But, 2 is not a multiple of 6. \ f is onto.
\ (2, 6) ∉ R. Hence, f is many-one onto.
Thus, (6, 2) ∈ R but (2, 6) ∉ R. 18. Let f(n1) = f(n2).
Hence, R is not symmetric. Case 1 : When n1 is odd and n2 is even, then
(iii) Transitivity : Let a, b, c ∈ N such that (a, b) ∈ R f(n1) = f(n2)
and (b, c) ∈ R ⇒ n1 – 1 = n2 + 1
⇒ (a is a multiple of b) and (b is a multiple of c) ⇒ n1 – n2 = 2
⇒ a = bd and b = ce for some d, e ∈ N If n1 is odd and n2 is even, then (n1 – n2) ≠ 2.
⇒ a = (ce)d ⇒ a = c(ed) Thus, we arrive at a contradiction.
⇒ a is a multiple of c So, in this case, f(n1) ≠ f(n2).
⇒ (a, c) ∈ R Similarly, when n1 is even and n2 is odd, then
\ (a, b) ∈ R and (b, c) ∈ R ⇒ (a, c) ∈ R. f(n1) ≠ f(n2).
Hence, R is transitive. Case 2 : When n1 and n2 both are odd, then
16. (i) Reflexivity : For any point P in set S, we find f(n1) = f(n2)
that distance between P and itself is 0 which is less ⇒ n1 – 1 = n2 – 1
than 2 units. ⇒ n1 = n2.
Thus, (P, P) ∈ R for all P ∈ S. Case 3 : When n1 and n2 both are even, then
So, R is reflexive on S. f(n1)= f(n2)
(ii) Symmetry : Let P and Q be two points in S such ⇒ n1 + 1 = n2 + 1
that (P, Q) ∈ R. ⇒ n1 = n2.
⇒ Distance between P and Q is less than 2 units. Thus, from all the cases, we get f(n1) = f(n2) ⇒ n1 = n2
⇒ Distance between Q and P is less than 2 units \ f is one-one.
⇒ (Q, P) ∈ R Now, we show that f is onto.
So, R is symmetric on S. Let n ∈ W.

MATHEMATICS TODAY | MAY‘18 81


Case 1 : When n is odd (21 y + 28 + 20 y − 28)
In this case, (n – 1) is even =
(15 y + 20 − 15 y + 21)
and f(n – 1) = (n – 1) + 1 = n. ...(i)
Case 2 : When n is even 41 y
= = y = I B ( y ).
In this case, (n + 1) is odd 41
and f(n + 1) = (n +1) – 1 = n. ...(ii) \ (fog) = IB.
Thus, each n ∈ W has its pre-image in W. Hence, (gof) = IA and (fog) = IB.
\ f is onto.
20. Clearly, R1 and R2 are subsets of X × X. In order
Thus, f is one-one onto and hence invertible.
Clearly, we have to prove that R1 = R2, it is sufficient to show that
R1 ⊂ R2 and R2 ⊂ R1.
(n − 1), when n is odd
f –1(n) =  [using (i) and (ii)] Now, the difference between any two elements
(n + 1), when n is even. of each of the sets {1, 4, 7}, {2, 5, 8} and
19. Let x ∈ A. Then, {3, 6, 9} is a multiple of 3.
 7x + 4  Let (x, y) be an arbitrary element of R1. Then,
(gof ) (x) = g[f(x)] = g 
 5x − 3  (x, y) ∈ R1
 7x + 4 
3 +4 ⇒ x – y is divisible by 3.
 5x − 3  (21x + 12 + 20 x − 12)
= = ⇒ x – y is a multiple of 3.
 7x + 4  (35x + 20 − 35x + 21) ⇒ {x, y} ⊂ {1, 4, 7} or {x, y} ⊂ {2, 5, 8}
5 −7
 5x − 3  or {x, y} ⊂ {3, 6, 9}
41x ⇒ (x, y) ∈ R2
= = x = I A (x ).
41 So, R1 ⊂ R2 ...(i)
\ (gof) = IA. Now, let (a, b) be an arbitrary element of R2. Then,
Again, let y ∈ B. Then, (a, b) ∈ R2
 3y + 4  ⇒ {a, b} ⊂ {1, 4, 7} or {a, b} ⊂ {2, 5, 8}
(fog) (y) = f [g(y)] = f 
 3y + 4   5 y − 7  or (a, b} ⊂ {3, 6, 9}
7 +4 ⇒ a – b is divisible by 3.
 5 y − 7  ⇒ (a, b) ∈ R1
=
 3y + 4  So, R2 ⊂ R1 ...(i)
5 −3
 5 y − 7  From (i) and (ii), R1 = R2


82 MATHEMATICS TODAY | MAY‘18


Contd. from Page no. 18 ∞
+n ∫ (sin x )n−1(− sin x ) + (cos x )(n − 1)cos x(sin x )n−2  e − x dx
 a 2 − h2  0
Solving we get the radial centre as  h, . ∞
 k  = n ∫  −(sin x )n + (n − 1)(1 − sin2 x )(sin x )n−2  e − x dx
Now in order to find the orthocentre we have to find 0
the equation of altitudes AD and BE. n(n − 1) 101 l10 101 10 × 9
= ln−2 , = × =1
AD is x = h and BE is n +12 90 l8 90 102 + 1
h+a
y −0= − ( x − a) 100 + n
k 45. (2) : A.M. = , G.M. = 100 × n ,
 k  2
 BE ⊥ AC , slope of AC is h + a  200n
H.M. =
100 + n
 a 2 − h2 
Solving, we get orthocentre  h, . Let n = 4k2 as for A.M and G.M we need a even perfect
 k 
square
(D) The function is defined for all values of x for 800k 2 200k 2
|x| – x > 0 ⇒ |x| > x. If x > 0, |x| = x \ H.M. = =
\ All positive values of x shall not be in domain of 100 + 4k 2 25 + k 2
x. But if x is negative, then |x| = –x > x 200k 2
⇒ 2x < 0, x < 0 \ (–∞, 0). Now, 100 < ≤ 500 ⇒ 2500 + 100k2 ≤ 200k2
2
25 + k
A B ⇒ 2500 ≤ 100 k2
43. (1) : In given DABC both and lie strictly
2 2 and 200k2 ≤ 12500 + 500k2 ⇒ –300k2 ≤ 12500
π
between  0,  and sinx is always increasing in ⇒ k2 = 25, 36, 49, 64, 81, 100, H.M. is an integer for
 2 k2 = 100 only
 π
 0,  whereas cosx is always decreasing throughout ⇒ n = 4k2 = 4 × 100 = 400.
2
 π
 0,  .
2
A B A B RAJASTHAN at
So if > ⇒ sin > sin ⇒ x1 > x2
2 2 2 2 • Competition Book House - Alwar Ph: 0144-2338391; Mob: 9460607836

1 1 • Nakoda Book Depot - Bhilwara Ph: 01482-239653; Mob: 9214983594


and 3 > 4 . Now, x12007 x 42006 = x22007 x32006 is not valid. • Alankar Book Depot - Bhiwadi Ph: 01493-222294; Mob: 9414707462
x x • Uttam Pustak Kendra - Bikaner Mob: 8955543195, 9414572625
A B • Yadu Books & Stationers - Bikaner Mob: 9251653481
Similarly for < • Goyal Books & Stationers - Jaipur
2 2 Ph: 0141-2742454; Mob: 9414326406, 9929638435
A B 1 1 • India Book House - Jaipur
sin < sin ⇒ x1 < x2 and < . Again equality Ph: 0141-2314983, 2311191, 2651784; Mob: 9829014143, 9414079983
2 2 x3 x 4 • Ravi Enterprises - Jaipur
is not possible. Ph: 0141-2602517, 2619958, 2606998; Mob: 9829060694
A B • Shri Shyam Pustak Mandir - Jaipur Ph: 0141-2317972; Mob: 9928450717
Therefore only possible case is when = • Sarvodaya Book Stall - Jodhpur Ph: 0291-2653734, 35; Mob: 8107589141
2 2
1 1 • Bhandari Stationers - Kota
⇒ x1 = x2 and = . Ph: 0744-2327576, 2391678; Mob: 9001094271, 9829038758
x3 x 4
• Raj Traders - Kota Ph: 0744-2429090; Mob: 9309232829, 9214335300
Hence in this case DABC is isosceles with ∠ABC = ∠CAB • Vardhman Sports & Stationers - Kota
Mob: 9461051901, 9351581238, 9828139717

 e − x (sin x )n  ∞ n −1 −x
• Jhuria Book Shop - Sikar Mob: 9784246419,9460838235, 8432943550
44. (1) : ln =   + ∫ n(sin x ) cos xe dx • Popular Book Depot - Udaipur
 −1 0 0 Ph: 2442881, 0487-2329436, 2421467; Mob: 9388513335, 9847922545


 (sin x )n−1 cos xe − x 
= 0+n 
 −1 
0

MATHEMATICS TODAY | MAY‘18 83


MPP-1 Class XII

T his specially designed column enables students to self analyse


their extent of understanding of specified chapters. Give yourself
four marks for correct answer and deduct one mark for wrong answer.
Self check table given at the end will help you to check your
readiness.

Relations and Functions


Total Marks : 80 Time Taken : 60 Min.
Only One Option Correct Type 6. Let * be a binary operation on N defined by
a * b = a + b + 10 for all a, b ∈ N. The identity
 π π element for * in N is
1. The largest interval lying in  − ,  for which the
 2 2 (a) –10 (b) 0
2 x  (c) 10 (d) Does not exist
function f (x) = 4–x + cos–1  − 1 + log cos x is
2  One or More Than One Option(s) Correct Type
defined as
 π π  π 7. Let n be a fixed positive integer. Let a relation R be
(a)  − ,  (b) 0,  defined in I (the set of all integers) as follows: aRb
 4 2  2
iff n/(a – b), that is, iff a – b is divisible by n. Then,
 π π the relation R is
(c) [0, p] (d)  − , 
 2 2
2. Let f : R → R, g : R → R be two functions such that
f(x) = 2x – 3, g(x) = x3 + 5. The function (fog)–1(x) is FUN CT
FA
Do you think this is arbitrary?
equal to
The resulting angle always reduces to 9.
1/3 1/3
x +7
(a)  (b)  x − 7  360° 180° 90°
 2   2 
(3+6+0=9) (1+8+0=9) (9+0=9)

1/3 1/3
x −7
(c)  x − 2  (d) 
90°
180°
 7   2 
360°

3x − 2
3. If f (x ) = , then f(f(x)) =
2x − 3 45° 22.5° 11.25°
1 1 (4+5=9) (2+2+5=9) (1+1+2+5=9)
(a) (b) x (c) x2 (d)
x x2
45°
4. f : R → R, f(x) = x|x| is 22.5° 11.25°

(a) one-one but not onto


(b) one-one onto
(c) onto but not one-one 5.625°
(5+6+2+5=18)
(d) none of these (1+8=9)

5. Let A = {7, 8, 9, 10} and R = {(8, 8), (9, 9), (10, 10),
(7, 8)} be a relation on A, then R is 5.625°
(a) transitive (b) reflexive
(c) symmetric (d) none of these

84 MATHEMATICS TODAY | MAY ‘18


(a) reflexive (b) symmetric Matrix Match Type
(c) transitive (d) none of these
3
16. Let f, g : R → R be the function defined by
8. Let f (x) = x and g(x) = 3 . The values of a such that
x
f(x) = x2 + 1 and g(x) = 2[x] – 1, where [x] is the
g(f (a)) = f (g(a)) are largest integer ≤ x. Then match the following:
(a) 0, 2 (b) 1, 3 (c) 0, ± 3 (d) 1, ± 2
Column-I Column-II
9. Which of the following functions f : R → R are
bijective ? 1
P. ( gof )   1. 3
(a) f(x) = x sinx (b) f(x) = x – sin2x 2
(c) f (x) = x + x 2 (d) f(x) = – x + cos2 x 8
Q. ( fog )   2. 10
10. Let '*' be a binary operation on Q0 (set of all non- 3
ab 3
zero rational numbers) defined by a * b = for all R. ( fogof )   3. 2
a, b ∈ Q , then 4 4
0
(a) identity element of * is 4 2
16 S. ( gofog )   4. 1
(b) inverse of an element a in Q0 is 3
(c) identity element of * is 8 a
4 P Q R S
(d) inverse of an element a in Q0 is (a) 1 3 4 2
a (b) 2 4 3 1
11. Let '*' be a binary operation on N given by (c) 4 2 3 1
a * b = L.C.M (a, b) for all a, b ∈ N, then (d) 3 1 2 4
(a) * is commutative (b) * is associative
(c) identity element of * is 1 Integer Answer Type
(d) identity element of * is 2 17. If f(x) is an even function and satisfies the relation
x −1 1
12. The function f (x) = 2 is x 2 f (x ) − 2 f   = g (x ) , where g(x) is an odd
x x
(a) one-one in (2, ∞) (b) one-one in (0, 1) function, then f(10) equals
(c) one-one in (–∞, 0) (d) one-one in (1, 2)
13. With reference to a universal set, the inclusion of a 18. If f : R → R, g : R → R and h : R → R is such that
subset in another, is a relation which is f(x) = x2, g(x) = tan x and h(x) = log x, then
(a) non Symmetric (b) equivalence π
(c) reflexive (d) none of these the value of [ho(gof)] (x), if x = will be
2
Comprehension Type 19. If f is an invertible function defined as
Consider the function f : R → (0, ∞) defined by 3x − 4
f (x ) = , then f –1(1) is
f(x) = 2x + 2|x| 5
(2 x − 1)
14. f(x) is 20. Let f : N → R : f (x ) = and
2
(a) one-one onto (b) one-one into
g : Q → R : g(x) = x + 2 be two functions, then
(c) many-one onto (d) many-one into
3
15. The number of solutions of the equation f(x) = ln p is ( gof )   is
2
(a) 0 (b) 1 (c) 2 (d) 3 
Keys are published in this issue. Search now! J
Check your score! If your score is
> 90% EXCELLENT WORK ! You are well prepared to take the challenge of final exam.

No. of questions attempted …… 90-75% GOOD WORK ! You can score good in the final exam.

No. of questions correct …… 74-60% SATISFACTORY ! You need to score more next time.
Marks scored in percentage …… < 60% NOT SATISFACTORY! Revise thoroughly and strengthen your concepts.

MATHEMATICS TODAY | MAY ‘18 85


86 MATHEMATICS TODAY | MAY ‘18
MATHEMATICS TODAY MAY ’18 87
88 MATHEMATICS TODAY MAY ’18
MATHEMATICS TODAY MAY ’18 89
90 MATHEMATICS TODAY MAY ’18
Registered R.N.I. No. 40700/1983 Published on 1st of every month Postal Regd. No. DL-SW-01/4045/18-20
Lic. No. U(SW)-29/2018-20 to post without prepayment of postage N.D.P.S.O.N.D-1 on 2-3rd same month

You might also like